Chest Pain, Headache and Neurologic, SAEM Peds, SAEM - Procedures, Psych Emergencies, Derm, SAEM Tox, Infxn, Optho, Foreign Bodies, SAEM AMS, 2017 CV, 2017 trauma, SAEM MISC, SAEM - Shock and Sepsis, Environment and Endocrine, Pulm Emergencies

Réussis tes devoirs et examens dès maintenant avec Quizwiz!

A 22 year old female presents to the emergency department with a "funny feeling" in her chest. She has had similar episodes but never lasting as long as the current episode (3-hour duration). Her heart rate is 200, blood pressure is 128/68, respiratory rate is 20 and her pulse oximetry is 96%. Her EKG is shown in the Figure. The best treatment option for this patient is: Answers: 1.adenosine 2.lidocaine 3.cardioversion 4.verpamil

1

A 29 year old woman is found seizing by her husband and is rushed to the emergency department. On presentation, she is noted to have a BP of 162/112, is still seizing, and looks puffy all over. Her husband tells you that they are expecting their first child in a few months. Which of the following is the next best step in this patient's care? Answers: 1.Control the seizures with magnesium sulfate. 2.Draw blood to check CBC, LFT's, BUN, and creatinine. 3.Notify the labor floor that the patient is in the emergency department. 4.Perform a CT scan of head if seizures persist. 5.Start hydralazine to decrease the patient's blood pressure.

1

A 75 year old female is brought the to emergency department by a family member with a history of progressive forgetfulness and confusion. She has a history of dementia. The most common cause of dementia in the elderly patient is: Answers: 1.Alzheimer's disease 2.Parkinson's disease 3.Pick's disease 4.Vascular dementia

1

A blunt trauma patient presents, transported by EMS from a motor vehicle collision, with inability to provide a history, due to alcohol intoxication. He has no signs of trauma on external evaluation, but he is hypotensive. An ED ultrasound is performed at the bedside, and is depicted in the figure. Given the patients' clinical condition and image seen, what is the most likely diagnosis? Answers: 1.Free intraabdominal fluid 2.Fat embolus from femur fracture 3.Rupturing abdominal aortic aneurysm 4.Ruptured gallbladder

1

A trauma patient resuscitated in the ED, has a post-tube thoracostomy computed tomography (CT) scan as depicted in the figure. What finding is present? Answers: 1.Persistent pneumothorax 2.Chest tube not within the thoracic cavity 3.Pericardial tamponade 4.Aortic rupture leading to a right hemothorax

1

An 8 year old female presents with a regular, narrow-complex SVT. You diagnose AV nodal reentrant tachycardia. Which pharmacologic agent would be most appropriate for initial management? Answers: 1.Adenosine 2.Digoxin 3.Diltiazem 4.Lidocaine

1

Choose the INCORRECT statement regarding thoracentesis from the anterior approach (needle decompression): Answers: 1.The recommended insertion site is the second intercostal space, midaxillary line. 2.A 14- to 20-gauge needle is inserted perpendicularly over the superior edge of the rib. 3.After the needle is inserted into the pleural space, a rush of air confirms the presence of a tension pneumothorax. 4.If a tension pneumothorax is confirmed via needle decompression, then a thoracostomy tube should be placed as soon as possible. 5.An upright chest X-ray should always be performed following a thoracentesis to confirm the successful relief of a tension pneumothorax and the absence of hemothorax or other complications.

1

Following a motor vehicle collision, in which of the following patients is an emergency department Caesarian section most likely indicated, assuming a fetus at 29-weeks gestation? Answers: 1.Mother pulseless and apneic for 2 minutes' duration and still in arrest; fetal heart tones of 100 beats per minute 2.Mother with abdominal pain, blood pressure of 80/40; fetal heart tones 100 beats per minute 3.Mother with vaginal bleeding, blood pressure 118/78; fetal heart tones 80 beats per minute 4.Mother with severe head trauma, blood pressure 170/90; fetal heart tones 120 beats per minute 5.Mother with gunshot wound to abdomen, blood pressure 96/42; fetal heart tones undetectable

1

The blood pressure at which malignant hypertension is defined as present is: Answers: 1.an elevated arterial pressure associated with end organ damage 2.an elevated arterial pressure that exceeds the patient's baseline by 33% 3.diastolic blood pressure of 110 or greater 4.systolic blood pressure of 170 or greater 5.systolic blood pressure of 180 or greater

1

The chest X-ray in the Figure was taken in an intoxicated patient who is conversant, but an unreliable historian. The X-ray findings are best described as indicating: Answers: 1.Esophageal foreign body 2.Intratracheal foreign body 3.mediastinitis from esophageal perforation 4.normal chest

1

The patient depicted in the figure presents to the ED just after sustaining a pellet-gun wound to the right eye. What do the arrows most likely indicate? Answers: 1.hyphema 2.hypopion 3.iritis 4.keratitis 5.pterygium

1

What type of rhythm disturbance is seen in the EKG below? Answers: 1.first degree atrioventricular (AV) block 2.second degree AV block type I 3.second degree AV block type II 4.third degree heart block

1

Where are the normally dominant pacemaker cells of the heart found? Answers: 1.sinoatrial node 2.atrioventricular node 3.bundle of His 4.accessory pathway of Kent

1

Which of the ocular findings below is associated with hypertension? Answers: 1.arterio-venous nicking 2.cherry red spot 3.increased cup-to-disk ratio 4.retinal nevus 5.Roth spots

1

With regard to specific causes of hypertension, which of the following is true? Answers: 1.Hypertensive encephalopathy is more likely than hypertensive stroke in patients whose mental status changes are reversible 2.Hypertensive encephalopathy causes adverse outcomes over days or weeks, rather than hours 3.Patients with stroke syndromes must have blood pressure normalized as quickly as possible to reduce the risks of worsening neurological deficit 4.Laboratory analysis is rarely useful in cases of confirmed pediatric hypertension 5.Laboratory analysis is rarely useful in cases of confirmed hypertension in pregnant patients

1

For which of the following cases is activated charcoal therapy most appropriate? Answers: 1. Acetaminophen overdose 2. Drain cleaner ingestion 3. Iron supplement overdose 4. Lithium overdose

1. Acetaminophen Overdose

A patient presents after an unknown ingestion. Her initial electrocardiogram (EKG) is shown in the image. Based on the EKG, an overdose with which of the following medications would be most likely? (EKG DEMONSTRATES QRS PROLONGATION; QRS>100) 1.Amitriptyline 2.Clonidine 3.Ibuprofen 4.Nifedipine

1. Amitriptyline

A teenager presents one hour after ingesting a "handful" of acetaminophen tablets. Which of the following statements is TRUE? 1.An acetaminophen level drawn at hour four dictates need for antidotal therapy. 2.Serial liver function tests are indicated in all acetaminophen ingestions. 3.Renal sequelae are expected. 4.The intravenous formulation of N-acetylcysteine is safer than oral N-acetylcysteine

1. An acetaminophen level drawn at hour four dictates need for antidotal therapy.

A 2 year old child presents with an overdose of her mother's iron containing multivitamins. What antidote should you consider for iron toxicity? 1.Deferoxamine 2.Glucagon 3.Methylene blue 4.Pyridoxine

1. Deferoxamine; Deferoxamine binds directly to free iron and thus is the antidote for iron toxicity. It is given intramuscularly or intravenously and often causes the patient's urine to turn color (vin rosé urine). Methylene blue is an antidote for methemoglobinemia. N-acetylcysteine is the antidote for acetaminophen. Pyridoxine is the antidote for isoniazid toxicity and glucagon can serve as an antidote for beta blocker, calcium channel blocker, or insulin overdoses.

A 26 year-old presents with agitation, chest pain and a heart rate of 142 bpm after intranasal cocaine use. The EKG is normal except for sinus tachycardia. What is the best medication to use in this situation? 1.Lorazepam 2.Diphenhydramine 3.Esmolol 4.Haloperidol

1. Lorazepam; Benzodiazepines are the treatment mainstay for cocaine toxicity. Lorazepam and diazepam can be titrated to treat the symptoms of agitation and increased adrenergic tone common to patients with cocaine toxicity. Beta blockers should not be administered due to a potential for unopposed alpha-adrenergic stimulation and resultant hypertension. Haloperidol and diphenhydramine can contribute to the hyperthermia common to patients with cocaine toxicity.

A 12-day-old term infant presents for evaluation of vomiting blood-streaked emesis once after feeding. She is well appearing and well hydrated with normal vital signs and an unremarkable exam. She is breast-fed. What should the physician do next? Answers: 1.Ask the mother if she has any bleeding from her nipples. 2.Begin a septic workup. 3.Start a workup for a bleeding diathesis. 4.Start an H2 blocker for reflux.

1.Ask the mother if she has any bleeding from her nipples.

A 5 year old child was eating an almond when he experienced sudden, intermittent bouts of choking and wheezing. Assuming this child aspirated an almond, which of the following is most likely to be seen on chest X-ray? Answers: 1.Atelectasis of the affected lung 2.Diaphragmatic flattening of the non-affected lung 3.Foreign body in the shape of an almond 4.Hypoinflation of the non-affected lung

1.Atelectasis of the affected lung

A 17 year old boy injured his right shoulder playing football. He tried to arm-tackle a player when his right arm was pulled away from his body and back (abducted and extended). He felt a sudden pain in his shoulder. He presents to the emergency department holding his arm in slight abduction and external rotation by his good arm. He has severe pain with adduction or internal rotation. What is the most common fracture associated with this injury? Answers: 1.Compression fracture of the posteriolateral aspect of the humeral head (Hill-Sachs deformity) 2.Avulsion fracture of the greater tuberosity of the humerus 3.Clavicle fracture 4.Acromioclavicular joint separation 5.Fracture of the anterior glenoid lip (Bankart's fracture)

1.Compression fracture of the posteriolateral aspect of the humeral head (Hill-Sachs deformity)

Which of the following statements regarding intraosseous (IO) access is INCORRECT? Answers: 1.Drug delivery by endotracheal route is preferred over the intraosseous route 2.Anterior compartment syndrome is a recognized complication of IO access 3.Tibial fracture is a recognized complication of IO access 4.Long bone fracture is a contraindication of IO access 5.Marrow and fat emboli are recognized complications of IO access

1.Drug delivery by endotracheal route is preferred over the intraosseous route

What is the most common heart rhythm seen in pediatric arrest? Answers: 1.bradycardia 2.ventricular fibrillation 3.paroxysmal atrial tachycardia 4.Wolff-Parkinson-White syndrome 5.atrial fibrillation Points achieved: 0Incorrect

1.bradycardia

Which of the following describes the most commonly indicated initial approach in neonatal resuscitation? Answers: 1.dry, warm, position, suction, stimulate 2.chest compressions 3.medications 4.establish effective ventilation 5.oxygen

1.dry, warm, position, suction, stimulate

In the post-arrest setting, which of the following is the drug of choice in treating hypotension in a child: Answers: 1.epinephrine infusion 2.dobutamine infusion 3.dobutamine bolus 4.dopamine infusion 5.nitroprusside infusion

1.epinephrine infusion

What is the most common category of shock in the pediatric population? Answers: 1.hypovolemic 2.distributive 3.cardiogenic 4.obstructive 5.dissociative

1.hypovolemic

n a newborn, bradycardia is most commonly an indicator of: Answers: 1.hypoxemia 2.hypoglycemia 3.hyperglycemia 4.hypothermia 5.hyperthermia

1.hypoxemia

What is the normal range for intraocular pressure in humans (in mmHg)? Answers: 1.0-10 2.10-20 3.20-30 4.30-40 5.40-50

10-20

A 32 year old female is shot with a 38-caliber pistol at close range in the right anterior chest. She presents to the emergency department intoxicated and yelling. Her vitals include a pulse of 92, blood pressure of 134/84, and oxygen saturation of 97%. She has clear breath sounds bilaterally. The entrance wound is just above the right breast and an exit wound is noted in the right axilla. What is the most appropriate management of this patient? Answers: 1.IV access, portable chest X-ray, tube thoracostomy, and exploratory thoracotomy in the OR to search for cardiac or pulmonary vascular injury 2.IV access, endotracheal intubation and simultaneous placement of a right chest tube, bedside ultrasound, portable chest X-ray, and admission to the ICU if stable 3.IV access, portable chest X-ray, right chest tube placement if X-ray shows a pneumo- or hemothorax, admission to the ICU for observation 4.IV access, endotracheal intubation, CT scan of chest to look for pneumo- or hemothorax, or injuries to the heart or great vessels 5.IV access, endotracheal intubation, emergency department thoracotomy to search for cardiac or pulmonary vascular injury

2

A 41 year old man is seen in the emergency department after a street fight where he punched another man in the mouth. He has a small, jagged laceration over the dorsum of the metacarpophalangeal joint of his right hand. The wound is irrigated copiously, tetanus is given, and the wound is left open to heal by secondary intention because of the infection risk. In addition to treating Streptococcus and Staphylococcus species, antibiotics must also treat which other bacterial species? Answers: 1.Pasteurella multocida 2.Eikenella corrodens 3.Escherichia. coli 4.Actinomyces israelii

2

A 70 year old male presents with lightheadedness. He is noted to be bradycardic. His EKG below reveals: Answers: 1.first degree AV block 2.second degree AV block Mobitz Type I 3.second degree AV block Mobitz TypeII 4.complete heart block

2

A previously healthy 25 year old female arrives at the emergency department with 3 days of headache, nausea, palpitations, and diaphoresis. She initially presented 2 days ago to the hospital's walk-in clinic, where her blood pressure was found to be moderately elevated. At her clinic visit, the initial evaluation for end-organ damage was negative. In the emergency department, assessment of tests sent from the clinic visit is noteworthy for a normal TSH, normal head CT scan, and markedly elevated urine metanephrine levels. If this woman were to require emergency department therapy for hypertension, which of the following agents should be AVOIDED: Answers: 1.alpha-adrenergic receptor blocker 2.beta-adrenergic receptor blocker 3.calcium channel blocker 4.ACE-inhibitor 5.nitroprusside

2

An 18 year old hockey player is hit in the mouth with a puck, fracturing a maxillary canine tooth. He brings the severed piece of tooth with him. On physical exam, the tooth is fractured halfway between the tip and the gumline. The root of the tooth is still firmly intact. The exposed fracture site has a yellowish tinge without blood. Of the following choices, which is the most appropriate management for this patient? Answers: 1.Replace fractured piece and place acrylic splint 2.Immediate dental consult to avoid abscess formation 3.Application of calcium hydroxide, placement of aluminum foil, and dental follow-up 4.Placement of tooth fragment in saline gauze, outpatient dental follow-up 5.No specific treatment required

2

In a patient with a suspected ruptured globe from penetrating trauma to the eye, all of the following should be performed EXCEPT: Answers: 1.administration of broad spectrum antibiotic therapy 2.ascertainment of intraocular pressure via tonometry 3.ascertainment of tetanus status 4.ophthalmology consultation 5.visual acuity assessment

2

In which of these patients is emergency department thoracotomy indicated? Answers: 1.Unbelted driver in a high-speed motor vehicle crash who loses his pulse while being extricated, and arrives at the E.D. after a 45-minute transport 2.Patient with a gunshot wound to the chest who upon arrival is unconscious and pulseless, with a systolic blood pressure of 60 3.Pedestrian struck with massive pelvic fractures who loses pulses and blood pressure at the scene 4.Patient with stab wound to the anterior chest who is dyspneic with an oxygen saturation of 80% and a blood pressure of 168/102 5.All of the above should undergo emergency department thoracotomy.

2

Of the following choices, which is the most likely diagnosis based on the EKG in the Figure? Answers: 1.left bundle-branch block 2.right bundle-branch block 3.anteroseptal myocardial infarction 4.Wolff-Parkinson-White syndrome

2

Of the options below, the therapy best for symptomatic 3rd degree heart block is: Answers: 1.lidocaine 2.transcutaneous pacer 3.atropine 4.oxygen 5.cardioversion

2

Using the rule of 9s, what is the approximate burn surface area of a victim who has sustained second-degree burns to the anterior chest and anterior area of both arms? Answers: 1.0.2 2.0.25 3.0.3 4.0.35

2

Which diagnosis is suggested by the EKG shown in the Figure? Answers: 1.digoxin overdose 2.right-ventricular ischemia 3.pericarditis 4.dextrocardia

2

Which is the most common associated neurological finding with a distal radius fracture? Answers: 1.Wrist drop 2.Decreased sensation over the thenar eminance 3.Weakness of finger adduction 4.Decreased sensation over the hypothenar eminance 5.Weakness with flexion at the finger MCP joints

2

Which of the following is an accurate statement? Answers: 1.Diagnostic peritoneal lavage usually cannot identify the presence of hemoperitoneum. 2.Diagnostic peritoneal lavage cannot determine the etiology of hemoperitoneum. 3.Bedside ultrasound can reliably determine the etiology of hemoperitoneum. 4.Bedside ultrasound can image the retroperitoneum. 5.Bedside ultrasound is the test of choice for diagnosing solid organ injury.

2

Which of the following statements regarding psychotic behavior is true? Answers: 1.Brief psychotic episodes, often precipitated by events such as death of a loved one, can be characterized by extremely bizarre behavior and speech 2.Delusions are defined as false beliefs that are not amenable to arguments or facts to the contrary 3.Delusional disorder usually results in impairment in daily functioning 4.Schizophreniform disorder is present when a patient meets the diagnostic criteria for schizophrenia but the process has been present for less than one year

2

You are practicing in a trauma center a receive a call from an outlying facility that they would like to transfer a male patient to you with a spinal cord injury after significant flexion and compression of the vertebral body. What does this injury pattern tell you about the patient's symptoms? Answers: 1.The patient likely disproportionately greater weakness in the lower extremities (as compared to the upper extremities) 2.The patient likely has paralysis and loss of sensation to pain and temperature bilaterally below the lesion 3.The patient likely has symptoms on only one side of the his body 4.Patients with anterior cord syndromes have only sensory symptoms

2

You need to treat an adult with no past medical history, who presents with a hypertensive emergency. You have access to all of the following agents. Which of the following is the preferred agent and initial dose? Answers: 1.clonidine PO 0.1 mg 2.esmolol IV 100-500 mic/kg load 3.hydralazine IM 0.1-0.2 mg/kg 4.labetolol IV 0.2-1.0 mg/kg bolus 5.metoprolol PO 10 mg

2

Parents bring in their 13 year old girl two hours after she ingested a large amount of Acetaminophen in suicide attempt. She tearfully refuses to drink the activated charcoal. Which of the following is TRUE regarding your ability to administer the charcoal? 1. A court injunction is needed to force her to drink the charcoal 2. A nasogastric tube may be placed to facilitate treatment 3. You cannot force her to take the charcoal 4. You must get parental permission prior to treating her

2. A nastogastric tube mat be place to facilitate treatment

A 23 year old woman is dropped off by her boyfriend after an unknown overdose. You notice that she is has very large pupils and is sweating profusely. Her respiratory rate, blood pressure and heart rate are elevated. Which of the following is the most likely agent to have caused her symptoms? 1. Jimson weed 2. Cocaine 3. Heroin 4. Insulin

2. Cocaine; cocaine has sweating, jimson weed would have anhidrosis (hot as a hare, dry as a bone, mad as a hatter, red as a beet, blind as a bat)

An 84 year-old with a history of congestive heart failure is brought in by his family for vomiting and diarrhea. He also complains that things "have weird colors". He has been having odd palpitations but cannot describe them further. His family expresses their concern that he has not been taking his medications correctly. Given his presenting symptoms, which medication are you most concerned about? 1.Amiodarone 2.Digoxin 3.Diphenhydramine 4.Metoprolol

2. Digoxin; Digoxin toxicity classically presents as weakness, fatigue,nausea/vomiting/diarrhea, confusion, and a visual disturbance hallmarked byyellow/green halos around objects.

A 3 year old girl presents after accidentally ingesting an alkali drain cleaner. Which of the following statements regarding her management is true? 1.Activated charcoal should be administered 2.Endoscopy is useful in the assessment of injury 3.Gastric lavage should be performed immediately to reduce gastric injury 4.Neutralization therapy using a strong acid is warranted

2. Endoscopy is useful in the assessment of injury

With which of the following substances is acute withdrawal most likely life threatening? Answers: 1. Cocaine 2. Ethanol 3. Heroin 4. Lithium

2. Ethanol

A mother brings in her 4 year old child who was happily eating "blackberries" from weeds in the garden and is now acting strangely. She has identified them as Belladonna from a quick internet search. Which physical examination finding might you also expect to find in this child? 1. Diaphoresis 2. Flushed skin 3. Miosis 4. Urinary incontinence

2. Flushed Skin, anticholinergics (hot as a hare, dry as a bone, mad as a hatter, red as a beet, blind as a bat)

A 17 year-old has presented after taking a large amount of nortriptyline prescribed for migraine prophylaxis. Clinically, you take care of stabilizing her and initiate appropriate treatment. After reviewing reference materials you calculate that she has taken a potentially lethal dose of this tricyclic antidepressant. Which of the following would you expect to see on her electrocardiogram? 1.Compacted QT intervals 2.Prolonged QRS intervals 3.Prolonged PR intervals 4.Right bundle branch block

2. Prolonged QRS intervals

A 27 year old woman is brought into the emergency department by her roommate 30 minutes after ingesting a bottle of aspirin in a suicide attempt. Which of the following acid-base disorders is most likely to be present in this patient? 1.Respiratory acidosis due to somnolence causing decreased respiratory drive 2.Respiratory alkalosis due to stimulation of the respiratory center and increased CO2 production 3.Primary respiratory acidosis with compensatory metabolic alkalosis 4.Primary metabolic acidosis with compensatory respiratory alkalosis

2. Respiratory alkalosis due to stimulation of the respiratory center and increased CO2 production; key is 30 mins! few hours for metabolic acidosis

Approximately what inside diameter size endotracheal tube is appropriate for an 8 year old child? Answers: 1.4 mm 2.6 mm 3.8 mm 4.10 mm 5.12 mm

2.6 mm

All of the following are true regarding chest compressions in the infant EXCEPT: Answers: 1.An appropriate position for performing chest compressions is to encircle the chest with both hands and place the thumbs side by side on the sternum. 2.Chest compressions should be initiated whenever an infant's heart rate is less than 60 bpm. 3.Compressions should be performed at a rate of 90 per minute. 4.Chest compressions should be accompanied by a ventilatory rate of 30 per minute. 5.Correct depth of compressions is one-third the anteroposterior diameter of the chest.

2.Chest compressions should be initiated whenever an infant's heart rate is less than 60 bpm.

A 2 year old is brought in to the emergency department by his mother for difficulty breathing. The mother thinks he might have swallowed or aspirated something. In regards to this patient, which of the following is TRUE regarding foreign body aspiration or ingestion? Answers: 1.Large objects in the upper airway typically present with mild symptoms 2.Objects lodged in the proximal airway have the worst prognosis 3.Small objects in the lower airways typically present with the most severe symptoms 4.The most difficult objects to remove are stiff, non-conformable objects

2.Objects lodged in the proximal airway have the worst prognosis

A 7 year old boy falls off his bike onto his outstretched arm and sustains a supracondylar fracture. The fracture originates in the metaphysis and a portion of it extends into the physis (growth plate) without extending through to the epiphysis. How is this fracture classified? Answers: 1.Salter I 2.Salter II 3.Salter III 4.Salter IV 5.Salter V

2.Salter II

Which of the following pairs of maneuvers are consistent with current recommendations for emergency care for a choking 5-month-old infant? Answers: 1.back blow then blind finger sweep 2.back blow then chest thrust 3.chest thrust then Heimlich maneuver 4.Heimlich maneuver then blind finger sweep 5.Heimlich maneuver then chest thrust

2.back blow then chest thrust

All of the following are generally accepted indications for endotracheal intubation of the pediatric trauma patient, EXCEPT: Answers: 1.respiratory failure from hypoxia or hypoventilation 2.gastric distension due to excessive volume or rate of ventilation impairing ventilatory function 3.GCS score less than or equal 9, to secure airway and provide controlled hyperventilation 4.any trauma patient in decompensated shock and resistant to initial fluid resuscitation 5.any inability to ventilate by bag-valve-mask methods or the need for prolonged control of the airway

2.gastric distension due to excessive volume or rate of ventilation impairing ventilatory function

A 22 year old running back is struck from behind by a 300-pound lineman. The blow occurs below the knee as his foot is firmly planted and two other linemen are holding his upper body. He presents to the emergency department with gross anterior dislocation of the tibia on the femur. His foot is cool and pale, and dorsalis pedis and posterior tibial pulses are not detected by Doppler ultrasound. What is the most appropriate management for this patient? Answers: 1.Immediate orthopedic consultation without attempts to manipulate the knee 2.Immediate arteriography to assess for popliteal artery disruption 3.Immediate reduction in emergency department under conscious sedation without X-rays 4.Open reduction in OR with exploration of popliteal artery

3

A 24 year old woman is playing racquetball and sustains a direct blow from the ball to the right eye. She presents to the emergency department complaining of eye pain and double vision. On exam, her right eye does not track properly with upward gaze. This finding suggests which of the following injuries? Answers: 1.Inferior orbital rim fracture 2.Superior orbital rim fracture 3.Inferior orbital wall fracture 4.Ethmoid fracture 5.Zygomatic arch fracture

3

A 36 year old man is a restrained driver involved in a high speed MVA where his car is struck on the driver's side door with significant intrusion. His physical exam is significant for a large contusion on his left flank. His abdominal exam is benign and rectal exam reveals a normal prostate. A Foley catheter is placed with return of gross hematuria. Which test is indicated to evaluate for the presence of urologic injury? Answers: 1.CT abdomen / pelvis with IV contrast alone 2.Ultrasound of the bladder 3.CT abdomen / pelvis with IV and transurethral contrast 4.CT abdomen / pelvis without contrast 5.Ultrasound of the kidneys

3

A 46 year old man is brought in by EMS after a motor vehicle collision in which he was an unrestrained driver. Although he has no obvious injury to his head or neck, he complains of chest pain and appears very short of breath. His vital signs are: T 99.2 F, BP 85/57, HR 123, RR 36, SpO2 95% on non-rebreather. The CXR demonstrates a tension pneumothorax. Of the following, which is the most appropriate next step in this man's care? Answers: 1.Performance of a chest CT scan to further delineate the pathology 2.Placement of a chest tube followed by a chest xray to determine proper placement 3.Placement of a needle decompression device, followed by repeat CXR 4.Transfusion of 2 units of O-negative packed red blood cells

3

A 5 year old boy presents with fever and sore throat. On physical exam, the child has enlarged tonsils with exudates. Which of the following is true? Answers: 1.The etiology of the infection is most likely bacterial 2.Viruses rarely cause exudative pharyngitis 3.If the infection is bacterial, the primary role of antibiotic therapy is to prevent complications 4.Treatment of choice is azithromycin 5.Vaccination prevents recurrence

3

A 54 year old female presents with palpitations. She is otherwise asymptomatic. EKG shows atrial fibrillation. Vital signs are HR 130-150, BP 148/78, RR 16, T 36.7. What management intervention is most important to accomplish next? Answers: 1.Anticoagulation 2.Cardioversion 3.Pharmacologic ventricular rate control 4.Radiofrequency ablation

3

A 65 year old male is brought to the emergency department after he was found wandering on the street. He is unkempt and confused. A diagnosis of delirium, rather than dementia, is more likely if which of the following is true? Answers: 1.there has been a slow progressive loss of memory 2.the sleep-wake cycle is unaffected 3.there is a change in the level of consciousness 4.the confusion is worse during the day

3

A 65 year old male presents to the emergency department with chest pain. Cardiac monitoring shows a wide complex tachycardia. Past medical history is significant only for hypertension. His BP is 100/66, HR 144, RR 24, and T. 37.5. In addition to ongoing chest pain, he reports dyspnea. His level of consciousness is mildly decreased. Management should proceed on the assumption that he has what abnormal rhythm? Answers: 1.Sinus tachycardia with LVH 2.Supraventricular tachycardia with aberrancy 3.Ventricular tachycardia 4.Wolff-Parkinson-White syndrome with retrograde conduction

3

A patient presents to the ED after a fall with chest pain. A chest xray shows a rib fracture but no pneumothorax, and a chest CT is ordered. What is the most appropriate treatment for a small pneumothorax, detected only on chest CT, in a hemodynamically stable trauma patient? Answers: 1.Immediate needle decompression 2.Chest tube placement 3.100% oxygen 4.Heliox by face mask

3

An 80 year old nursing home patient is brought to the emergency department with an acute onset of confusion. Which of the following metabolic abnormalities is the most likely explanation? Answers: 1.hypocalcemia 2.hypokalemia 3.hypernatremia 4.hyperphosphatemia

3

Following a brawl at a local bar, a gentleman presents with an impressive right-sided periorbital ecchymosis. All of the following physical examination findings would suggest an orbital blowout fracture EXCEPT: Answers: 1.anesthesia of the right infraorbital region 2.diplopia with upward gaze 3.proptosis 4.right-sided epistaxis 5.right-sided infraorbital subcutaneous emphysema

3

Labetalol differs from propanolol in that labetalol is: Answers: 1.a mixed alpha1-agonist and beta-antagonist 2.a mixed alpha-antagonist and beta-agonist 3.an alpha-and beta antagonist 4.characterized by an elimination half-life of minutes rather than hours 5.selective for the alpha2-adrenergic receptor

3

The major abnormality on the image below is at which level? Answers: 1.C3/C4 2.C4/C5 3.C5/C6 4.C6/C7

3

The patient in the figure sustained minor blunt trauma to the eye, and has a normal head/orbital computed tomography (CT) scan. Ophthalmological examination is normal, other than the blood as shown in the figure (the blood does not cross the limbus). Of the choices below, which diagnosis is the most likely based upon the figure? Answers: 1.ruptured anterior chamber 2.hyphema 3.subconjunctival hemorrhage 4.globe rupture 5.foreign body

3

What does it mean to have 20/200 vision OD? Answers: 1.The patient's right eye sees at 200 feet what a normal eye sees at 20 feet. 2.The patient's left eye sees at 200 feet what a normal eye sees at 20 feet. 3.The patient's right eye sees at 20 feet what a normal eye sees at 200 feet. 4.The patient's left eye sees at 20 feet what a normal eye sees at 200 feet. 5.The intraocular pressure in the right eye is 20 and the intraocular pressure of the left eye 200.

3

What is the most common cause of death in Americans aged 20 to 40 years? Answers: 1.AIDS-related illness 2.Cancer 3.Trauma 4.Drug overdose 5.Pneumonia

3

Which factor is least reliable in differentiating between organic and inorganic causes of confusion? Answers: 1.acute versus chronic onset 2.Vital sign abnormalities 3.Presence of attention deficit 4.Signs of trauma

3

Which of the following cervical spine fractures is considered stable? Answers: 1.Flexion teardrop fracture 2.Bilateral facet dislocation 3.Transverse process fracture 4.Jefferson fracture of C1 5.Hangman's fracture of C2

3

Which of the following trauma patients can be managed conservatively without immediate laparotomy in the OR? Answers: 1.27 year old man with hemoperitoneum by bedside ultrasound; hypotensive 2.19 year old man with splenic laceration; peritoneal signs on exam 3.24 year old man with liver laceration; hemodynamically stable 4.30 year old man with a gunshot wound to the epigastrium 5.All of the above should go to the OR for exploratory laparotomy.

3

A 25 year old presents with an ingestion of acetaminophen 2 hours prior to arrival. Which of the following statements is TRUE? 1.AST of 32 and ALT of 27 from arrival labs indicate the absence of hepatotoxicity from this ingestion. 2.An acetaminophen level of 84 mg/dl from arrival labs necessitates use of n-acetylcysteine 3.Activated charcoal is indicated to treat this ingestion 4.Acetaminophen toxicity is predicted to occur at a dose of 20 mg/kg.

3. Activated charcoal is indicated to treat this ingestion; NAPQI -- the prime toxic mediator -- builds up when glutathione stores deplete and thus causes hepatotoxicity. The first stage of acetaminophen toxicity is largely asymptomatic. The toxic acetaminophen dose, when a single ingestion of nonsustained-release preparation is taken, is about 140 mg/kg. Therapy is guided by the Rumack-Matthew nomogram, provided the ingestion is an acute one involving nonsustained-release preparations. The antidote, N-acetylcysteine, prevents toxicity by inhibiting the binding of NAPQI to hepatocytes.

A 45 year-old is brought in 8 hours after a large overdose of his lithium. What is the best treatment method for this overdose? Answers: 1.Activated charcoal 2.Gastric lavage 3.Hemodialysis 4.Whole bowel irrigation

3. Hemodialysis; Dehydration, over-diuresis, and drug-drug interaction (particularly NSAIDs) are common precipitants of lithium toxicity in the patient chronically taking lithium

Severe lead toxicity can commonly result in which of the following clinical symptoms: 1.Constipation 2.Dermatitis 3.Memory loss 4.Stocking glove peripheral neuropathy

3. Memory loss; Lead toxicity affects a variety of systems. The central nervous system effects are many and range from encephalopathy and seizure to sleep disturbance and memory deficits. The peripheral nervous system can also be involved, with paresthesias and wrist drop being common

A young woman presents with an amitriptyline overdose. She is agitated and confused. In overdoses of this class of medications, an indicator of severe toxicity would include: Answers: 1. Elevated osmolar gap > 20 2. Metabolic acidosis with a pH < 7.25 3. Prolonged QRS interval 4. Serum amitriptyline level > 200 mcg/dl

3. Prolonged QRS interval

During opiate withdrawal which of the following symptoms would you expect to find? 1.Constipation 2.Pruritis 3.Tachypnea 4.Urinary retention

3. Tachypnea

A 72 year-old presents with an intentional overdose of a bottle of aspirin about 3 hours prior to presentation in the ED. Which of the following arterial blood gas results would you expect to come from this patient? 1.pH 7.14 pCO2 68 pO2 102 HCO3 23 2.pH 7.33 pCO2 48 pO2 58 HCO3 29 3.pH 7.45 pCO2 21 pO2 124 HCO3 14 4.pH 7.47 pCO2 31 pO2 96 HCO3 25

3. pH 7.45 pCO2 21 pO2 124 HCO3 14; metabolic acidosis with respiratory alkalosis

An 8 year old boy falls off his bike onto his outstretched hand with his elbow in extension. He presents to the emergency department with obvious anterior bowing of his distal humerus. His distal neurovascular exam is intact. The X-ray shows a transverse supracondylar humerus fracture with dorsal displacement and angulation of the distal fragment. Of the following, which is the most appropriate treatment? Answers: 1.Splinting the extremity in its current position, and arranging for orthopedic follow-up 2.Fracture reduction and casting by the E.D. physician 3.Orthopedic consultation for possible open reduction and internal fixation (ORIF) 4.CT scan of the elbow 5.Splinting and hospital admission for neurovascular checks

3.Orthopedic consultation for possible open reduction and internal fixation (ORIF)

Which of the following is NOT suggestive of perinatal asphyxia? Answers: 1.umbilical artery academia (pH < 7.00) 2.5-minute Apgar score of 5-7 3.neonatal neurological sequelae 4.multiorgan dysfunction 5.5-minute Apgar score of 3 or less

3.The pediatric airway is more posterior than the adult airway.

Which of the following methods is NOT used to establish the correct endotracheal tube size in a pediatric patient over 1 year of age: Answers: 1.(age in years + 16) divided by 4 2.approximation with child's little finger 3.age in months divided by 3 4.approximation with child's nares 5.body length using a Broselow emergency tape

3.age in months divided by 3

A 4-month-old infant presents with 1 day of episodic fussiness followed by normal activity. In the last several hours he has vomited 2-3 times. The patient is interactive and has a normal exam. While his parents receive discharge instructions he becomes fussy and passes a large reddish, jelly-like stool. What is the next course of action? Answers: 1.abdominal CT scan to rule out appendicitis 2.admit him for observation 3.arrange for a barium enema and surgical consultation 4.give him antibiotics for infectious diarrhea

3.arrange for a barium enema and surgical consultation

In pediatric resuscitation the following drugs may be given by the endotracheal route, EXCEPT: Answers: 1.epinephrine 2.naloxone 3.digoxin 4.atropine 5.lidocaine

3.digoxin

hich of the following vital signs is a cause for concern in the term newborn? Answers: 1.heart rate of 165 2.respiratory rate of 50 3.heart rate of 95 4.respiratory rate of 70 5.systolic blood pressure of 65

3.heart rate of 95

Low body temperatures in newborns can lead to severe physiologic consequences, which include all EXCEPT: Answers: 1.metabolic acidosis 2.increased oxygen consumption 3.hypoglycemia 4.apnea 5.hyperglycemia

3.hypoglycemia

All the following are signs of hypoxemia in a newborn, EXCEPT: Answers: 1.cyanosis 2.lethargy 3.tachycardia 4.unresponsiveness 5.bradycardia

3.tachycardia

A 23 year old man is stabbed in the anterior neck with a 3-inch knife during a street fight. At the scene, there is some bleeding, which is controlled with direct pressure. He presents to the emergency department breathing comfortably and in no distress. His pulse is 88, blood pressure 126/76, and oxygen saturation 99% on room air. There is a 1cm laceration 2cm above the right sternoclavicular junction, lateral to the trachea. There is mild oozing and no obvious underlying hematoma. There is no obvious subcutaneous air, and he has clear lung sounds. What is the most appropriate management for this patient? Answers: 1.Immediate operative exploration 2.Local wound exploration and discharge home if no significant injury identified 3.Local wound exploration and discharge home after 6-hour observation period 4.Angiography, esophogram, and admission for observation 5.CT scan of the neck and discharge home after 6 hours of observation

4

A 23-year-old male presents after a syncopal episode. EKG findings include normal sinus rhythm, a short PR interval (less than 0.12 seconds), QRS duration of 0.11seconds, and the presence of a "delta wave" (a slurred upstroke to the QRS complex). What condition most likely caused the syncopal episode? Answers: 1.Brugada syndrome 2.Dextrocardia 3.Vasovagal reaction 4.Wolff-Parkinson-White syndrome

4

A 32 year old man is struck several times in the head with a baseball bat. Upon emergency medical service arrival, he is mildly confused, vomits once, and complains of a severe headache. The emergency medical technicians establish two large-bore IVs. Prior to arrival at the emergency department, he loses consciousness and begins to seize. He is actively seizing when he is brought into the trauma bay. What should be the first step in the management of this patient? Answers: 1.Administration of 2 liters NS bolus 2.Administration of phenytoin 1000mg IV 3.Administration of mannitol 50 g IV 4.Rapid sequence intubation using paralytic agent 5.Emergency craniotomy

4

A 40 year old male presents to the emergency department complaining of severe ankle pain after inverting the foot during a soccer game. The triage nurse records the following vital signs: temperature 98.8, pulse 94, respiratory rate 18, BP 188/118. Which of the interventions below is the most appropriate step to take in response to the blood pressure assessment? Answers: 1.Administer a sublingual antihypertensive agent since the patient probably only has an ankle sprain and will not need an intravenous line 2.Establish intravenous access in order to optimize the onset of action of parenteral antihypertensive medications 3.Ignore the blood pressure since the patient is asymptomatic other than having ankle pain 4.Order an antihypertensive agent to be given in the emergency department because the patient will be discharged with a prescription for one

4

A 46 year old construction worker falls 6 feet off a ladder onto a concrete surface and has sudden and severe low back pain. The pain radiates down his right leg and he develops numbness over the anterior shin and dorsum of the foot. On physical exam he has decreased sensation to pinprick over the dorsum of the right foot (medially) and some weakness in right foot dorsiflexion. At which level is a protruding intervertebral disc most likely? Answers: 1.L1-L2 2.L2-L3 3.L3-L4 4.L4-L5 5.L5-S1

4

A 47 year old man with a history of alcohol abuse presents to the emergency department after having a seizure. His past includes both seizures and blackouts. His last alcoholic drink was the previous evening. This morning he experienced palpitations, diaphoresis, and dizziness before losing consciousness and having a seizure lasting under a minute. Which of the following is a true statement with regard to alcohol and its association with seizures? Answers: 1.In people with an underlying seizure disorder, excessive alcohol intake is a risk-factor for seizure due to increased likelihood of head injury, predisposition to metabolic disorders, and lowered seizure threshold. 2.Alcohol intake itself can precipitate seizures due to the neurotoxic effects of alcohol and it metabolites 3.Cessation of alcohol can precipitate seizures as part of the alcohol withdrawal syndrome 4.All of the above statements are true. 5.All of the above statements are false.

4

A 60 year old woman presents to the emergency department with palpitations. Her EKG, shown in the Figure, reveals: Answers: 1.normal sinus rhythm 2.atrial flutter 3.ventricular tachycardia 4.atrial fibrillation

4

A 65 year old male presents to the emergency department with palpitations. His heart rate is 250, blood pressure is 140/88, respiratory rate is 24 and oxygen saturation is 95%. The EKG shown in the Figure demonstrates: Answers: 1.ventricular tachycardia 2.a rhythm requiring verapamil as first line therapy 3.a rhythm that is difficult to identify with certainty 4.a rhythm which requires immediate defibrillation

4

A 68 year old diabetic male, previously living independently, is brought in by his family. He has been acting abnormally for two days. The family reports he is awake all night and sleepy during the day. He is confused about where he is and the time of day, and sometimes doesn't recognize his daughter and son-in-law. At other times he appears and acts almost normally. Answers: 1.Infection is an unlikely cause of his condition unless his temperature is > 102° F. 2.Dementia is the most likely cause of his condition and the family must be counseled about the future course of the disease. 3.Medications are an unlikely cause of this condition in the elderly. 4.Patients can be agitated and combative, or calm and quiet in this condition. 5.Treatment includes maximizing sensory input.

4

A 76 year old restrained driver is involved in a head-on collision at about 35 mph. He arrives at the emergency department in a cervical collar and on a backboard. His only complaint is neck pain, and he has mild posterior neck tenderness. A CT scan of the neck shows no fracture and only degenerative arthritis. Upon re-evaluation you note the patient has difficulty raising his arms against gravity and there is decreased grip strength bilaterally. The remainder of his neurological exam is normal. What is the most appropriate management for this patient? Answers: 1.Immediate neurosurgical decompression 2.Flexion and extention radiographs to rule out ligamentous injury 3.Discharge home with a hard cervical collar with neurosurgical follow-up 4.Administration of IV steroids and ordering of cervical MRI 5.Reassurance and discharge with NSAIDs given the non-anatomical distribution of weakness

4

A patient falls onto his face, and has a CT scan of the face as shown in the Figure. Which indirect finding suggestive of possible facial fracture is present on the CT? Answers: 1.Exopthalmos 2.Extra-sinus air 3.Nasal fracture 4.Fluid (blood) in the sinuses

4

A patient with a ventriculo-peritoneal shunt presents to the E.D. with lethargy and vomiting. A CT scan obtained emergently (see the Right image in the Figure) is compared with a CT scan (see the Left image in the Figure) from a month ago. What is the diagnosis? Answers: 1.encephalitis 2.meningitis 3.pseudotumor cerebri 4.hydrocephalus and shunt malfunction 5.subarachnoid hemorrhage

4

A right-handed patient sustains a circumferential burn (see figure) to the distal right forearm and hand, which is cool despite warm ambient temperature. Regarding the initial assessment and management of the patient, which of the following is correct? Answers: 1.Prophylactic penicillin should be given to any patient with a significant burn 2.Pain medication should be withheld pending obtaining operative consent for emergency burn debridement 3.As shown in the Figure, the burn represents about 10% of total body surface area 4.The involved areas of this 3rd-degree burn would be expected to be mostly insensate

4

A young male patient presented to the emergency department with a history of single gunshot wound from unknown source and caliber. He was hemodynamically stable and had little pain in the right lower quadrant of the abdomen, which was the site of two wounds about 6 inches apart (see Figure). No other wounds were identified during physical examination. Which of the following statements regarding this case is true? Answers: 1.If it is suspected that the two wounds are from the same missile, the emergency department physician's documentation should note which wound is the entrance, and which is the exit 2.The two wounds are more likely than not the result of two separate missiles 3.The superior-medial wound (at the top right of the Figure) is likely the entrance wound, and the inferior-lateral wound is likely the exit wound, of a single missile 4.Wound description is essential for the emergency medicine specialist. However, description of a wound as to entrance or exit is best left to forensic examination.

4

An 82 year old woman with osteoporosis slips and falls onto her right hip. She cannot get up and is brought to the emergency department by ambulance. As you enter the room you notice her right leg is abducted and externally rotated. What type of injury does she most likely have? Answers: 1.Posterior hip dislocation 2.Subtrochanteric femur fracture 3.Intertrochanteric femur fracture 4.Femoral neck fracture 5.Acetabular fracture

4

During a bar fight, a 42 year old man is stabbed in the left side with an unknown weapon. He presents to the emergency department with dyspnea, pulse of 108, blood pressure of 138/92, and oxygen saturation of 94% on room air. He has absent breath sounds on the left side; you note a small puncture wound in the midaxillary line at the level of the 10th rib. His abdominal exam is normal. Two large-bore IVs are established. What is the appropriate management of this patient? Answers: 1.Endotracheal intubation, left-sided chest tube, portable chest x-ray, and admission 2.Left-sided chest tube, portable chest x-ray, and admission 3.Left-sided chest tube, portable chest x-ray, diagnostic peritoneal lavage, and admission 4.Left-sided chest tube, portable chest x-ray, and abdominal CT scan 5.Endotracheal intubation, portable chest x-ray, exploratory laparotomy in the OR, and admission

4

Following a motor vehicle crash, a 25 year old man presents complaining of a painful right eye. Visual acuity is 20/200 in the right eye and 20/25 in the left eye. The right eye protrudes from the orbit and the patient has right eye pain with extraocular movement. What is the most likely cause of his symptoms? Answers: 1.chemosis 2.hyphema 3.orbital blow-out fracture 4.retrobulbar hematoma 5.ruptured globe

4

In a patient with malignant hypertension, the patient's blood pressure should be reduced to what value in the initial 2 hours of treatment? Answers: 1.120/80 mm Hg 2.100/70 mm Hg 3.90/60 mm Hg 4.75% of the pretreatment mean arterial pressure (MAP) 5.50% of the pretreatment MAP

4

In differentiating high voltage electrical injury from lightning injury, which of the following is your best discriminator? Answers: 1.Loss of consciousness 2.Cardiac arrest 3.Fractures or dislocations 4.Deep burns

4

Of the following choices, which diagnosis is most likely in a 35-year old female with intermittent palpitations and the EKG shown in the Figure? Answers: 1.digoxin overdose 2.asthma 3.pericarditis 4.Wolff-Parkinson-White syndrome

4

The best treatment, of the options below, for a patient with second degree AV block Mobitz Type II is: Answers: 1.amiodarone 2.epinephrine 3.aspirin 4.transvenous pacing 5.lidocaine

4

Which of the following statements regarding fever and WBC is TRUE? Answers: 1.Elevated WBC indicates serious bacterial infection. 2.WBC is sensitive for serious bacterial infection. 3.WBC is specific for serious bacterial infection. 4.WBC is a poor discriminatory predictor of serious bacterial infection. 5.WBC is of no clinical value.

4

A 42 year-old woman presents with an overdose of her Xanax (alprazolam) that her family indicates she has been taking for years to help with her anxiety. The bottle indicates that the prescription was filled yesterday with 90 pills and is now empty. The patient is minimally responsive to painful stimuli and does not react when you suction secretions out of her posterior pharynx. What is your next management step? 1.Administration of flumazenil 2.Administration of narcan 3.Close observation 4.Intubation for airway support

4. Intubation for airway support

A 55 year-old man is brought down from the outpatient procedures clinic after becoming severely short of breath during an endoscopy under light sedation. His pulse oximeter is reading 100% on a non-rebreather mask. You notice an interesting discoloration of his blood when it is drawn. What antidote should be administered? 1.Amyl nitrite 2.Deferoxamine 3.Hydroxycobalamin 4.Methylene blue

4. Methylene blue; Nitrates/nitrites, local anesthetics, dapsone, and phenazopyridine are the common causes of methemoglobinemia. Methemoglobinemia causes the oxygen dissociation curve to shift to the left, making the remaining hemoglobin less likely to give up oxygen to the tissues. Blood from patients with methemoglobinemia is a chocolate brown color. Methylene blue is the antidote. Pulse oximetry is unreliable in patients with methemoglobinemia, since the pulse oximeter cannot differentiate oxyhemoglobin from methemoglobinemia.

The clinical presentation of clonidine toxicity most closely mimics toxicity from which of the following classes of medication? 1.Beta blockers 2.Cholinergics 3.Stimulants 4. Opioids

4. Opioids; the hallmark signs and symptoms of clonidine toxicity include:hypotension, bradycardia, mental status change, respiratory depression, and miosis. The presentation very closely mimics opioid toxicity.

For which of the following cases (all of which are characterized by an ingestion history known with certainty) is gastric lavage most likely indicated? 1. Adult patient, ibuprofen ingestion (800 mg/tab x 5 tabs, 20 minutes prior to arrival (PTA) in the ED 2. Adult patient, kerosene ingestion (8 oz, 20 minutes PTA) 3.Adult patient, metoprolol ingestion (100 mg/tab x 100 tablets, 6 hours PTA) 4.Adult patient, nortriptyline ingestion (50 mg/tab x 100 tablets, 45 minutes PTA)

4.Adult patient, nortriptyline ingestion (50 mg/tab x 100 tablets, 45 minutes PTA)

A mother brings her 3 year old daughter into the emergency department for an arm injury. The mother was holding her hand to cross a busy street. She pulled hard on her daughter's arm to hurry across the street and the child began to cry. Since the incident the child has kept her arm against her body in a slightly flexed and pronated position. She is tender over the radial head and refuses to move her elbow, but there is no swelling or ecchymosis. What is the most appropriate management for this girl? Answers: 1.Obtain immediate X-rays of the elbow to rule out fracture 2.Obtain immediate orthopaedic consult for presumed elbow dislocation 3.Apply posterior elbow splint and follow-up with orthopedics within one day 4.Apply pressure to the radial head while flexing and supinating elbow 5.Apply traction to elbow and splint under conscious sedation

4.Apply pressure to the radial head while flexing and supinating elbow

All of these steps are involved in pediatric rapid sequence intubation (RSI) EXCEPT: Answers: 1.If administering succinylcholine, pretreatment with atropine is always indicated in children under age 10. 2.Lidocaine pretreatment is usually provided to children with head trauma. 3.When deciding which size endotracheal tube to use, one may approximate by using the size of the small finger or nares as a reference. 4.Due to the fragility of the pediatric C-spine, a cervical collar should always be placed prior to intubation for children under age ten. 5.Uncuffed endotracheal tubes are the preferred devices used in children under 6-8 years of age.

4.Due to the fragility of the pediatric C-spine, a cervical collar should always be placed prior to intubation for children under age ten.

Management of a 4 year old child with a two-week history of malodorous purulent nasal drainage should include, as an initial step: Answers: 1.Oral amoxicillin for 10-14 days 2.Referral to an otolaryngologist 3.Social services consultation to investigate potential child abuse 4.Speculum examination of the nares

4.Speculum examination of the nares

A 27 year old G2P1 female presents to the emergency department in labor at 41 weeks estimated gestational age. The amniotic sac breaks on admission and has thick, brown-tinted fluid. Prior to other steps in resuscitation, the newborn infant should: Answers: 1.be resuscitated with a bag-valve mask for 45 to 60 seconds. 2.receive 500,000u penicillin-G intramuscularly. 3.receive 8 mg doxycyline intravenously. 4.have his/her trachea suctioned. 5.be left unswaddled.

4.have his/her trachea suctioned.

A 22 year old man is punched in the nose during a fight. He presents to the emergency department with obvious nasal bone deformity. Pressure controls the bleeding. Physical exam reveals no maxillary bone or orbital rim tenderness, intact vision and extraocular movement. The oropharynx and mandible are unremarkable. Nasal inspection reveals a swollen, ecchymotic, tender nasal septum. Which of the following is the most appropriate initial step? Answers: 1.Plastic surgery consult for immediate reduction of nasal fracture 2.Facial CT scan to rule out more serious facial fractures 3.Outpatient follow-up with an ENT specialist to surgically correct a deviated septum 4.Needle aspiration of the septal hematoma 5.Incision and drainage of the septal hematoma followed by nasal packing

5

A 40 year old female presents to the emergency department complaining of a few days of headaches, excessive sweating, anorexia, heat intolerance and palpitations. She has also been having upper respiratory symptoms over the past week. She is found to have a blood pressure of 170/106 and an EKG, urinalysis, fundoscopic examination, serum creatinine, and neurological evaluation are negative. What is the next step in the evaluation/management? Answers: 1.Administer sublingual nifedipine while the work-up continues 2.Perform CT scan of the abdomen 3.Avoid sublingual or intravenous therapy in the ED and prescribe an oral beta-blocker 4.Schedule a clonidine suppression test to evaluate for pheochromocytoma 5.Obtain a medication history

5

All of the following are true regarding adult epiglottitis EXCEPT: Answers: 1.Type b H. influenza is the most common bacterial pathogen to cause acute epiglottitis. 2.Adult epiglottitis is a cellulitis of the supraglottic structures of the airway including the epiglottis, vallecula, and the base of the tongue. 3.The incidence of adult epiglottitis has increased in recent years, whereas the incidence of pediatric epiglottitis has decreased. 4.Smokers have a higher incidence of adult epiglottitis. 5.The rapidity of symptoms' onset does not correlate with the likelihood of need for airway intervention.

5

The most sensitive bedside test for nerve injury in a finger after trauma is: Answers: 1.light touch 2.O'Riain wrinkle test 3.pain 4.temperature sensation 5.two-point discrimination

5

Which is not part of the Ottawa ankle rules? Answers: 1.inability to walk 4 steps at the time of the injury 2.inability to walk 4 steps in the emergency department 3.tenderness over the lateral malleolus 4.tenderness over the medial malleolus 5.tenderness over the talus

5

Which of the following is not a feature of febrile seizures? Answers: 1.generalized tonic-clonic seizure 2.duration less than 15 minutes 3.associated with a rapid rise in body temperature 4.occurs in children ages 3 months to 5 years 5.associated with postictal state of 30 minutes

5

Which of the following patients should undergo abdominal trauma evaluation? Answers: 1.22 year old with stab wound to fourth intercostal space on right 2.30 year old with pelvic pain and tenderness after fall 3.25 year old restrained passenger in high-speed MVA; no abdominal complaints 4.None of the above 5.All of the above

5

With regard to targets for therapy of elevated blood pressure identified during an emergency department visit, which of the following is generally true? Answers: 1.Patients with hypertensive emergencies should have blood pressure normalized (for age) within an hour or less 2.Patients with hypertensive emergencies should have mean arterial blood pressure lowered by 50% within 50 minutes 3.Patients with hypertensive urgencies are preferably treated with sublingual nifedipine, as compared with intravenous agents 4.Patients with hypertensive urgencies should have blood pressure normalized (for age) within an hour 5.The target systolic pressure for patients with acute aortic dissection is an absolute number rather than a percent pressure reduction

5

Regarding pediatric head injury, all the following are true EXCEPT: Answers: 1.Head trauma is the leading cause of death among injured children. 2.A child's cranial vault is larger and heavier in proportion to its total body mass than an adult's. 3.Pediatric epidural hematomas are venous in origin. 4.A brief seizure occurring immediately after the insult, with rapid return to normal level of consciousness is usually unassociated with intracranial parenchymal injury. 5.Retinal hemorrhages are a common finding in mild-moderate trauma.

5.Retinal hemorrhages are a common finding in mild-moderate trauma.

Which of the following factors in the fetus is NOT associated with an increased risk for neonatal resuscitation? Answers: 1.prematurity 2.intrauterine growth failure 3.multiple gestation 4.thick meconium in amniotic fluid 5.alkalosis (as assessed via fetal scalp capillary monitoring)

5.alkalosis (as assessed via fetal scalp capillary monitoring)

Which of the following is the commonest type of pediatric rhythm in the setting of cardiopulmonary arrest? Answers: 1.supraventricular tachycardia 2.ventricular tachycardia 3.atrial flutter 4.atrial fibrillation 5.asystole

5.asystole

Which of the following is NOT a frequent cause of airway obstruction in the neonate? Answers: 1.mucus 2.blood 3.meconium 4.tongue 5.maternal drugs

5.maternal drugs

A newborn presents to the emergency department a few days after hospital discharge. The infant has been noted to be drooling, choking, and coughing, and the mother reports seeing bubbles at the baby's mouth. Which is the most likely explanation? Answers: 1.osteogenesis imperfecta 2.inborn error of metabolism 3.congenital diaphragmatic hernia 4.foreign body 5.tracheoesophageal fistula

5.tracheoesophageal fistula

A 21-year-old woman presents to the emergency department with fevers, headache, neck stiffness, and mild confusion over the past several days. Her temperature is 38.0 C (100.4 F), pulse 106, and blood pressure 116/74. On physical exam she looks ill, and her neck is stiff. Her neurologic exam is normal. A lumbar puncture reveals 105 WBC and 1240 RBC in tube #1 and 126 WBC and 1360 RBC in tube #4; all white cells are lymphocytes. The CSF protein is 68 and the glucose is 78. This patient most likely has which of the following? A. HSV encephalitis B. Pneumococcal meningitis C. Subarachnoid hemorrhage D. Subdural hematoma

A

A 27 year old man is shot in the right leg. He is unconscious. The wound appears to be pulsatile. The medics report he has lost a lot of blood. His heart rate is 160, and his BP is 70/30. He has received 2 liters of IVF normal saline. The next step in management would be: A. Administer Type O Rh+ blood B. Check a hemoglobin level and hematocrit C. Wait for type-specific blood D. Wait for cross-matched blood E. Give more saline

A

A 3-week old presents to the emergency department with a fever of 100.8. The child is otherwise well appearing. After blood culture, the best management of the options below is: A. UA plus urine culture, LP, antibiotics, and admit B. Check WBC before further management. C. Admit for observation. D. Discharge home E. ampicillin and discharge

A

A college student who had a mild upper respiratory tract infection last week, presents during the spring. He appears toxic, with fever, headache, and a rash (see figure) which was also noted on the wrists, ankles, flanks, and axilla. Of those listed below, which is the most likely diagnosis? [image] A. meningococcemia B. Lyme disease C. pemphigus vulgaris D. angioedema E. herpes zoster

A

An early sign and symptom of shock is: A. Tachycardia B. Bradycarda C. Decreased respiratory rate D. Cyanosis E. Hypotension

A

Dermatological anthrax may occur wherever spores come into contact with the skin. Of the following options, which statement is FALSE concerning dermatological anthrax: A. Initial diagnosis is usually made by Gram's stain analysis. B. The organism most likely to cause dermatological anthrax is Bacillus anthracis. C. Mortality rates are over 20-30% lower than for pulmonary anthrax. D. Antibiotics do not affect the course of local disease. E. Characteristic lesion is a black escar preceded by a vesiculopapular lesion 1 week prior.

A

The four classic types of shock include all of the following EXCEPT: A. Traumatic B. Hypovolemic C. Obstructive D. Cardiogenic E. Distributive

A

The pathophysiology of cardiogenic shock is: A. Cardiac pump failure B. Vasodilation C. Endotoxins D. Hypoxia E. Hypovolemia

A

A 44 year old woman presents complaining of the acute onset of left eye pain while walking into a movie theater one hour ago. On physical examination, she has stable vital signs. Her visual acuity is 20/25 on the right eye and 20/200 on the left eye. Her pupillary exam is notable for a minimally reactive dilated left pupil and a steamy cornea. The left eye is red and obviously painful. Which of the following conditions is most likely? Answers: 1.acute angle closure glaucoma 2.central retinal artery occlusion 3.hypopyon 4.optic neuritis 5.vitreous hemorrhage

AACG

For a patient who has suffered a severe anaphylactic reaction, which of the following medications would serve little purpose if prescribed upon discharge? Answers: 1.prednisone 60mg daily for 5 days 2.benadryl 25-50mg every 4-6 hours 3.Epi-Pen 4.albuterol inhaler 2 puffs every 4 hours

Albuterol

A 4 year old boy presents with fever, sore throat and stridor. Physical exam reveals T103 in an ill-appearing, drooling, stridorous child in mild respiratory distress. The next steps include: Answers: 1.albuterol 2.antibiotics and airway management 3.chest X-ray 4.dexamethasone 5.observation

Antibiotics and airway management; A child presenting to the emergency department with stridor most likely has croup, with epiglottitis being a less common - but more serious - etiology. Croup tends to be the etiology in younger, nontoxic-appearing children, who usually have a characteristic barking ("seal") cough. Treatment of croup includes cool mist (though the literature supporting this is limited), racemic epinephrine (which is probably no better than nebulized l-epinephrine), and steroids. Epiglottitis is a true emergency. It presents in children who are older, with high fever, and who are ill appearing. One of the key clinical features is drooling, which indicates swelling and pain interfering with handling of secretions.

A 19 year old man is stabbed in the left chest in the 3rd intercostals space just to the left of the sternum. His vital signs are: T 97.9, HR 130, BP 60/48, RR 18, SPO2 84%. He has significant JVD and his lungs are clear. The patient's diagnosis is: A. Tension pneumothorax B. Cardiac tamponade C. Lung laceration D. Spinal cord injury E. Hypovolemic shock

B

All of the following are signs and symptoms of hypovolemic shock EXCEPT: A. Tachycardia B. Warm, moist skin C. Cool, clammy skin D. Narrow pulse pressure E. Decreased capillary refill

B

As compared to adults, children with shock usually: A. Have more reliable signs and symptoms B. Are able to maintain their blood pressure better C. Have similar epidemiology (i.e. causes for shock states) D. Do not need specialized care E. Have different treatment priorities

B

A 22 year old male with IVDA presents to the emergency department with a fever and dyspnea. His physical exam reveals T 101.5, clear lungs and a murmur. Of the following, the correct treatment plan is? Answers: 1.blood cultures and discharge 2.amoxicillin and discharge 3.blood cultures and admission 4.blood cultures, vancomycin, and admission 5.surgery

BC, Vanc, Admit; infective endocarditis, staph aureus

What is treatment of conjunctivitis?

Bacterial- unilateral thick purulent discharge tx with topical abx (erythro, tmp-smx topical) Viral- bilateral clear watery discharge (adenovirus)

All patients with shock should receive as the first priority: A. Packed red blood cells B. Trendelenburg positioning C. Supplemental oxygen D. Intravenous fluids E. Antibiotics

C

During hypovolemic shock, hypotension tends to develop after the loss of what percent of blood volume? A. 10% B. 20% C. 30% D. 40% E. 50%

C

Which of the following is not a known complication of subarachnoid hemorrhage in the immediate several weeks following the initial bleed? A. rebleeding B. hydrocephalus C. hypernatremia D. seizure E. cerebral artery vasospasm

C

While temperatures vary with time of day and method assessed, the generally accepted upper limit of normal temperature is: A. 98.6 F (37 C) B. 99.5 F (37.5 C) C. 100.4 F (38 C) D. 101.3 F (38.5 C) E. 102.2 F (39 C)

C

All of the following are common causes of septic shock EXCEPT: A. Streptococcus pneumoniae B. Escherichia coli C. Staphylococcus aureus D. Group A beta-hemolytic Streptococcus E. Pseudomonas aeruginosa

D

The best IV access for volume resuscitation of the hypovolemic patient is: A. intraosseous line B. PICC line C. triple-lumen internal jugular central venous catheter D. 16g catheter in the antecubital fossa E. 22g catheter in the dorsum of the hand

D

Which of the following is a common physiologic finding in septic shock? A. Normothermia B. Increased cardiac index C. Increased pulmonary wedge pressure D. Decreased urine output E. Increased systemic vascular resistance (SVR)

D

Women who develop pre-eclempsia in their first pregnancy have a long-term risk for which of the following? Answers: 1.diabetes mellitus 2.hypertension 3.liver disease 4.habitual miscarriage 5.renal failure

DM

All of the following are common complications of septic shock EXCEPT: A. Adult respiratory distress syndrome (ARDS) B. Acute tubular necrosis (ATN) C. Disseminated intravascular coagulation (DIC) D. High-output congestive heart failure (CHF) E. Pulmonary embolus (PE)

E

Shock is defined as: A. hypovolemia B. tachycardia C. hypotension D. altered mental status E. inadequate tissue and organ perfusion

E

Which of the following are potential complications of bacterial meningitis? A. Seizure disorder B. Focal paralysis or sensory loss C. Intellectual impairment D. Sensorineural hearing loss E. All of the above

E

Which of the following is FALSE regarding the common skin disorder, seborrheic dermatitis? A. differential diagnosis includes Tinea capitis, psoriasis of scalp, cutaneous lupus erythematosus B. uncommon between infancy and puberty C. found in skin folds and hair-bearing of face scalp, chest and groin D. characterized by erythema and waxy scaling E. initial therapy often consists of high dose topical steroids

E; Initial therapy for seborrheic dermatitis is application of anti-dandruff shampoo lathered onto the area and left on for 5-10 minutes. Shampoos can contain zinc pyrethrin (Head and Shoulders), selenium sulfide (Selsun Blue), salicylic acid (Neutrogena T-Sal) or tar (Polytar or Neutrogena T-Gel).e

With growing regionalization of care for many patient types and conditions, the traditional province of "prehospital" care is growing to include "out-of-hospital" care. The increasing need for critical care transport to regional centers has translated into regularly-occurring out-of-hospital, intratransport utilization of all the following EXCEPT: Answers: 1.extracorporeal membrane oxygenation (ECMO) 2.intra-aortic balloon counterpulsation 3.mechanical ventilation 4.ventilation with nitric oxide-containing gas 5.continuous propofol infusion

ECMO

Esophageal foreign bodies: Answers: 1.Occur at the upper Esophageal Sphincter 2.Require surgical treatment 3.Commonly perforate 4.Can be treated with endoscopy 5.Cannot be treated with medications

Endoscopy

Which of the following statements regarding the topical ophthalmic anesthetic proparacaine is TRUE? Answers: 1.It is an amide. 2.It is an ester. 3.It is cardiotoxic. 4.It is often prescribed for home-going analgesia in patients with corneal pathology. 5.It stains the tears orange.

Ester (2), check for allergies as some patients have ester or amide allergies, only used in ED as can decrease corneal healing

A collection of purulent material in the location as depicted in the figure represents a: (underneath distal phalynx) Answers: 1.paronychia 2.herpetic whitlow 3.felon 4.furuncle 5.cellulitis

Felon; felon is infection underneath distal phalynx, if tense suspect abscess and do I&D; paronychia is infection/swelling of nail bed, herpetic whitlow will have blisters

A patient presents complaining of severe pain and swelling in the distal aspect of his index finger. He has no history of trauma. On exam he has tense swelling and redness on the pad of the digit. Of the following, which is the best next step? Answers: 1.antibiotics and discharge 2.consult orthopaedics for pinning of a probable fracture 3.incision and drainage 4.radiograph 5.treat for herpetic whitlow and discharge

I&D

Which of the following analgesics operates by a non-opioid mechanism? Answers: 1.hydromorphone 2.meperidine 3.fentanyl 4.ketorolac 5.codeine

Ketorolac

A 20 year old otherwise healthy obese woman presents complaining of headache, nausea, vomiting. She is afebrile and her vital signs are normal. Physical examination reveals papilledema, but an otherwise normal neurological exam without meningismus. Non-contrast CT head scan is negative for intracranial pathology. What is the most appropriate next step? Answers: 1.IV antibiotics 2.EEG 3.MRI/MRA 4.erythrocyte sedimentation rate/C-reactive protein 5.lumbar puncture

LP; psudomotor cerebri (increased ICP, papilledema, NL CT and CSF)

Since hiking in Connecticut 2 weeks ago, a 27-year-old woman has developed a rash and low-grade temperature. The anular-macular rash appears to be spreading from one location. She remembers multiple insect bites. The correct diagnosis is: Answers: 1.Rocky Mountain spotted fever 2.Lyme disease 3.scarlet fever 4.varicella 5.tinea

Lyme disease-erythema migrans, Doxy or Amox

A 32 year old G1P0 at 33 weeks EGA comes into the emergency department complaining of a severe headache. She has contractions every 3 minutes. She is experiencing flashes of light in front of her eyes. Her pregnancy has been uncomplicated until this time, and her only medical problem is mild asthma. Her vital signs are: T 36.5 C (97.7 F), BP 172/114, P 78, R 14, and a room air SpO2 99%. Her lungs have bilateral crackles at the bases, and her cervix is dilated at 3 cm and effaced at 50%. Her urinalysis has 2+ protein, and her complete blood count shows: WBC 8,000/mm3, hematocrit 38%, platelets 215,000/mm3. Her BUN and creatinine are normal, her AST is 250 U/L, and her ALT is 316 U/L. The electronic fetal monitor shows a reactive and variable heart tracing at a rate in the 150s. What is the appropriate next step in management for her? Answers: 1.Start PGE2 gel 2.Discharge the patient and tell her to return for a cervical check the next day 3.Give furosemide 4.Start terbutaline, as a tocolytic that could prophylax against an asthma flare as well 5.Administer magnesium sulfate

Magnesium

All of the following may be used in the treatment of acute angle closure glaucoma (AACG) EXCEPT? Answers: 1.carbonic anhydrase inhibitors 2.laser iridectomy 3.mannitol 4.mydriatic agents 5.topical beta blockers

Mydriatic agents. In fact, mydriatic agents are contraindicated. Miotic drops, like Pilocarpine by contrast, are used. Other agents employed in the treatment of AACG include: carbonic anhydrase inhibitors (decreased aqueous production), topical beta cblockers (decreased aqueous production), mannitol (decreased intraocular pressure), and iridectomy

A 16 year old male presents with a large swollen right knee. He denies any trauma. His physical exam reveals T101 and a swollen right knee. The knee is hot, and there is pain with motion. Also noted is a diffuse rash. The correct diagnosis is: Answers: 1.gout 2.pseudogout 3.juvenile rheumatoid arthritis 4.Neisseria gonorrhea 5.Osgood-Schlatter

N. Gonorrhea

A mother brings her 35 year old son to the emergency department because of tremor and mutism for the past three days. His mother found him in his room this morning lying stiffly in his bed, soiled with urine and feces. He appears confused and will not respond to questions. He was diagnosed with schizophrenia last year and has been on several medications. Last month after his most recent hospital admission for schizophrenia, he was discharged with a prescription for haloperidol. On physical exam, he is visibly diaphoretic and has vital signs as follows: T 102.7, BP 140/98, P 112, R 12. His neuromuscular exam shows extremely rigid extremities, and his laboratory values are notable for a white blood cell count of 15000/mm3 and abnormally elevated creatine phosphokinase levels. What is the most likely explanation for these findings? Answers: 1.neuroleptic-induced acute dystonia 2.neuroleptic malignant syndrome 3.schizophrenia, catatonic type 4.tardive dyskinesia

NMS

A 19 year old presents with bizarre behavior and a friend admits to use of PCP. What ocular findings would you expect? 1.Afferent pupillary defect 2.Monocular diplopia 3.Nystagmus 4.Mydriasis

Nystagmus

Which of the following women are considered at increased risk for pre-eclampsia? Answers: 1.cigarette smoker 2.multiparous 3.obese 4.over 20 years old 5.woman with a single intrauterine pregnancy

Obese

Regarding optic neuritis, all of the following are true EXCEPT: Answers: 1.associated with afferent pupillary defect 2.commonly affects color vision 3.may be the initial manifestation of multiple sclerosis 4.oral steroids are indicated for treatment 5.preferentially affects women

Oral steroids, patient should receive IV steroids not oral

What is the most common cause of fatal anaphylaxis? Answers: 1.foods, including shellfish, nuts, and eggs 2.insect stings 3.penicillin 4.radiographic contrast dye

PCN

Which of the following drugs is NOT associated with potential toxic side effects related to the inner ear? Answers: 1.gentamicin 2.aspirin 3.furosemide 4.penicillin 5.phenytoin

PCN

A 10 year old boy presents with high fever and a rash. The rash started on his wrists and ankles and has spread to his trunk, palms, and soles. The correct diagnosis is most likely: Answers: 1.Rocky Mountain spotted fever 2.Lyme disease 3.scarlet fever 4.varicella 5.tinea

RMSF; rash starts wrists and ankles and spreads to palms, soles, trunk, prodrome of headache, malaise, fever; Doxycycline, even in children...chloramphenical for pregnant

You have a 3 year old female present with her mother with compaints of two days of left ear pain. On exam, you are unable to visualize the tympanic membrane due to an obstructing mass. You suspect a foreign body. Which of following is TRUE regarding the removing a foreign object from the patient's ear canal? Answers: 1.A. Avoid suction as it may lead to a perforation of the tympanic membrane 2.B. Referral to an otolaryngologist for foreign body removal under general anesthesia may be required in an uncooperative infant 3.C. To remove a live insect from the external ear canal, grasp a leg with hemostats and pull firmly 4.D. You should avoid the use of lidocaine and other topical anesthetics due to the risk of localized tissue ischemia

Referral to ENT for removal

A 22 year old college student arrives in the emergency department complaing of painful, tearing, and redness in the right eye. Blinking increases the pain. She wears contact lenses for distance vision but has no other ocular history. During your examination, you evert her upper eyelid. ED management of this patient includes: Answers: 1.A. Avoidance of Fluorescein examination due to concern for globe rupture 2.B. Immediate application of pilocarpine drops 3.C. Instructions to wear an eye patch for the next 48 hours 4.D. Removal of the foreign body with a moistened cotton swab

Removal of foreign body with cotton swab

Which of the following is the most serious toxic effect of the use of MgSO4 in treatment of eclampsia? Answers: 1.Loss of deep tendon reflexes 2.nausea and vomiting 3.neonatal hypotonia 4.renal failure 5.respiratory depression

Respiratory depression

A 2 and a half year old girl is brought in to the emergency department by her mother for "loud breathing" and a fever. The mother states the child has been previously healthy and is up to date on all her vaccines. Initial evaluation of the child reveals an ill-appearing child with her head propped upright, who is drooling and making stridorous noises. Her vital signs are notable for a temperature of 104.2 F. Which of the following is most likely in this child? Answers: 1.Chlamydia pneumonia 2.viral croup 3.Stapholococcus aureus retropharyngeal abscess 4.choanal atresia 5.viral upper respiratory tract infection

Staph aureus retropharyngeal abscess; This patient exhibits several classic symptoms of retropharyngeal abscess including fever, neck stiffness, and drooling, as well as her generally toxic appearance. Retropharyngeal abscess in children is often associated with foreign body ingestion leading to perforation of the hypopharynx or esophagus

Which pharmacologic treatment for hyperkalemia works through stabilization of cardiac membranes? A. Calcium B. Insulin and glucose C. Magnesium D. Bicarbonate

The answer is A. "Immediate antagonism of K+ at the cardiac membrane is achieved with IV administration of calcium chloride or gluconate. This is indicated in patients with unstable dysrhythmia or hypotension."

A 71-year-old male presents after a syncopal episode. He reports 12 hours of recurrent substernal chest pressure. A report from the patient's primary care physician's office states that an EKG performed four days ago was completely normal. Repeat EKG in the ED reveals no ST-segment elevation, but you do note a right bundle-branch block, and a left anterior fascicle block. Troponin I is elevated above normal at 1.6. What intervention would be indicated to provide definitive management for the findings seen on EKG in this patient? A. Urgent placement of a cardiac pacemaker B. Radiofrequency ablation C. Emergent revascularization with thrombolytics or percutaneous coronary intervention (PCI) D. Continuous cardiac monitoring for 24-48 hours

The answer is A. "In the face of an AMI, the risks of complete heart block are much greater when new or preexisting bi- or trifascicular conduction blocks are present. In this setting, prophylactic placement of a ventricular demand pacemaker is indicated."

"Silent Suicide" is defined as: A. an act of slowly killing oneself by nonviolent means, such as starvation or non-compliance with essential medical treatment B. a self-destructive act disguised as an accident C. recurrent self-destructive acts, such as heavy drinking in the presence of alcoholic liver disease D. suicide involving a number of willing and sometimes not so willing participants E. an attempted suicide

The answer is A. "Silent Suicide" is most common in elderly patients and frequently goes unrecognized. Such patients may present to the emergency department repeatedly because of non-compliance with treatment of their medical disorders. "Occult Suicide" is applied to self-destructive acts disguised as accidents and should be suspected in those who have "accidental" self-inflicted gun shot wounds, and in those who "unintentionally" overdose, or who fall from a height. "Chronic Suicidal Behavior" consists of recurrent self-destructive acts. "Mass or Group Suicide" is suicide involving a number of people. "Parasuicide" is an attempted suicide, which is seen more as a gesture than a serious act.

All of the following may be indications for thoracentesis in the emergency department EXCEPT: A. biopsy of a lung mass (anterior or posterior approach) B. diagnostic evaluation of a pleural effusion (posterior approach) C. evacuation of a simple stable pneumothorax (anterior approach) D. acute treatment of a large symptomatic pleural effusion (posterior approach) E. diagnosis and treatment of suspected tension pneumothorax (anterior approach)

The answer is A. All the other answers are common indications for performing a thoracentesis. Lung biopsy is not performed in the emergency department.

A 24 year old female without prior medical history presents with a one day history of left sided facial weakness. It was preceded by a headache behind her left ear. On exam she is unable to wrinkle her left forehead or close her left eye. The corner of her mouth droops on the left. The rest of the exam is normal. Which of the following would be inappropriate in the care of this patient? A. CT of the brain with and without intravenous contrast. B. Acyclovir. C. Evaluation of Lyme disease if the patient lives in or has visited a Lyme endemic area. D. A short course of prednisone. E. Protecting her left eye with moisturizing drops and a patch at bedtime.

The answer is A. Bell's palsy is an idiopathic palsy of the facial nerve. Although it is the most common cause of a facial nerve palsy, other etiologies must be ruled out. Inability to move the forehead muscle indicates a peripheral lesion, making stroke much less likely. If the remainder of the neurological exam is normal, then imaging is not needed. Lyme disease is a well known cause of facial nerve palsy and patients should be evaluated for this if they live in an endemic region. Half of cases present with retroauricular pain around the time of onset. Most neurologists recommend a short course of prednisone as part of treatment. There is evidence that herpes simplex virus is involved as a causative agent, and acyclovir is recommended. Also extremely important is to protect the involved eye, as it is at risk of drying out because lacrimal gland functioning can be impaired, and the eye is unable to close fully.

Regarding the pathogens involved in community-acquired pneumonia, which of the following is true? A. Co-infection with multiple bacteria, such as Chlamydia and Strep pneumoniae commonly occur B. Etiologic agents for patients admitted to the ICU with pneumonia most commonly include Neisseria meningitidis and Strep pneumoniae C. Milder cases of community acquired pneumonia are frequently caused by Chlamydia D. Q fever, caused by Coxiella burnetii, may present as pneumonia, particularly in patients exposed to rabbits

The answer is A. Co-infection with multiple bacteria, such as Chlamydia and S. pneumoniae, is a well-recognized occurrence and should be sought out to ensure appropriate antibiotic coverage.

Epidural hematomas are least likely in which age group? A. Children less than 2 years B. Prevalence is the same throughout age groups C. Adults excluding elderly D. Children between 8 and 14 E. Elderly

The answer is A. Epidural hematoma (EDH) is less likely in children and elderly because of the close attachment of the dura to the periostium of the skull. This is especially true of children less than 2 years because of the added elasticity of the skull.

Regarding the eyebrow laceration, in a 30-year old female, as depicted in the Figure, which of the following is true? [image] A. If there is not much tension on the wound, a topical adhesive (such as Dermabond) may be used to approximate the wound edges B. The eyebrow hair should be shaved to optimize the ability to closely approximate the wound edges C. This laceration is not suitable for topical anesthesia as an adjunct to repair D. An infraorbital nerve block would provide adequate anesthesia to the area of the laceration E. If lidocaine anesthesia is to be used, the solution should not contain epinephrine

The answer is A. Eyebrow hair tends to not regrow after being shaved; a petroleum jelly can be used to "retract" the hair if necessary. If anesthesia is to be used to close this wound (wound glue would be a reasonable alternative), topical anesthesia is a good approach as long as care is taken to prevent drip into the eye. An infraorbital nerve block would not provide anesthesia to the area of the laceration. Epinephrine can be used safely in the supraorbital region.

You are performing procedural sedation to repair a complex facial laceration on a child in the ED. Unfortunately your patient is progressing from clinical signs of moderate sedation to deep sedation and is in need of reversal using flumazenil. In which of the following scenarios is the use of flumazenil appropriate? A. Patient is younger than 3 years old and had a glass of milk four hours ago B. Patient has been given IV ketamine as a single agent for sedation C. Patient exhibits signs of seizure activity during procedural sedation D. Patient has been given IV fentanyl as a single agent for sedation

The answer is A. Flumazenil should not be used on a patient exhibiting seizure activity (A). Also co-ingestion of drugs with pro-convulsant properties (cyclic antidepressants) is associated with an increased risk of seizures, presumably due to loss of the benzodiazepine's protective anticonvulsant effect when the antagonist is administered. For similar reasons (related to GABA effects) flumazenil can theoretically precipitate seizure activity in patients who chronically take benzodiazepines or chloral hydrate. The co-administration of flumazenil also risks cardiac side effects for these patients. Flumazenil would not be indicated for reversal of sedation when using only fentanyl or ketamine (B, D). Naloxone (or another opioid antagonist), not flumazenil, is the appropriate reversal agent for fentanyl. There is no reversal agent available for ketamine. Flumazenil can be used for the acute reversal of benzodiazapine overdose in procedural sedation for children younger than 3. The patient's consumption of milk four hours earlier met the ASA fasting guidelines and is irrelevant.

With regard to laboratory findings in hypothyroidism, which of the following is false? A. Free thyroxine (T4) is always depressed in hypothyroid states. B. T3 level may be normal in hypothyroid states. C. Total thyroxine levels may be normal due to elevated thyroxine-binding globulin (TBG) levels. D. Free T4 and TSH levels are typically low in secondary and tertiary hypothyroidism. E. Serum thyroid-stimulating hormone (TSH) is the most sensitive test to diagnose primary hypothyroidism.

The answer is A. Free T4 may be normal in early stages of hypothyroidism due to physiologic compensation from elevated TSH levels.

Laboratory abnormalities typically seen with adrenal insufficiency include all of the following EXCEPT: A. hypokalemia B. hyponatremia C. hypercalcemia D. azotemia E. hypoglycemia

The answer is A. Hyperkalemia is seen in approximately 64% of patients with adrenal failure. Typically this is because of aldosterone production failure that normally enhances potassium excretion. Even more common is hyponatremia, present in 88% of patients. Hypoglycemia is present in two-thirds of patients and is a significant cause of morbidity and mortality associated with adrenal failure. Hypercalcemia is seen in 6 to 33% for unclear reasons; azotemia and increased hematocrit from hypovolemia may also be present.

A 36 year old male backpacking in the wilderness loses his way in a snowstorm. Temperatures are well below zero degrees and his clothing is inadequate. He is rescued 5 days later and presents to the ED. Rescue crew has already initiated passive rewarming and have removed patient's damp clothing. On arrival, vital signs show pulse of 100 and temperature of 35.5C. On physical exam, you note patient has several toes that are purple with hemorrhagic blisters on his feet. Which of the following is the most appropriate initial management? A. Immersion in warm water bath B. Debridement of necrotic tissue C. Tetanus prophylaxis D. Administration of morphine

The answer is A. Immersion of the affected extremity is the mainstay of treatment for patients with frostbite. Numbness of the affected area is the most common initial symptom and severe pain is frequently encountered after rewarming. Tetanus prophylaxis and debridement is indicated , but is not the most appropriate initial step in the management of patients with frostbite. CDEM Hypothermia.

A pneumonia caused by which of the following organisms is classically associated with currant jelly sputum? A. Klebsiella B. Mycoplasma C. Strep. pneumoniae D. viral E. H. influenzae

The answer is A. Klebsiella is not a common cause of community-acquired pneumonia. It generally occurs in the elderly, smokers, alcoholics, and those with other co-morbidities. It is classically associated with a bulging fissure on chest X-ray and currant jelly sputum.

An 82 year old man presents from the nursing home with fever, cough, nausea, vomiting and diarrhea. His vitals signs are T 102, P 65, BP 100/50, RR 24, and SpO2 92%. The most likely causative organism is: A. Legionella B. H. influenzae C. Strep. pneumoniae D. viral E. Mycoplasma

The answer is A. Legionella is an atypical pathogen commonly causing pneumonia. It is often found in the elderly or others with co-morbid illnesses. It is classically associated with GI symptoms and relative bradycardia.

A 14 year old presents just after smoking crack cocaine and complains of chest pain. He describes it as sharp and stabbing in the middle of his chest. His EKG is normal. The intern reads the CXR as "negative" but your supervising resident asks you to have another look (see Figure), after which you make the diagnosis of: [image] photo courtesy of eMedicine.com A. Pneumomediastinum B. Pneumonia C. Congestive heart failure D. Aortic dissection

The answer is A. Look closely along the left heart border and mediastinum. There is a thin strip of air. Pneumomediastinum and pneumopericardium result from Valsalva maneuvers, barotrauma, asthma, and cocaine inhalation from positive pressure devices. On physical exam there may be a Hamman's sign or mediastinal crunch heard over the precordium. Westermark's sign is dilation of pulmonary vessels proximal to a pulmonary embolism resulting in a cut-off appearance of the vessel on CXR.

60 year old male presents to the emergency department with chest pain. His monitor strip, shown below, reveals: [image] A. second degree AV block Mobitz Type 2 B. second degree AV block Mobitz Type 1 C. complete heart block D. first degree AV block

The answer is A. Mobitz type 2 is characterized by an unexpected, non-conducted atrial impulses. The R-R interval and P-R intervals are constant.

You suspect that your patient has swallowed a nail. Which of the following is an indication for endoscopic or surgical removal of this object? A. Radiography visualizes the nail in the gastric fundus B. The object has progressed from the jejunum through the ileum after 24 hours C. Abdominal CT scan shows a 1cm nail in the distal sigmoid colon D. Plain films do not reveal a radiopaque foreign body in the chest or abdomen

The answer is A. Observation can manage most ingested foreign objects - the patient should pass the object after several days. Propulsive agents can be given to speed movement along the gastrointestinal tract. Serial radiography monitors movement. Endoscopic or surgical intervention is indicated for sharp objects (which may cause perforation), objects greater than 2 cm in width (which are likely to lodge at the pylorus or the ileocecal valve), and long rigid objects (which may have trouble passing through the right angles of the duodenum). Surgery is indicated if an object fails to move after 24 hours (indicating impaction), or if the patient develops symptoms of obstruction or perforation. Body packers may have ingested substances such as cocaine or heroin, which can cause great harm should the packaging be disrupted. Proper management is observation, as most will pass the package(s) without complications. Urgent package retrieval should not be performed because of pressures from law enforcement. Endoscopy can in fact be dangerous to the patient, as it can disrupt the packaging and release toxic drugs. Endoscopic or surgical intervention is warranted should the patient develop signs of systemic drug toxicity. The patient can also be monitored with serial serum drug levels.

In a 70kg male DKA patient with serum glucose of 573 mg/dL, all of the following statements with regard to fluid and electrolyte imbalances are true EXCEPT: A. A normal magnesium level is reassuring and obviates the need for magnesium replacement. B. Despite a serum potassium level of 4.8 mEq, the patient is probably total body potassium depleted. C. The patient is likely to be total body phosphorus depleted. D. Serum sodium of 129 mEq represents dilutional hyponatremia and the corrected value is approximately 137 mEq. E. Total body water deficit is approximately 5L.

The answer is A. Patients with DKA are typically severely dehydrated with a total body water deficit of approximately 70-80 mL/kg, in addition to being total body depleted of potassium, magnesium, and phosphorous despite initially normal serum levels of these electrolytes.

Regarding the role of malignancy in the diagnosis of pulmonary embolism (PE): A. 25% of PE patients without identifiable risk factors are diagnosed with cancer within 2 years. B. Hematologic malignancies such as leukemia and have the highest incidence of venous thromboembolism C. The risk of PE is decreased in patients on chemotherapy D. Autopsy studies indicate that greater than 60% of patients who die of ovarian cancer have PE.

The answer is A. Patients with esophageal and laryngeal cancer, as well as leukemia and lymphoma have a low incidence of PE, whereas those with ovarian or colon cancers are at higher risk for developing PE. Autopsy studies indicate that up to 30% of patients who die of ovarian cancer have PE. Chemotherapy increases the risk of developing PE. Development of PE without any identifiable risk factors warrants a search for an underlying malignancy.

Regarding the treatment of hyperosmolar hypertonic nonketotic coma (HHNC) and its associated symptoms, which of the following is correct: A. Phenytoin (Dilantin) is often ineffective for seizures associated with HHNC. B. Half of the fluid deficit should be corrected over the first hour and the remainder over the following 8 hours. C. Since patients are not acidotic, close monitoring of glucose is not necessary. D. Hyperosmolarity should be corrected within the first few hours in the emergency department. E. In HHNC patients with severe dehydration, bleeding diathesis is a major clinical concern.

The answer is A. Phenytoin (Dilantin) is contraindicated in patients with HHNC as it may impair endogenous insulin release and is often ineffective in the management of seizures associated with HHNC. Half of the fluid deficit should be replaced over the first 8 hours, and the remainder over the ensuing 24 hours. Glucose must be tightly monitored as fluid resuscitation alone may normalize serum glucose or precipitate hypoglycemia in aggressive fluid resuscitation. Too-rapid correction of hyperosmolarity may result in development of cerebral edema, especially in children. Subcutaneous heparin should be considered in patients with severe dehydration due to increased risk of thrombosis from hypovolemia and hyperviscosity.

Which of the following are associated with a higher risk of spontaneous pneumothorax? A. Cigarette smoking B. Female gender C. Physical exertion D. Trisomy 21

The answer is A. Physical exertion has not been associated with an increased risk of spontaneous pneumothorax.

Regarding pulsus paradoxus and asthma, which of the following statement s is correct A. The presence of pulsus paradoxis in asthma indicates severe disease B. Pulsus paradoxus is pathognomonic for asthma C. The absence of pulsus paradoxis in asthma rules out severe disease D. Pulsus paradoxus is a fall in systolic blood pressure during inspiration

The answer is A. Pulsus paradoxus is defined as a fall in systolic blood pressure of greater than 10mm Hg upon inspiration. It is typically present during acute asthma exacerbations in severe asthma; however, its absence does not rule out severe disease. Although initially present, a pulsus paradoxus may disappear after only minimal improvement in air flow through the larger airways. Pulsus parodoxus may occur in other diseases besides asthma (for example, pericardial tamponade).

Generally speaking, a patient with a TIA history who presents with a new stroke, likely has which kind of stroke? A. thrombotic B. there is equal likelihood for any stroke type C. hemorrhagic D. hypoperfusion E. embolic

The answer is A. TIAs are associated with increased risk for thrombotic strokes, the result of ulceration of cerebral artery plaque. Patients with TIA have a 5 to 6% percent chance per year of having a stroke. Antiplatelet therapy reduces risk of stroke in these patients.

A pattern of diffuse PR depression suggests PE A. Tachycardia is the most common finding of EKG associated with PE. B. An S1Q3T3 pattern is pathognomonic for PE. C. Right bundle branch block indicates pulmonary infarction from PE. D. A pattern of diffuse PR depression suggests PE

The answer is A. Tachycardia and non-specific ST-segment and T-wave abnormalities are the most common findings on EKG of patients with PE; however up to 25% will have EKGs unchanged from their baselines. An S1-Q3-T3 pattern is suggestive but not diagnostic of PE. Right bundle branch block may also be seen in PE, but does not indicate pulmonary infarction or severity of PE. Diffuse PR depression is typically seen in pericarditis.

With respect to larygneal assessment, the Figure depicts what grading scale? [image] A. Cormack-Lehane B. LMA C. Macintosh D. Mallampati E. Miller

The answer is A. The Cormack-Lehane scale allows communication of relative ease of visualization of laryngeal structures during laryngoscopy and intubation. The Miller and Macintosh are types of laryngoscope blades, and the LMA (laryngeal mask airway) is a type of airway.

Of the following choices, which diagnosis is most likely in a 53-year old cancer patient with chest pain, dyspnea, and the EKG shown in the Figure? [image] A. pulmonary embolism B. third degree heart block C. pericarditis from metastatic disease D. infectious myocarditis E. anterolateral myocardial ischemia

The answer is A. The EKG's S1Q3T3 pattern (S in lead I, Q and inverted T in lead III), while not pathognomonic, is suggestive of PE as a cause of this patient's symptoms.

A healthy 32-year old female comes to the emergency department complaining of acute severe lower back pain which is worse with coughing. Her lower spine is tender to palpation. Three days prior she had a lower extremity surgical procedure performed under epidural anesthesia. The surgery and post-operative period were uneventful. The etiology of this patient's pain is most likely: A. Spinal epidural hematoma B. Spinal epidural abscess C. Adhesive arachnoiditis D. Anterior spinal artery thrombus

The answer is A. The back pain in this patient is likely secondary to a space-occupying lesion in the spinal canal. Epidural hematoma is the most likely option since it was sudden in onset, worse with coughing and occurred soon after the procedure. Adhesive arachnoiditis (A) would be manifest as a progressive loss of nerve function. Anterior spinal artery thrombus would be expected to present with painless paraplegia (B). A spinal epidural abscess would be unlikely so soon after the procedure and is more gradual in onset. The patient is not immunosuppressed and she does not have a fever, rigors or sweats which can be seen in up to 75% of patients.

The treatment of cardiogenic shock may include all of the following EXCEPT: A. Phenylephrine B. Treatment of ischemia C. Dobutamine D. Intra-aortic balloon pump E. Dopamine

The answer is A. The goal of therapy is to improve oxygenation, minimize ischemia, improve pump function, and decrease afterload. Dobutamine is the agent of choice in the setting of heart failure. An intra-aortic balloon pump may be a temporizing measure. Phenylephrine would increase afterload and worsen cardiac output.

A 28 year old man is shot in the right chest. His vital signs are: T 97.9, HR 120, BP 70/50, RR 12, SPO2 84%. He has significant JVD, his trachea is deviated to the left, and he has decreased breath sounds on the right. The patient's diagnosis is: A. Tension pneumothorax B. Spinal cord injury C. Hypovolemic shock D. Lung laceration E. Cardiac tamponade

The answer is A. The patient has a gunshot wound to the right chest near the sternum. His hypotension and JVD suggest an obstructive shock. Given the tracheal deviation to the left and absent breath sound on the right, he most likely has a tension pneumothorax. His emergent treatment includes needle decompression followed by chest tube placement.

A 15 year old boy dives into a swimming pool, hits his head on the bottom, and subsequently is found to have no sensation or motor function below the nipple line. His vital signs are: T 97.9, HR 76, BP 80/40, RR 12, SPO2 84%. He has no JVD and his lungs are clear. The patient's diagnosis is: A. Neurogenic shock B. Sepsis C. Spinal shock D. Near drowning E. Hypovolemic shock

The answer is A. The patient is most likely suffering a spinal cord injury, producing a disruption of the autonomic nervous system leading to vasodilation and hypotension (without the expected tachycardic response). This entity, called neurogenic shock, is a type of distributive shock like anaphylactic shock It is important to rule out other internal injuries in this patient, and then institute therapy with a pressor agent such as phenylephrine.

A thin 18 year old female complains of acute onset of sharp right-sided chest pain this morning. She has developed some mild shortness of breath during the morning and thought she should get it checked out. Her chest X-ray is shown in the Figure. The next course of action should be: [image] A. Chest tube placement B. Antibiotics C. Electrocardiogram D. Needle decompression

The answer is A. This patient has a large right-sided spontaneous pneumothorax that is not under tension. She needs oxygen and chest tube placement. This can be done with proper procedural analgesia and sedation since there is no immediate threat. Primary spontaneous pneumothorax tends to occur in healthy young men (and, less commonly, women) of taller than average height. Other risk factors include cigarette smoking, asthma, COPD, interstitial lung disease, connective tissue diseases, and lung cancers.

You are performing procedural sedation on a 42 year old male who is quite muscular and requires high doses of analgesia to reduce a dislocated hip sustained during an MVA. He begins to show signs of respiratory depression and needs manual ventilation with a bag valve mask. You are having great difficulty providing manual ventilations due to presumed spasm of the glottis and rigidity of the chest wall musculature. Which of the following analgesics was likely used? A. Fentanyl B. Morphine C. Demerol D. Dilaudid

The answer is A. This patient is exhibiting clinical signs of chest wall rigidity and glottic spasm, which is a rare but classic side effect of using high doses of intravenous fentanyl. Chest wall rigidity and glottic spasm, which may make ventilation difficult, are unique complications seen with very high doses of fentanyl given rapidly (generally > 15 mcg/kg). It has been observed at lower doses. This may not reliably be antagonized by naloxone and may require neuromuscular blockade and intubation to enable adequate ventilation. This complication is very rarely reported with the dosages of fentanyl used for PSA but can still happen.

Treatment with hyperbaric oxygen (HBO) is associated with contraindications. Which of the following is not a relative or absolute contraindication to HBO? A. pregnancy B. untreated pneumothorax C. COPD with air trapping D. otitis media

The answer is A. Untreated pneumothorax is an absolute contraindication to HBO therapy. The reason is concern that it can progress to tension pneumothorax, especially during the decompression phase of therapy. The COPD patient with a large bleb represents a relative contraindication for similar reasons. Treatment with doxorubicin, and many other drugs -- such as cisplatin (Cisplatinum®), bleomycin (Blenoxane®), disulfiram (Antabuse®), and mafenide acetate (Sulfamylon®) -- contraindicates HBO therapy because of potentially toxic effects when combined with HBO. URI illnesses such as otitis media are relative contraindications, due to the potential for tympanic membrane rupture secondary to inability of the ears to equalize pressure during therapy. This can be addressed through myringotomy with placement of tubes (in cases where multiple HBO treatments are anticipated). In pregnant patients, HBO therapy has been shown to be safe for the fetus when given at appropriate levels and "doses" (durations). In fact, pregnancy lowers the threshold for HBO treatment of carbon monoxide-exposed pregnant patients. This is due to the high affinity of fetal hemoglobin for CO.

A 60 year old male presented to the emergency department with chest pain. He subsequently became unresponsive. The monitor shows the rhythm below. The rhythm is: [image] A. ventricular tachycardia B. atrial flutter C. atrial fibrillation with rapid ventricular response D. sinus tachycardia

The answer is A. Ventricular tachycardia is wide and complex. It is distinguished from supraventricular tachycardia by width and morphology of the QRS complexes. (Though there are numerous exceptions, supraventricular tachycardias usually exhibit narrow QRS complexes with morphology similar to that when the patient is in sinus rhythm.)

A 72-year-old male presents with five hours of substernal chest pain and pressure despite taking three sublingual nitroglycerin. You order an EKG. What findings on the EKG would indicate that this patient is potentially a candidate for thrombolytic therapy? A. Ventricular tachycardia B. ST-segment elevation of at least 1 mm in two or more contiguous leads C. ST-segment depression of at least 2mm in any precordial lead D. Atrial fibrillation with a rapid ventricular response

The answer is B. "Fibrinolytic therapy is indicated for patients with STEMI (as a reperfusion option) if time to treatment is <6 to 12 hours from symptom onset, and the ECG has at least 1-mm ST-segment elevation in two or more contiguous leads."

A 46 year old male presents with acute onset frontal and bitemporal headache, associated with neck pain and sensitivity to bright lights. In the emergency department, the patient is febrile, reports several episodes of vomiting, and complains of worsening neck pain with head movement. A lumbar puncture is performed, the results of which are consistent with viral meningitis. A day later the patient complains of worsening headache. Which of the following is correct regarding this complication of lumbar puncture (LP)? A. Post LP headaches are typically unilateral and worse with supine position B. Post LP headache is the most common complication of lumbar puncture C. A larger diameter needle decreases the incidence of post LP headache D. Lying supine for up to six hours will help prevent post LP headache

The answer is B. A smaller-diameter needle (not larger) is associated with a lower incidence of post-puncture headache because it causes a smaller dural hole (A). Simple analgesics are commonly prescribed, but they have no apparent advantage over bedrest and fluid intake. Lying supine for up to six hours carries no advantage over getting up after the procedure in the prevention of post spinal headache (B). Post LP headaches are typically bilateral and worse when sitting up (D). They improve with the supine position. Treatment of post-LP headache commonly involves keeping the patient supine to maximize intracranial CSF volume, use of oral caffeine, and for severe long-lasting headaches, autologous blood patch. The blood patch involves injecting one's own blood at the LP site in order to form a clot around the meningeal puncture site to avoid further leakage.

A 60 year old male with known coronary artery disease presents complaining of recent chest pain. The chest pain typically occurs after exertion and lasts about 15 minutes. He takes a sublingual nitroglycerin or rests and the pain subsides. He is currently pain free. He has had similar episodes for the last 6 months with no change in frequency or intensity of the chest pain. He most likely has: A. acute myocardial infarction B. stable angina C. unstable angina D. acute coronary syndrome E. variant (Prinzmetal's) angina

The answer is B. Acute coronary syndrome is a spectrum of myocardial ischemia through myocardial necrosis. The spectrum includes unstable angina, stable angina and acute myocardial infarction. Unstable angina is of new or recent onset, of changing character, or angina at rest. Stable angina or angina pectoris is chronic and episodic, usually lasting 5 to 15 minutes and relieved by rest or nitroglycerin. Variant angina usually occurs at rest, often precipitated by tobacco or cocaine use. It is defined as ST elevation that resolves as pain goes away. It is thought to be due to artery spasm.

With respect to laboratory findings in diabetic ketoacidosis (DKA) and hyperglycemic hyperosmolar nonketotic coma (HHNC), all of the following guidelines are generally true EXCEPT: A. BUN is elevated more in patients with HHNC (>50 mg/dL) than in patients with DKA (25-50 mg/dL). B. Serum bicarbonate is typically severely low (<10mEq) in patients with either DKA or HHNC. C. Serum ketones are present in patients with DKA but not usually in patients with HHNC. D. Patients with HHNC typically have blood glucose > 700 mg/dL, whereas patients with DKA have blood glucose > 350 mg/dL. E. Serum osmolality in patients with HHNC is typically > 350 mOsm/L.

The answer is B. Although patients with DKA typically have profound metabolic acidosis with serum bicarbonate < 10mEq, acidosis is typically absent in patients with HHNC and serum bicarbonate is usually > 15 mEq.

All of the following are commonly used in the supportive treatment of thyroid storm EXCEPT: A. acetaminophen to manage hyperpyrexia B. amiodarone to control dysrhythmias C. corticosteriods D. oxygen E. diuretics to treat congestive heart failure

The answer is B. Amiodarone is an iodine-rich antidysrhythmic with poorly-defined effects on thyroid function that has been associated with both hyperthyroidism and hypothyroidism. It should therefore be avoided in the management of thyroid disease. Propranolol is standard therapy in thyroid storm and, in addition to its effects of adrenergic blockade, also may reduce dysrhythmias. Of note, aspirin should be avoided in the treatment of hyperpyrexia as it may increase the level of active thyroid hormone by displacing thyroid hormone from thyroglobulin.

A patient presents within an hour after sustaining a laceration caused by a knife into palm. Which of the following regarding this patient/presentation is TRUE? [image] A. Randomized clinical trials have established the importance of prophylactic antibiotics in patients such as this one. B. An ulnar nerve block would be a preferred method for wound anesthesia. C. Vertical mattress sutures should be used for wound closure. D. The wound should be closed with a topical skin adhesive such as 2-octyl cyanoacrylate. E. Since it is difficult to sufficiently irrigate the palm, the laceration should be left open.

The answer is B. An ulnar nerve block provides anesthesia and is likely less painful than direct injection into the wound (which is another reasonable approach to wound anesthesia). Vertical mattress sutures are not recommended for the palm, as this technique places deep structures at risk. Topical skin adhesives are best avoided in the palm, which is prone to sweating and thus increasing the possibility of resultant wound dehiscence. Though many would recommend use of prophylactic antibiotics in a patient with a sufficiently deep palmar laceration, no controlled trials address use of antibiotics in patients with this - or just about any other - type of laceration.

As a general rule, when is blood transfusion indicated in the treatment of hypovolemic shock resulting from acute hemorrhage? A. first line treatment B. massive hemorrhage > 30% C. minor hemorrhage <10% D. after 1L of crystalloid bolus E. after dopamine

The answer is B. Blood transfusion can play a vital role in the treatment of hypovolemic shock from acute hemorrhage. It is generally not the first line treatment. It is indicated in massive blood loss or shock that is not responsive to significant crystalloid infusion (2L or 30 ml/kg). Pressors are not indicated in hypovolemic shock. Elderly patients and those with co-morbid illnesses may require blood products earlier than healthy adults.

Regarding the epidemiology of asthma in the United States, which of the following is true? A. Etiology is thought to be genetic, not environmental B. Prevalence increased in the 1980's, and then decreased in the 1990's C. More common in males than females in adult and pediatric populations D. Incidence is comparable for Caucasians and African-Americans

The answer is B. Despite an increase in asthma prevalence in the United States, Canada, Great Britain and Australia in the 1980s, the 1990s saw a decrease in prevalence in these areas. Regarding gender, male children are more likely than female children to have asthma, however the reverse is true with adults. African-Americans have a higher prevalence of asthma than Caucasians. Migrants who relocate from an area of low asthma prevalence to an area of high asthma prevalence tend to have an increased prevalence of asthma suggesting a role for environmental factors in the development of asthma.

A 65-year-old female presents 2 weeks after an MI complaining of chest pain, fever, and shortness of breath. She has a new friction rub on exam and a leukocytosis. She most likely has: A. pneumonia B. Dressler's syndrome C. congestive heart failure D. pulmonary embolism E. new myocardial infarction

The answer is B. Dressler's syndrome is fever, pleuritis, leukocytosis, pericardial friction rub, and evidence of pericarditis or pleural effusion occurring several weeks after MI. It is thought to be autoimmune in nature and is treated with NSAIDs.

A 64 year old female presents to the emergency department with chief complaints of occipital headache and chest pain. Physical examination reveals a blood pressure of 200/118 as well as edema of the optic disk. Of the diagnoses below, the most likely is: A. hypertensive urgency B. hypertensive crisis C. white-coat hypertension D. acute hypertensive (non-emergency/non-urgency) episode E. moderate hypertension

The answer is B. Elevated blood pressure in the setting of optic disk edema is a hallmark of malignant hypertension (also known as hypertensive emergency or hypertensive crisis). While hypertensive urgency is not consistently defined in the medical literature, this patient's presentation indicates that there is some end-organ damage and thus the diagnosis is malignant hypertension. The "white-coat" syndrome, in which patients' blood pressures are elevated only in the clinical setting and not at home, has been shown to account for as many as a fifth of all cases of newly diagnosed "hypertension." Understanding of this phenomenom is important for emergency physicians, since its frequency explains why patients should not be given a diagnosis of new-onset hypertension based on E.D. measurements.

Which of the following descriptors of epidural hematoma is FALSE? A. Present in only about 1% of severe head injury patients B. Most often a result of a skull fracture that traverses a venous sinus C. Biconcave blood collection between the skull and dura D. Immediate surgical evacuation is indicated E. Classically associated with a "lucid" interval prior to coma

The answer is B. Epidural hemorrhage is most often associated with skull fracture across the course of the middle meningeal artery.

Which of the following is not a common sign or symptom of thyrotoxicosis? A. congestive heart failure B. hypothermia C. hyperhidrosis D. nervousness E. tachycardia

The answer is B. Fever, not hypothermia, is commonly seen in thyrotoxicosis. Other common signs and symptoms include tachycardia, congestive heart failure, wide pulse pressure, tremor, thyrotoxic stare, thyromegaly, nervousness, weight loss, and palpitations.

All of the following statements about lumbar punctures are true EXCEPT: A. In the adult and older pediatric population, lumbar punctures may be performed as high as the L2/L3 interspace and as low as the L5/S1 interspace. B. Patients should be told to keep their neck in maximal flexion throughout the procedure. C. The subarachnoid space extends to the S2 vertebral level. D. Patients are positioned in lateral recumbent position with their lower back arched toward the physician. E. In locating the puncture site, a line connecting the posterior superior iliac crests will intersect the midline at approximately L4.

The answer is B. Flexion of the neck does not facilitate the LP to any great extent and may add to a patient's discomfort as well as compromise the unconscious patient's airway. Typically, patients are positioned in the lateral recumbent position but an LP may be performed in the upright seated position. This position may be desirable in the pediatric population where the midline may be more difficult to identify (answer A). Answers B, C, and E are all true statements regarding the anatomy relevant to performing a lumbar puncture.

A 35 year old female is brought to the emergency department after family members called the police to say she was threatening to kill herself by jumping out a window. She has a long history of depression. Regarding suicide, which of the following statements is FALSE? A. All states have laws giving law enforcement the right to place into custody any individual suspected of being a danger to themselves or others. B. Family sitters provide the best option for close observation of suicidal patients since they often have a calming influence on the patient. C. The patient's room needs to be cleared of all potentially dangerous objects such as blunt instruments, glass objects, and the patient's belongings. D. The majority of suicide attempts involve minor injuries or drug overdoses.

The answer is B. For any patient presenting with suicidal ideation, the emergency department physician must first stabilize medical condition as most attempts involve minor injury or drug overdoses treatable by emergency room staff. Secondly, all objects and substances should be kept strictly out of reach of the suicidal patient. Once stabilized these patients may be kept under direct supervision. It is not recommended that family members provide the direct supervision because of a possibility of collaboration between family and patient to leave the hospital. In this event, or even before coming into the hospital, all law enforcement has the right to bring into custody any patient at risk of harming self or others.

Regarding the diagnosis and treatment of hypoglycemia, which of the following is correct? A. Glucagon, administered intramuscularly or subcutaneously, is a safe and universally effective means for increasing blood sugar in hypoglycemic patients. B. Hypoglycemia can present with virtually any neurological deficit. C. Hypoglycemia in adults is typically symptomatic at or below serum glucose of 60 mg/dL. D. Patients who overdose on oral hypoglycemic agents such as sulfonylureas must have their serum glucose monitored for a minimum of 6 hours before emergency department discharge. E. Patients with type 1 diabetes do not typically develop hypoglycemia.

The answer is B. Glucagon is ineffective in patients without adequate glycogen stores, as would be expected in alcoholics. Further, glucagon can precipitate a severe lactic acidosis in patients with glycogen storage diseases and therefore should not be used in children with hypoglycemia of unknown etiology. Typical symptoms of hypoglycemia include sweating, tachycardia, nervousness, hunger, and neurologic symptoms. Symptoms should not be attributed to hypoglycemia unless the level falls below 40-50 mg/dL. Type 1 diabetics practicing strict control of serum glucose are at high risk for hypoglycemic episodes precipitated by skipping a meal, or by increasing energy output or insulin dose. Due to the extended half-lives of the oral hypoglycemic agents, hospitalization and 24-hour observation (at minimum) are the typical management for overdose of these agents.

A 25-year old female presents to the ED with dyspnea and chest pain. Chest CT, with contrast, is performed and some pertinent "slices" are shown in the Figure. What is the diagnosis? [image] A. Aortic dissection, Type I B. Bilateral pulmonary embolism C. Gas embolism D. Acute Respiratory Distress Syndrome

The answer is B. Helical CT studies of the pulmonary vasculature are increasingly used for detection of pulmonary embolism. Though there are questions about CT's ability to detect small (e.g. subsegmental) emboli, CT scans have high sensitivity for proximal embolism such as that depicted in the accompanying figure. The patient whose images are shown was found to have moderate-severe right ventricular dysfunction and received thrombolytic therapy in the ED - she had an excellent outcome.

Symptoms of secondary adrenal insufficiency include all of the following EXCEPT: A. anorexia B. hyperpigmentation C. nausea and vomiting D. weight loss E. weakness

The answer is B. Hyperpigmentation is seen in greater than 90% of primary adrenal insufficiency. It is a result of compensatory adrenocorticotropic hormone (ACTH) and melanocyte-stimulating hormone (MSH) secretion. The secretion is a feedback mechanism that is not activated in secondary adrenal insufficiency, for example, adrenal insufficiency from pituitary infarction or hypothalamic insufficiency.

A 25 year old male is brought to the emergency department by his family, with a complaint of feeling depressed for the past week. In obtaining the history, which of the following statements regarding this patient would support a diagnosis of major depression? A. The patient has no history of medical illness B. The patient has a history of Crohn's disease C. There is no family history of depression D. The screening mnemonic in SAD CAGES is negative

The answer is B. In SAD CAGES is a screening mnemonic for symptoms, not causes, of major depression. Depression is more common in patients with history of other medical illnesses, some of which may actually cause depressive symptoms. As compared with major depression, dysthymic disorder is a more chronic, and less severe, form of depressive illness.

For a young otherwise healthy patient in anaphylactic shock, the initial best treatment of those listed below is: A. Broad spectrum antibiotics B. Epinephrine C. Steroids D. Diphenhydramine E. Dopamine

The answer is B. In a patient suffering from anaphylactic shock, the correct emergency treatment is epinephrine (especially in a young person in whom coronary disease is not a major concern). Steroids and diphenhydramine play a role in treatment of allergic reaction, but they are of a lower priority in anaphylaxis.

A 29-year-old male presents to the emergency department complaining of substernal chest pressure. The patient used cocaine and alcohol 3 hours prior to admission. On exam, the patient has a blood pressure of 160/100 mm Hg and heart rate of 150 beats per minute with ST-segment changes in the inferior leads on EKG. Which of the following is the best medication to treat the patient's cardiovascular status? A. Lidocaine B. Lorazepam C. Metoprolol D. Phenoxybenzamine

The answer is B. In a patient with suspected myocardial ischemia secondary to cocaine abuse, beta blockade is probably contraindicated as it may lead to uncontrolled alpha-agonism and could cause worsening hypertension (this notion continues to be debated). Lidocaine is contraindicated and the use of nitroglycerin is controversial.

A 19 year old female college student presents to the emergency department with fever, headache, and confusion. Physical exam reveals T103. She is lethargic. The HEENT exam is normal, she has nuchal rigidity, and her lungs are clear. Of the following choices, the next step in her treatment should be: A. azithromycin IV B. ceftriaxone IV C. levofloxacin PO or IV D. head CT E. head CT, followed by lumbar puncture

The answer is B. In patients with meningitis, early antibiotics administration is of the utmost importance. Antibiotic administration should not be delayed to await diagnostic work up. Ceftriaxone, administered in some regions with vancomycin depending on the resistance profile of likely etiologic agents, is generally considered an antibiotic of choice in meningitis. Azithromycin and levofloxacin do not have good CNS penetration and therefore are not indicated for meningitis.

Which of the following psychiatric disorders is associated with the greatest increased risk of committing suicide? A. schizophrenia B. panic disorder C. major depression D. post-traumatic stress disorder (PTSD)

The answer is B. Most people who commit suicide suffer from either alcoholism or a diagnosable psychiatric illness. 15-20% of people with major depression and 10% of people with schizophrenia will commit suicide. Up to 40% of people with panic disorder will attempt suicide at some point in their lives. PTSD also carries an increased risk.

Which of the following is true about myasthenia gravis? A. The "atropine test" is diagnostic when 0.5 mg of atropine is given intravenously and the patient's symptoms improve within two minutes. B. A myasthenic crisis involves an exacerbation of weakness, especially of respiratory muscles, often necessitating intubation. C. Cooling exacerbates the symptoms, and heat alleviates them. D. It typically presents as an ascending weakness of the peripheral nervous system. E. Weakness improves as the involved muscles are used repeatedly.

The answer is B. Myasthenia gravis is an autoimmune disease that results from antibodies directed against the acetylcholine receptor (AChR) at the neuromuscular junction. Destruction of the AchR leads to fewer receptors available to bind acetylcholine, with a resulting muscle weakness. Ocular symptoms are usually the first to occur, with diplopia and ptosis being common. The disease typically worsens as the day progresses because of repeated use of the muscles involved. Diagnosis is made with the tensilon test, where edrophonium is given and the patient's symptoms are observed to transiently improve. The administration of atropine is not a diagnostic test. Cooling helps the symptoms and heat exacerbates them. A myasthenic crisis is a feared complication. Patients develop respiratory failure requiring intubation, frequently for prolonged periods.

Which of the following is correct regarding the use of corticosteroids in acute asthma exacerbation? A. Beneficial effects occur within the first hour of administration. B. Inhaled steroids should be avoided C. Intravenous steroids are superior to the oral route D. Tapering is needed with all corticosteroid regimens

The answer is B. Oral and intravenous steroids are equally efficacious in treating an asthma exacerbation. Yet, in the setting of a severe asthma exacerbation, a patient may have difficulty taking oral medications and the intravenous route is preferred

A 44 year old male presents with fever, cough and shortness of breath. He is a nonsmoker. He has a lobar infiltrate on CXR. The most likely cause of pneumonia is: A. H. influenzae B. Strep pneumoniae C. Moraxella catarrhalis D. Mycoplasma pneumoniae E. viral

The answer is B. Streptococcus pneumoniae is the most common cause of community- acquired pneumonia. Viral is the second most common cause.

A 36 year old woman presents to the emergency department two hours after the sudden onset of a severe occipital headache and nausea. She has a history of migraine headaches that typically occur in the right frontal area and are associated with an aura. Her temperature is 98.8 degrees Fahrenheit, her neck is supple, and her neurological exam is normal. A non-contrast CT scan of her head is normal. Of the options below, what is the next step in her management? A. Consult a neurologist for evaluation of atypical migraines. B. Perform a lumbar puncture to rule out the possibility of subarachnoid hemorrhage. C. Discharge her home with prochlorperazine and close instructions to return if her symptoms worsen. D. Observe for 6 hours, administer acetaminophen and normal saline, and discharge home if she feels better. E. Observe for 6 hours and then obtain a repeat CT scan; if normal, discharge home.

The answer is B. Sudden onset headache with nausea, vomiting, photophobia, or neck stiffness should raise the concern for spontaneous subarachnoid hemorrhage. Sensitivity of a non-contrast CT scan varies with respect to many factors (e.g. time since bleed) but is generally in the range of 90%; therefore, if the clinical suspicion is high, a lumbar puncture should be performed and a cell count for red blood cells done.

Regarding mycobacterium tuberculosis (TB), which of the following is TRUE? A. Worldwide, approximately 45-50% of the population is infected with TB B. Worldwide, TB causes more deaths than any other infectious agent C. In the United States, approximately 12-15% of the population is infected with TB. D. In the United States, the incidence of TB has been steadily increasing since the 1950s.

The answer is B. TB causes more deaths worldwide than any other infectious agent. Approximately one-third of the world's population is infected with TB. In the United States, only about 4-6% of the population is infected with TB. The incidence of TB infection in the United States began decreasing in the 1950s as a result of advancements in public health and pharmacologic treatments. Yet, the incidence has been rising since the 1980s, especially in poor, homeless, minority, and prison populations. The incidence of multi-drug resistant TB has been gradually increasing, especially among AIDS patients.

f the following, which diagnosis is most likely given the EKG shown in the Figure? [image] A. posterior myocardial infarction B. inferior myocardial infarction C. anteroseptal myocardial infarction D. anterior myocardial infarction

The answer is B. The EKG demonstrates classic findings (ST-segment elevations in II, III, AVF) associated with inferior myocardial infarction.

Regarding the diagnosis and treatment of thyroid storm in the emergency department, which of the following is true? A. A stat thyroid-stimulating hormone (TSH) level is required to make the diagnosis. B. Patients suspected of having thyroid storm should undergo treatment prior to a definitive diagnosis due to the potentially life-threatening nature of this disease. C. The diagnosis of thyroid storm is generally a straightforward clinical diagnosis and rarely confused clinically with other disorders such as psychiatric or other endocrine disorders. D. Thyroid storm cannot be diagnosed in the absence of altered mental status. E. Treatment of thyroid storm should only be undertaken after consultation with an endocrinologist.

The answer is B. The diagnosis of thyroid storm in the emergency department may be challenging due to the relatively infrequent occurrence of the disease and its typically nonspecific signs and symptoms. Treatment should be initiated in a timely fashion in any patient suspected of having thyroid storm due to the potential lethality of this disease. Immediate laboratory testing is typically not available to confirm clinically suspected cases, although thyroxine (T4) radioimmunoassay and free T4 index are good screening tests for hyperthyroidism. Clinical presentation of thyroid storm may be mistaken for psychiatric illness, heat stroke, sympathomimetic toxidromes, hypoglycemia and withdrawal syndromes, among others. Altered mental status, though frequently present, is not a prerequisite for diagnosis.

A 23 year old college novice mountain climber decides to climb a mountain with friends as a graduation present. His first night is spent at an altitude of 8,500 feet at a mountain resort. The next morning he starts to experience a mild headache and nausea. His symptoms get worse throughout the day. His friends want to get to 10,000 feet by nightfall. As a physician at the hotel, the worst advice you can give him is: A. Take supplemental oxygen. B. Ascend with the rest of the team. C. Take acetazolamide. D. Take ibuprofen.

The answer is B. The syndrome of high altitude illness ranges from mild AMS (Acute Mountain Sickness) to life threatening conditions of HAPE (High Altitude Pulmonary Edema) and HACE (High Altitude Cerebral Edema). This student is experiencing mild AMS. After the symptoms of altitude illness occur, further ascent to a higher sleeping altitude is contraindicated. Halting ascent or activity to allow further acclimatization may reverse symptoms. Acetazolamide is a carbonic anhhydrase inhibitor that induces a renal bicarbonate diuresis, causing a metabolic acidosis and thereby increasing ventilation and arterial oxygenation. Supplemental oxygen addresses the hypoxic insult of high altitude exposure. Ibuprofen is useful for the treatment of his headache. Dexamethasone can help with the symptoms of AMS, but does not play a role in acclimatization.

Which coronary vessel is usually the cause of the myocardial infarction in a patient with ST elevation in V1, V2, and V3? A. right coronary artery (RCA) B. left anterior descending (LAD) C. right ventricular branch of the right coronary artery D. left circumflex artery E. posterior descending branch of the right coronary artery

The answer is B. This EKG pattern is consistent with that of anterior wall myocardial infarction (MI). The LAD supplies the anterior wall of the myocardium. The left circumflex artery, the LAD, or a branch of the RCA supplies the lateral wall of the left ventricle. Proximal occlusion of the LAD will give ST elevation in leads V1-6, aVL and I (an anterolateral MI). Occlusion of a branch of the RCA will result in an inferolateral MI (ST elevation in leads II, III, aVF and I, aVL, V5 and V6). The RCA supplies the inferior wall and SA node. Occlusion in leads II, III and aVF causes an inferior MI. The right ventricle is usually supplied by the RCA or, less commonly, a dominant left circumflex. ST elevation in leads V4 and V5 of a right-side leads EKG suggests infarction of the right ventricle. A posterior MI (ST depression in V1-V3) results from occlusion of the RCA, its posterior descending branch, or a dominant left circumflex.

A patient sustains a forearm laceration as shown in the Figure. Regarding the wound, which of the following is true? [image] A. Because of the high tension on the wound, a Penrose drain should be placed to reduce chances of infection. B. "Undermining" subcutaneous tissues may be a useful technique to reduce post-closure skin tension. C. All visible fatty tissue should be excised before wound closure. D. Due to presence of the flap, the wound should be closed with 6-0 suture. E. After shaving of the nearby hair, the wound is a good candidate for closure with tissue adhesives.

The answer is B. Undermining of skin "recruits" tissue by separating the skin from the deeper subcutaneous structures. Though in some cases, slight excision of protruding fatty tissue may be necessary, trimming of all visible fatty tissue is unnecessary. The wound is subject to high tension, and thus 6-0 suture or tissue adhesives are not optimal choices for closure. Drains are rarely indicated for initial wound care in the ED, and would not be necessary in the wound in the Figure.

Regarding the etiology of viral pneumonia, which of the following is true? A. Influenza virus is the most common cause of viral pneumonia in children B. Cytomegalovirus does not usually cause pneumonia in immunocompetent adults C. Parainfluenza virus is the most common cause of viral pneumonia in adults D. Varicella zoster causes pneumonia more frequently in children than in adults

The answer is B. Varicella zoster virus (VZV), the etiologic agent of chicken pox, more commonly presents as pneumonia in adults, especially smokers or pregnant women. Influenza outbreaks, which occur in winter months, may cause up to 40,000 deaths annually in the United States with elderly people making up more than 90% of the victims.

Suicide risk is increased in this patient population: A. Patients who have not been involuntarily committed B. Patients who directly questioned about suicide C. Patients who are elderly and Caucasian D. Patient who takes lithium for bipolar affective disorder

The answer is C.

A 58-year-old male previously in good health presents with chest pain for two hours. Vital signs are BP 126/78, HR 80 (sinus rhythm), RR 14, oxygen saturation 99%, T 36.8. His EKG shows ST segment elevation in leads II, III, aVF and V1. ST-segment elevation is greater in lead III than in lead II. What additional diagnostic test is indicated prior to giving nitroglycerin? A. d-dimer B. Echocardiogram C. EKG with right-sided leads D. CXR

The answer is C. "Nitrate-induced hypotension is also suggestive of right ventricular infarction, and of tamponade. Initial therapy for both would include volume loading and avoidance of vasodilators or other agents that may lower the blood pressure." "ST segment elevation in lead V1 in the setting of inferior MI (i.e., ST segment elevation in leads II, III, and aVF rather than in the setting of concomitant ST segment elevation in all anterior precordial leads) is suggestive of right ventricular infarction." "ST segment elevation is usually greater in lead III than in lead II when right ventricular infarction coexists with inferior AMI." "Application of "right-sided" precordial leads is the best means to diagnose right ventricular infarction with the ECG. These leads, as a mirror image of the left precordial leads, demonstrate ST segment elevation with right ventricular infarction in leads V3R to V6R, with V4R having the highest sensitivity."

A 48 year old farmer is plowing his field when a thunderstorm rapidly overcomes him. Drivers on a nearby highway see him struck by lightening. You respond to the scene with EMS. What is the least likely finding on physical exam? A. cardiac asystole B. Glascow Coma Score of 3 C. extensive skin burns D. respiratory arrest

The answer is C. A lightning strike is the discharge of a massive amount of current over a very short period of time. This often causes "short-circuiting" of electrical systems such as heart, respiratory centers, and central and autonomic nervous systems, in addition to arterial and muscular spasm. However, significant skin burns and deep tissue destruction seldom occur.

Which of the following is NOT a potential indication for hyperbaric oxygen therapy? A. carbon monoxide poisoning B. decompression sickness C. necrotizing enterocolitis D. necrotizing fasciitis

The answer is C. According to the Undersea and Hyperbaric Medical Society, items A-D are all reasonable indications for the use of hyperbaric oxygen therapy. Other potential indications include patients with air or gas embolism, crush injury, and compartment syndrome.

Which of the following is a risk factor for developing pulmonary embolism A. Albumin deficiency B. Alcohol use C. Immoblization D. Corticosteroid use

The answer is C. Alcohol use has not been associated with an increased risk for pulmonary embolism. Virchow's triad of venous stasis, endothelial injury and hypercoagulability is predictive of the long list of risk factors for pulmonary embolism. Other risk factors include history of deep venous thrombosis or pulmonary embolism, malignancy, recent surgery, immobilization, hematologic abnormalities such as factor V Leiden deficiency, protein C or S deficiency, antithrombin III deficiency or homocysteinuria, among others.

A 22 year-old man, recently released from hospital with a newly diagnosed psychiatric disorder, was found dead at his home from an overdose of medications prescribed by his doctor. Of the following drugs, which one (taken in isolation), would be most likely to be associated with fatal outcomes in an overdose scenario? A. fluoxetine B. lithium C. amitriptyline D. lorazepam

The answer is C. Antidepressant overdose is the most common cause of suicide by ingestion. Cyclic antidepressants are associated with a higher potential for lethality than other antidepressant medications. Often during the early stages of recovery from major depression, patients may have a "mobilization of energy" which allows them to act on their suicidal thoughts, for which they previously lacked the energy.

When using the "SAD PERSONS" scale to determine suicide risk, which factor conveys the least amount of points? A. stated future intent B. depression or hopelessness C. separated, divorced or widowed D. rational thinking loss

The answer is C. Being separated, divorced or widowed is an important but less significant factor in determining suicide risk and so is assigned 1 point on the suicide scale. All the others are high-risk factors and are each assigned 2 points on the suicide scale. A score of 6 or more has a sensitivity of 94% and a specificity of 71% compared with formal psychiatric evaluation to identify the need for hospitalization in patients who present immediately after a suicide attempt.

A 51-year-old male with long-standing hypertension presents with abrupt onset of severe chest pain radiating to the back. He describes a tearing sensation. Vital signs are HR 110, BP 175/105, RR 20, T 37.4. EKG shows LVH. CBC, electrolytes, BUN/Creatinine are all normal. CXR is as shown below. What diagnostic test would be most appropriate for making a definitive diagnosis at this time? [image] A. MRI of the thoracic spine B. Esophagram using Gastrograffin C. CT of the chest with IV contrast D. Aortogram

The answer is C. CXR source: http://cdemcurriculum.org/ssm/cardiovascular/images/cxr_with_widened_mediastinum.jpg http://cdemcurriculum.org "CT of the chest is the test most often used to confirm the diagnosis of aortic dissection. CT is readily available in most Emergency Departments, and has a sensitivity of 83-98% and specificity of 87-100% for aortic dissection (highest accuracy with helical scans). Other benefits associated with the use of CT include the ability to identify intramural thrombus, pericardial effusion, and potentially reveal another etiology for the patient's pain. The major disadvantage of CT is the need for iodinated contrast, which requires normal renal function."

What does the dotted line in the figure depict? [image] Figure used with permission from Hamilton et al, Emergency Medicine: An approach to clinical problem-solving A. The approach for subcuticular suture. B. The injection plane for local anesthesia infiltration. C. The needle entry angle that optimizes eversion of sutured skin edges. D. Placement site for skin clamps. E. Use of a "finder needle" to mark suture entry points.

The answer is C. Eversion of the skin edges is maximized by directing the needle entry as shown in the figure. Injection for local anesthesia should usually be performed through the wound, rather than through intact skin. Use of skin clamps can damage tissue; in cases where skin stabilization is needed gentle forceps application is preferred. Subcuticular sutures are placed deep to the skin.

Which of the following is an absolute contraindication to surgical cricothyrotomy? A. Acute laryngeal disease B. Bleeding diathesis C. Age < 5 D. Massive neck edema

The answer is C. Given that surgical cricothyrotomy is often resorted to only after other techniques have been unsuccessful and/or the patient is not oxygenating or ventilating, most authors state that the only absolute contraindication is age. Because of the anatomic differences between children versus adults including the smaller cricothyroid membrane and the rostral funnel shaped more compliant pediatric larynx, surgical cricothyrotomy has been contraindicated in infants and young children. However, the exact age at which a surgical cricothyrotomy can be done is controversial and not well defined. Various textbooks list the lower age limit from 5 years to 12 years. Choices A, C, and D are all relative contraindications to cricothyrotomy but may be overlooked in an emergent situation when the first priority is to obtain an airway.

Of the choices below, the best treatment of the patient with hyperkalemia and EKG changes is: A. amiodarone B. defibrillation C. calcium D. vasopressin E. lidocaine

The answer is C. Hyperkalemia with EKG changes is treated with calcium to stabilize cardiac membranes. Calcium works quickly and is relatively safe unless patients are digitalized. Other treatments for acute hyperkalemia include sodium bicarbonate and insulin/glucose.

All of the following are true regarding the epidemiology of hypothyroidism EXCEPT: A. Approximately half of myxedema cases are diagnosed after admission to the hospital. B. Hypothyroidism occurs three to ten times more frequently in women than men. C. Hypothyroidism does not occur in infants under six months of age. D. Peak incidence of hypothyroidism is in the seventh decade. E. Most cases of hypothyroidism manifest in the winter months.

The answer is C. Hypothyroidism may occur at any age including the very young, but is infrequently seen in infants due to regular newborn screening for hypothyroidism. The increased frequency of the disease in women is attributed to the increased prevalence of autoimmune thyroid conditions in women. The majority of cases present in winter months due to the body's decreased ability to accommodate to cold weather in a hypothyroid state.

An 18 year old male is transported to the emergency department after being involved in a motor vehicle collision. On initial evaluation, he is found to be comatose, hypotensive, and is diagnosed clinically as having a tension pneumothorax on the left side. What is the correct statement regarding needle decompression or chest tube placement in this patient? A. Placement of the needle should be in the 3rd ICS, midaxillary line. B. The chest tube should be inserted under the lower edge of the rib C. A chest x-ray is unnecessary before needle decompression D. An 18 French chest tube would be appropriate in this situation.

The answer is C. In an unstable patient such as this, a chest x-ray would delay the care of this patient and is unnecessary. This patient needs immediate intervention with needle decompression. A chest tube size of #28 French or greater would be indicated since this patient may have a hemothorax (B). Needle decompression involves placement of a #14 gauge needle in the 2nd intercostal space at the midclavicular line (C). The chest tube should be inserted over the upper border of the rib to avoid the neurovascular bundle at the inferior margin of each rib.

A 42 year old male with end stage liver disease due to chronic hepatitis C infection arrives to the emergency department in stable condition after an unsuccessful suicide attempt by bilateral wrist laceration. He reports no history of depression or psychiatric disorder. Aside from his liver disease, for which he takes interferon alpha and ribavirin, he reports that he is in good health and takes no other medications. Which of the following factors increased this patient's risk of new-onset suicidal ideation? A. chronic hepatitis C infection B. end-stage liver disease C. interferon alfa therapy D. ribavirin therapy

The answer is C. Interferon alfa, an important cytokine in the early immune response to viral infection, has both antiproliferative and antiviral properties. It is the only therapy approved by the Food and Drug Administration for hepatitis C infection. Interferon alfa has been associated with high rates of central nervous system side effects, including anhedonia, fatigue, anorexia, impaired concentration, sleep disturbance, and suicidal ideation. Clinicians should always look up the side effects of their patients' medications, especially unfamiliar drugs. This can both expedite diagnosis of drug-induced complications and prevent them with appropriate pretreatment. Pretreatment with a selective serotonin reuptake inhibitor appears to be an effective strategy to minimize depression induced by interferon alfa. Chronic hepatitis C infection and end-stage liver disease can both be difficult diseases to live with, however, these conditions are not known to significantly increase a patients' risk for new suicidal ideation. Depression is not a known side effect of ribavirin treatment. Although males have a higher percentage of successful suicide attempt than females, females have a much higher incidence of suicide attempt and ideation than males overall.

Which of the following is true regarding the use of iodine in the treatment of thyroid storm? A. Iodine should be administered even in patients with known iodine allergy. B. Iodine should be the first drug administered in the treatment of thyroid storm. C. Iodine should be administered at least one hour after propylthiouracil (PTU) has been given. D. Iodine should be administered only after treatment with propranolol. E. Dexamethasone must be given 30 minutes prior to iodine administration.

The answer is C. Iodine inhibits preformed thyroid hormone release and should be administered at least one hour after treatment with PTU to prevent organification of iodine. A typical dose is potassium iodide (SSKI) 5 drops every 6 hours PO or NG, or sodium iodide 1 gm slow IV drip every 8 to 12 hours. Iodine should not be administered to patients with known iodine allergy.

A 7 year old girl with severe asthma presents to the emergency department in severe respiratory distress. She clearly has difficulty breathing on her own and is obviously "tiring out." Her oxygen saturation is 85% and falling. The decision is made to intubate her. Of the following agents, which is often recommended (due to its bronchodilatory effects) as the induction agent of choice? A. Pentobarbital B. Midazolam C. Ketamine D. Etomidate

The answer is C. Ketamine is a dissociative anesthetic and relaxes bronchial smooth muscle, either by blocking parasympathetic effects or by increasing sympathomimetic stimulation. This relaxation can decrease airway resistance within minutes of administration. While the clinical relevance of ketamine's bronchodilation is subject to debate, most major texts mention it as an agent of choice for intubation of patients with reactive airways disease. All of the other induction agents mentioned do not cause bronchodilation.

A 20 year old man was ice skating on a frozen pond and fell through the ice. The water was only about six feet deep and he was able to keep his head above water while bystanders were able to extract him after 10 minutes. The patient was transported to the emergency department and had an initial core temperature of 30 C. The patient's wet clothes were immediately removed and rewarming was initiated. Which of the following physical examination findings is expected? A. Hyperventilation B. shivering C. Altered mental status D. Tachycardia

The answer is C. Moderate hypothermia is associated with temperatures of 28-32 C. Shivering ceases at about 32 degrees Celsius. Moderate hypothermia is associated with altered mental status, absence of shivering, bradycardia, and bradypnea.

Which of the following drugs is MOST beneficial in an acute COPD exacerbation? A. Methylxanthines such as theophylline B. Steroids such as solumedrol C. Beta adrenergic agonists such as albuterol D. Mucokinetic agents such as acetylcysteine

The answer is C. Mucokinetic agents should not be used acutely in treatment of COPD exacerbation. These agents act to mobilize secretions, and this increases the work of the patient's breathing.

Which clinical scenario is use of chemical restraint indicated? A. Patient is uncooperative with the history B. Patient's remarks are felt to warrant negative reinforcement C. Patient's behaviors and actions pose an imminent harm to self D. Patient is responding to verbal tactics, but requires multiple attempts

The answer is C. Physical and chemical restraints should only be used when verbal tactics have failed and when the patient is an immediate threat to himself, others or the integrity of the emergency department. From a medico-legal standpoint, clear documentation is very important when a decision is made to apply restraints. Answers A, C, D, and E are all valid reasons for physical or chemical restraints.

Toxicological screening is indicated in which patient with suicidal ideation? A. Patient who threatens to cut both wrists with a knife B. Patient who ingested unknown amount ibuprofen 48 hours earlier C. Patient who ingested a "bottle" of tylenol D. Patient who takes lithium for bipolar affective disorder

The answer is C. Routine toxicological screening is unnecessary in the evaluation of suicidal patients in whom there are no clinical indications for such testing. With the exception of acetaminophen, essentially all patients with dangerous overdoses and poisoning will demonstrate clinical signs within several hours of ingestion. History, physical examination, and risk determination of suicide, however, is part of the routine evaluation of the suicidal patient

Which of the following is true regarding the treatment of acute asthma exacerbation in the Emergency Department? A. Intramuscular terbutaline is preferred over intravenous B. Heliox should only be used in the intubated patient. C. Anticholinergics by inhalation may be beneficial D. Intravenous albuterol may be indicated

The answer is C. Salmeterol is a long-acting beta2-selective adrenergic agonist that has no role in the treatment of an acute asthma exacerbation, but it is frequently preferred for outpatient asthma management due to its BID dosing schedule.

A 21-year old male presents with a clean-knife wound sustained 24 hours ago. The laceration is 2cm in length, and located between the MCP and PIP levels of the nondominant hand index finger, on the flexor surface; the wound is well-approximated (i.e., not gaping). The patient has not had tetanus immunization within 10 years and has no complaints other than pain at the laceration site. Which of the following regarding this patient/presentation is TRUE? A. The wound should be sutured, primarily to minimize the chances of infection. B. The wound should be sutured, primarily due to the potential for dehiscence due to tension on the skin edges. C. Simple wound cleaning, bandaging, and tetanus immunization are appropriate therapy for this patient. D. It is too late (at 18 hours post-injury) to provide tetanus immunization. E. Since the wound appears superficial, there is no risk of involvement of the neurovascular bundle.

The answer is C. Tetanus can be probably be effective if administered within the first few days of a wound; 24 hours is not too late. Suturing of this wound is not indicated, since it is an old wound, well-approximated, and is on the flexor surface of the digit where skin forces will be minimal. Though suturing is not indicated, careful assessment of the finger for neurovascular injury is appropriate given the anatomical location of the wound over the bundle (the superficial appearance of the wound may be misleading).

A patient with nontraumatic chest pain is administered nitroglycerin in the field and has subsequent drop in blood pressure. An EKG reveals ST-segment elevation in lead V4R. What is the diagnosis? A. unstable angina B. anteroseptal MI C. right-ventricular MI D. pericarditis E. pulmonary embolism

The answer is C. The ST-segment elevation in the right-sided lead V4R is strongly suggestive of right-ventricular MI.

In the emergency department, regarding the disposition of patients diagnosed with pneumonia: A. Patients over 50 years of age should be admitted B. Patients under 2 years of age should be admitted C. Patients with co-morbidities should be admitted D. Patients with normal vital signs should be discharged home

The answer is C. The decision whether to admit or discharge patients diagnosed with pneumonia in the emergency department is not straightforward. Despite numerous guidelines including those recommending admission for patients with co-morbid disease such as HIV, CHF, malignancy, renal disease, liver disease and others, the decision ultimately resides in the clinical judgment of the emergency department physician. Likewise, though most patients with normal vital signs are typically well enough to be treated as outpatients, circumstances may exist in which this choice is not optimal, such as an elderly patient who lives alone and has poor follow-up.

Suicide rates are consistent with the average population in this population: A. Females with breast implants B. Men with AIDS C. African-American males D. Presence of underlying panic disorder

The answer is C. The epidemiology of suicide varies according to age, gender and race. Overall, men have higher rates of completed suicide and women have greater numbers of suicide attempts. Suicide risk among men shows a bimodal distribution with increased risk between the ages of 15 and 24 and after age 65. Suicide risk among women peaks after age 60. People who suffer from a chronic disease such as AIDS are at increased risk for committing suicide. The dramatic increase in suicide rates among adolescents is thought to be due to changing demographics and greater access to firearms. Underlying psychiatric illness increases a person's risk of suicide. Major depression, bipolar, borderline personality disorder, schizophrenia, and panic disorder are all associated with increased suicide rates. Of note, One study demonstrated that 40% of people who suffer from panic attacks would attempt suicide at some point in their lives. Studies have demonstrated that the presence of breast implants portends a higher risk of suicide.

All of the following are reasonable fluids for resuscitation of hypovolemia EXCEPT: A. Blood B. Lactated Ringer's C. D5W D. Albumin E. Normal saline

The answer is C. The goal of IV resuscitation is to restore intravascular volume. Fluids that are isotonic are preferred. D5W is hyptonic, and therefore a poor choice for volume resuscitation.

A 28 year old patient arrives after helicopter transfer from an outlying center, where he had been intubated for altered mental status after significant alcohol intoxication. There were no reported signs of chest trauma, but the patient now has decreased breath sounds on the left. His vital signs are stable. Based upon the chest X-ray in the figure, what is the next step in management of this patient? [image] A. Nasogastric tube placement B. Chest tube placement C. Endotracheal tube adjustment D. Needle decompression

The answer is C. The patient has a right-mainstem intubation and resultant opacification of the left lung secondary to unilateral lack of ventilation. In an adult male, the ETT should generally be inserted to a depth (to the lip line) of 22-24cm; the corresponding depth range for an adult female is 21-23cm

A 55 year old male, who has been missing for several days in wintertime, is found in a forested area several miles away from his house. He is brought to the ED where he is found to have a core temperature of 27 degrees Celcius. He clearly has diminished mental capacity. His initial ECG demonstrates atrial fibrillation with a ventricular rate of 110. Which of the following is the best treatment option? A. Start calcium channel blockage B. Apply a Bair Hugger C. Administer warm IV fluids D. Immerse in a warm water bath at 40 Celcius

The answer is C. The patient is suffering from severe hypothermia. Atrial dysrhythmias are common below 32o C and are associated with a slow ventricular response. It usually converts spontaneously with rewarming. While answers B through E are all active rewarming techniques (active external - Bair Hugger, AVA rewarming, immersion, active core - peritoneal lavage), the best answer for someone with severe hypothermia with mental status change and cardiac dysrhythmias is probably active core rewarming . This technique minimizes rewarming collapse in patients with temperatures below 32o C. The patient will likely need intubated as ileus, bronchorrhea, and depressed protective airway reflexes are common with hypothermia.

A 24 year old woman presents with difficulty breathing after eating Chinese food. Her vital signs are: T 97.9, HR 120, BP 80/40, RR 28, SPO2 86%. Her voice is hoarse and her lung auscultation reveals wheezes. She has no JVD. The patient's most likely diagnosis is: A. Acute myocardial infarction B. Pulmonary embolism C. Anaphylactic shock D. Spinal shock E. Cardiogenic shock

The answer is C. The patient's respiratory symptoms, absence of JVD, and vital signs suggest anaphylaxis (distributive shock). The proximity of symptoms to ingestion of food also suggests an allergic reaction.

A 79 year old man is brought into the emergency department by his two daughters for shortness of breath. They state that he has become increasingly short of breath over the last several weeks, and that he has been losing weight and sleeping more over the last several months. The patient says he feels fine and wants to know when he can have a cigarette. Physical exam reveals an oxygen saturation of 94% on room air and decreased breath sounds on the right. CXR is notable for a 3 x 4 cm opaque lesion in the right upper lobe that appears to be partially constricting the right mainstem bronchus. Which of the following is true regarding this man's condition and treatment? A. He should be urgently intubated due to the partial constriction of the right mainstem bronchus B. He should undergo urgent fluoroscopic-guided needle biopsy of the lesion to attain a diagnosis C. The lesion on CXR is worrisome for malignancy and warrants a chest CT D. Pulmonary angiogram is indicated to rule out pulmonary embolus

The answer is C. This man's symptoms of gradual onset of fatigue, weight loss and shortness of breath, as well as his history of cigarette smoking, are worrisome for a lung malignancy. The finding of mass on CXR supports this theory and is best further evaluated by chest CT. Although malignancy would put this man at risk for pulmonary embolus, a chest CT is also useful in working up pulmonary embolus and is typically used prior to pulmonary angiogram. Given that this man is not in severe respiratory distress at this point, there is no indication for intubation.

A 56 year old female presents to the emergency department complaining that she can't catch her breath. She has associated intermittent sharp chest pain on the right side of her chest that began 3 days ago after she returned from a trip to Europe. She has a history of hypertension (HTN) and is on a beta blocker and hormone replacement therapy. Her physical exam is unremarkable except for a heart rate of 110 and respiratory rate of 28. Her EKG shows sinus tachycardia. Her SpO2 is 90% on 4L nasal cannula and her chest X-ray is normal. The next test should be: A. Lower extremity doppler B. Exercise treadmill C. Chest CT scan D. Bedside echocardiogram

The answer is C. This patient most likely has a PE and has a sufficient presentation to warrant immediate anticoagulation therapy with heparin unless contraindications are present. Risk factors for PE include history of deep venous thrombosis (DVT), recent surgery or pregnancy, limb immobilization, confinement to bed, or underlying malignancy. Other risk factors include HTN, obesity, estrogen replacement therapy or oral contraceptives, autoimmune diseases, and cancer. Symptoms of PE include: dyspnea, pleuritic chest pain, apprehension, cough, hemoptysis, sweating, and syncope. The diagnosis is made: (1) if DVT is demonstrated by duplex US, venography, CT, MRI or some other technique; (2) if V/Q scan is convincingly positive; or (3) if pulmonary angiography, spiral CT, or another convincing test is positive.

For a patient in neurogenic shock, the correct treatment would likely be all of the following EXCEPT: A. Phenylephrine B. High dose steroids C. Blood transfusion D. IV fluid bolus E. Spinal immobilization

The answer is C. Treatment of a spinal cord injury with neurogenic shock includes high dose steroids, IV fluids, immobilization, and potentially pressors. Blood transfusions are generally not indicated, and care must be taken to avoid fluid overload.

A 30-year old female, without past medical history, presents with "an ingrown hair" in her thigh, as depicted in the Figure. She is afebrile, nontoxic, and has no regional lymphadenopathy or lymphangitis. Examination reveals marked fluctuance and induration under the erythematous region of the thigh. Which of the following is the best course of therapy? [image] A. incision and drainage, using a cruciate incision B. needle aspiration with a 30-gauge needle followed by antibiotics and reassessment within 5 days C. CT scan of the thigh to rule-out necrotizing fasciitis D. incision and drainage with a linear incision E. antibiotics for one week, followed by reassessment

The answer is D. A fluctuant, indurated area such as that pictured and described, tends to not respond to antibiotics (which cannot penetrate well into the abscess cavity). Cruciate incisions are unnecessary and risk wound healing problems. A 30-gauge needle is too small, and needle drainage of an abscess in this location is not generally used (it is more likely appropriate in facial abscesses).

A 24 year old female gardener presents to the emergency department with foot pain 30 minutes after working barefoot in her garden. She saw a scorpion in the area. Which of the following signs or symptoms are most expected? A. Cranial nerve abnormalities B. Puncture mark C. Local erythema and swelling D. Pain and paresthesias

The answer is D. Although there are many toxic species of scorpions in the world, and all can sting humans, only a few cause serious toxicity. In the United States, only Centruroides exilicauda is capable of causing systemic toxicity. The sting is followed immediately by localized pain and paresthesias, and these can progress to involve the entire extremity or body. Systemic symptoms are unusual in adults, but more common and severe in children. Evidence of a sting, such as a puncture wound is almost never seen on exam. The mainstay of treatment is analgesia. Although antivenom is very effective in alleviating symptoms, both immediate and delayed allergic reactions occur with its use. Routine use of antivenom is not indicated, as most symptoms usually resolve in 1-2 days

A 7 year old girl is brought in by her father complaining of difficulty breathing after being stung by a bee. She is otherwise in good health. Neither she nor her father recall whether she has been stung previously, but state she has never before had a reaction like this. Which of the following treatments would be inappropriate? A. inhaled racemic epinephrine B. albuterol nebulizer C. heliox by face mask D. intravenous antibiotics

The answer is D. Antibiotics are not indicated for allergic reactions. Inhaled racemic epinephrine decreases inflammation of the airways and promotes bronchodilation. Heliox decreases turbulence of air flow to facilitate air movement into the airways. Albuterol nebulizer is a beta-agonist that causes bronchodilation, and intravenous cimetidine is an antihistamine that helps mitigate the allergic response.

A 65-year-old female presents with a chief complaint of palpitations and dyspnea on exertion. Vital signs are BP 130/84, HR 160 (and irregularly irregular), RR 14, T 37.8. EKG shows a narrow complex, irregularly irregular rhythm with absence of p-waves and an undulating baseline. What endocrine abnormality is most likely to be a direct cause of this abnormal rhythm? A. Hyperparathryoidism B. Addison's disease C. Cushing's syndrome D. Hyperthyroidism

The answer is D. Atrial fibrillation is a common arrhythmia. Its hallmark is the absence of P waves and irregular rhythm. It is associated with many medical conditions including ischemic heart disease and thyrotoxicosis. Atrial fibrillation increases the risk of thrombus formation and arterial embolism. AF's many treatment options include calcium channel blockers, beta blockers, amiodarone, quinidine, and cardioversion. Pacing is not a treatment option.

A 22 year old presents with chest pain and the following EKG: [image] He reports no past medical history and no family history of medical problems. Which substance should you specifically question him about using? A. Methamphetamine B. Heroin C. Ecstasy D. Cocaine

The answer is D. Cocaine toxicity can cause a variety of cardiovascular sequelae including: cardiac dysrhythmias, coronary artery vasospasm, myocardial ischemia/infarction, and aortic dissection. The central nervous system is also commonly involved with seizures, intracranial hemorrhages/infarctions and hypertensive encephalopathy being common. Mesenteric ischemia can occur as well as rhabdomyolysis.

A 56 year old woman presents to the emergency department with a complaint that "my head is spinning." For two days she has experienced a spinning sensation periodically, one that she does not associate with any specific position or movement. It subsides after 30 to 45 minutes. It is associated with nausea. She denies visual changes, weakness, or numbness. She feels unsteady on her feet during these episodes. On physical exam, she has horizontal nystagmus; but her neurologic exam is normal, including cranial nerves, motor strength, reflexes, coordination, and gait. Her tympanic membranes are normal; but Rinne and Weber tests reveal decreased hearing on the right with bone conduction that localized to the left ear suggesting right sensorineural hearing loss. Which of the following conditions is associated with these findings? A. brainstem stroke B. vestibular neuronitis C. multiple sclerosis D. labyrinthitis E. benign positional vertigo

The answer is D. Determination of hearing loss is important in the evaluation of vertigo. Central vertigo, such as that which may be associated with multiple sclerosis or stroke, is not accompanied by acute hearing loss because of the distributed nature of the CN VIII nuclei. Acoustic neuroma, a central process, can cause hearing loss because of direct compression of the CN VIII. Even though vestibular neuronitis involves inflammation of CN VIII, it is not associated with hearing loss. Benign positional vertigo is caused by loose particles in the semicircular canals that induce a false sense of motion; however, auditory hearing is unaffected. Labyrinthitis, or inflammation of inner ear structures including semicircular canals and cochlea, can result in sensorineural hearing loss.

A radiographic finding that may be seen in an elderly patient who aspirated food is: A. Visualization of the food bolus in the right mainstem bronchus B. An opacity in the right middle lobe bronchus C. Tension pneumothorax on the right, with leftward mediastinal shift D. Hyperinflation of the left lung

The answer is D. Due to its wider diameter and the fact it forms a more obtuse take-off angle with the trachea, the right mainstem bronchus is a more likely site of aspiration than the left mainstem bronchus in adults. On chest X-ray, aspiration is more likely to be detected because of secondary signs, such as hyperinflation or atelectasis of the involved lung, rather than direct visualization of the aspirated contents. In this clinical scenario, the adult patient may have aspirated food into the right mainstem bronchus. One would therefore expect either atelectasis (collapse) or hyperinflation on the right side. The lobar bronchus to the right middle lobe has an acute take-off angle and poor collateral ventilation, which make it prone to atelectasis. Thus, it is the part of the right lung most likely to exhibit atelectasis after an aspiration event.

A 67-year old male presents in acute respiratory failure. You have chosen etomidate as the induction agent to perform rapid sequence intubation (RSI) on your patient. He is allergic to eggs and penicillin. Which of the following is true regarding etomidate? A. Has a large side effect profile and other agents should be considered first for induction in RSI B. Is not safe to use in patients with cardiovascular disease due to its detrimental effects on myocardial contractility C. It should not be used in patients allergic to soy or eggs D. May cause transient adrenal suppression and should be used with caution in septic patients

The answer is D. Etomidate has an acceptable side effect profile and is one of the primary induction agents in RSI. It is a sedative, reduces anxiety and is cardio-protective. Therefore it is primarily indicated for induction when decreased myocardial contractility is a concern. Patients allergic to soy or eggs should not receive propofol, but etomidate is safe to administer. Etomidate has been known to cause transient adrenal suppression and its use in septic patients is controversial.

An 80 year old female presents to the ED with mental status changes after her neighbors found her this morning wandering in the stairwell. Patient was last seen normal 4 days ago and has no medical problems. On arrival to the ED, she is agitated and confused. Vital signs include RR of 20, HR of 100, BP of 90/50, and temperature of 40.6 Celsius. Pt is oriented to person only and is inattentive to exam, but appears to move all extremities symmetrically. She does not follow commands. Mucous membranes are dry and skin is dry and hot. What is the most likely diagnosis? A. Serotonin syndrome B. Encephalitis C. Thyroid storm D. Heat stroke

The answer is D. Heat stroke is a life-threatening illness defined clinically as a core body temperature that rises above 40.5 degrees Celsius and is usually accompanied by hot, dry skin (though in some cases sweating may be present) and central nervous system abnormalities such as delirium, convulsions, and coma. Treatment goals include lowering the core temperature to < 39.4 degrees Celsius by promoting cooling through conduction and evaporation and treating the complications that might arise with heat stroke, including seizures, respiratory failure, hypotension, rhabdomyolysis, and multi-organ dysfunction syndrome. Acute anticholinergic toxicity may sometimes present with a picture like that of heat stroke, and although the answer options C, D, and E may lead to confused behavior and/or fever in an elderly individual, the constellation of symptoms seen in this woman is most suggestive of heat stroke.

A 56 year old energy plant worker with a history of coronary heart disease and mild asthma severs a sharp, metal wire that snaps back and cuts his finger. He does not report a great deal of bleeding, but a 3-cm laceration on the distal right index finger requires sutures for repair. Of the approaches below, which is the best choice for pre-suturing anesthesia? A. lidocaine-epinephrine injection (lidocaine 2.0%, epinephrine 1:1000) as a digital nerve block anesthesia B. lidocaine-epinephrine injection (lidocaine 2.0%, epinephrine 1:1000) as a local infiltration anesthetic C. lidocaine-epinephrine topical solution (lidocaine 2.0%, epinephrine 1:1000) D. lidocaine injection (lidocaine 2%) as digital nerve block anesthetic

The answer is D. In some cases surgical or dermatological consultants may utilize epinephrine-containing anesthetics in tissues with end-arterial blood supply. Although epinephrine-containing solutions are used routinely by podiatrists in digital blocks of the toes, without morbidity, when performing a digital block, it is advised to use anesthetics that do not contain epinephrine. If epinephrine-containing solutions are inadvertently used for a digital block in otherwise healthy individuals without peripheral vascular disease, it is unlikely that serious ischemic injury will occur. In the emergency department you should follow the general rule proscribing use of such agents in the digits, tip of the nose, penis, and pinna. The rationale for this prohibition is that vasoconstriction in these regions can result in ischemic complications, especially if the patient has underlying peripheral vascular disease. This patient was at risk. Standard concentrations of lidocaine are not likely to achieve effective analgesia when used topically.

You have a 3 year old female present with her mother with complaints of 2 days of left ear pain. On exam, you are unable to visualize the tympanic membrane due to an obstructing mass. You suspect a foreign body. Which of the following is TRUE regarding the removing a foreign object from the patient's ear canal? A. To remove a live insect from the external ear canal, grasp a leg with hemostats and pull firmly B. You should avoid the use of lidocaine and other topical anesthetics due to the risk of localized tissue ischemia C. Avoid suction as it may lead to a perforation of the tympanic membrane D. Referral to an otolaryngologist for foreign body removal under general anesthesia may be required in an uncooperative infant

The answer is D. Irrigating the ear canal with warm water is acceptable in many instances, and may remove a foreign object with minimal discomfort. Direct the water jet gently past the object, against the tympanic membrane, and then back out the ear canal, hopefully dislodging the foreign object in the process. However, this technique should not be used in cases of bean or seed insertion because such objects swell when moistened, which makes subsequent removal more difficult and increases the risk of pressure necrosis. Irrigation should also be avoided in cases of tympanic membrane perforation.

Which finding is commonly associated with pneumonia caused by Klebsiella pneumoniae? A. Absence of sputum B. Encephalitis C. Low mortality D. Empyema

The answer is D. Klebsiella pneumoniae can cause severe pneumonia, especially in the elderly or debilitated host. The mechanism of Klebsiella pneumoniae infection leads to a necrotizing, hemorrhagic process responsible for the classic "currant-jelly" sputum. Abscess formation, empyema and bacteremia are frequent complications, and the mortality rate is high.

A 45 year-old construction worker has sustained a 4 cm, superficial laceration over his dorsal, left forearm by a segment of broken glass. Which of the following local anesthetics is characterized by average potency (lipid solubility), low toxicity and rapid onset of action? A. procaine B. tetracaine C. bupivacaine D. lidocaine

The answer is D. Lidocaine and bupivacaine are amides but the latter is much more potent, intermediate in onset and longer lasting. Procaine and tetracaine are esters and both are slow in onset, but tetracaine is applied topically and has a potency comparable to bupivacaine. Procaine is the least potent of the listed anesthetics.

A 4 year old girl is brought to the ED two hours after being stung by a scorpion while on a camping trip in Arizona. She has periods of agitation and restlessness alternating with calmness. Her vital signs are: blood pressure 106/61, pulse 120, respiratory rate 24, temperature 37.0C, and oxygen saturations of 99% on room air. On physical examination you note drooling, a disconjugate gaze, and occasional jerking movements of the extremities. Which of the following is the most correct regarding the treatment of a scorpion sting in this child? A. Treatment with antivemon is not indicated because these symptoms will be self-limiting B. Analgesics have a minimal role in controlling symptoms C. The patient should be intubated because respiratory failure is expected D. Complications of treatment with antivenom include delayed serum sickness

The answer is D. Most scorpion envenomations are mild, limited to pain and paresthesias at the site of envenomation. Children are affected more severely than adults: restlessness, jerking movements of the limbs, roving eye movements, and drooling are seen in severe cases. Anaphylaxis can also occur. Intubation is required rarely. Most envenomations require analgesics only; antivenom is indicated for severe reactions and anaphylaxis. Antivenom treatment is not without complications - serum sickness, and immediate and delayed hypersensitivity reactions occur. Without antivenom treatment, symptoms usually last for 1-2 days.

Which of the following patients is the most likely to develop S. pneumoniae pneumonia? A. 59 year old woman who is a cigarette smoker. B. 61 year old man with hypertension C. 65 year old woman with no past medical history D. 64 year old man with type 2 diabetes

The answer is D. Otitis media has not been associated with the development of S. pneumoniae pneumonia. The most common pathogen in community-acquired pneumonia, S. pneumoniae, is found in the nasopharynx of 40% of adults. Persons at risk for developing S. pneumoniae pneumonia include the elderly (>65), and those with a history of alcoholism, diabetes, cardiovascular disease, splenectomy, sickle cell disease, malignancy, and immunosuppressive disorders. Vaccination is recommended for all people at increased risk.

Regarding the development of cerebral edema in patients being treated for DKA, all of the following are true EXCEPT: A. Children have a higher incidence of cerebral edema. B. Patients with serum glucose below 250 mg/dL still being treated with insulin are most likely to develop clinically evident cerebral edema. C. Mortality of patients developing cerebral edema is 90%. D. Mannitol and steroids should be administered immediately to any patient suspected of developing cerebral edema. E. Cerebral edema typically occurs six to ten hours following onset of treatment.

The answer is D. Steroids are not indicated for treatment of cerebral edema and may actually worsen DKA. Mannitol 0.25-2.0 mg/kg should be administered upon any change in mental status of children being treated for DKA as they are at high risk for developing cerebral edema especially when being treated with insulin and serum glucose is below 250 mg/dL.

A 55-year-old female with a history of end-stage renal disease presents to the emergency department with weakness. Her electrocardiogram is shown below. What is her most likely diagnosis? [image] A. Acute MI B. Hypocalcemia C. Pericarditis D. Hyperkalemia

The answer is D. The EKG shows signs of hyperkalemia as characterized by diffuse peaked T waves. Other EKG changes include widening of the QRS complex and biphasic QRS-T segments. The heart rate may be slow, with ventricular fibrillation and cardiac arrest as the terminal events. Acute myocardial ischemia can be represented by hyperacute T waves as well, but in these cases the T wave changes are more likely to be focal (i.e. in an anatomical distribution corresponding to the area of threatened myocardium).

Of the following choices, which diagnosis is most likely in a 50-year old male with substernal chest pain and the EKG shown in the Figure? [image] A. inferior myocardial ischemia B. pulmonary embolism C. pericarditis D. right-ventricular myocardial ischemia

The answer is D. The EKG's marked ST-segment elevation in V1, in the absence of ST-segment elevation in the other anteroseptal leads (V2-V3), is suggestive of right-ventricular ischemia. Right-sided leads should be performed to further assess this possibility.

Which of the following patients presenting to the emergency department with a chief complaint of "shortness of breath" is most likely to have cavitary lesion with air-fluid levels on CXR? A. 64 year old female with history of congestive heart failure B. 54 year old male with history of recent myocardial infarction C. 32 year old female who is 36 weeks pregnant D. 43 year old homeless male with history of alcohol abuse

The answer is D. The most common pathogens to generate cavitary lesions include tuberculosis, anaerobic bacteria (common pathogens in aspiration pneumonia), aerobic gram-negative bacilli, S. Aureus, and fungal disease. The patient described in C has multiple risk factors for developing a cavitary pneumonia, including a higher risk of tuberculosis from being homeless and a higher risk of aspiration pneumonia due to alcoholism. Congestive heart failure would cause a diffuse pattern of edema on CXR

A 25 year old male college student with a history of type I diabetes mellitus presents to the emergency department complaining of worsening headache, vomiting, and fever. On exam, he has a temperature of 101.6 F, meningismus, and photophobia. A lumbar puncture is performed and reveals a CSF glucose consistent with bacterial meningitis. What is the normal ratio between CSF and serum glucose? A. 1:1 B. 2:1 C. 0.4:1 D. 0.6:1

The answer is D. The normal range of CSF glucose is 50 to 80 mg/dL, which is 60% to 70% of the glucose concentration in the blood. Ventricular fluid glucose levels are 6 to 8 mg/dL higher than in lumbar fluid. A ratio of CSF glucose-to-blood glucose of less than 0.5 or a CSF glucose level below 40 mg/dL is invariably abnormal. The ratio is higher in infants, for whom a ratio of less than 0.6 is considered abnormal. Hyperglycemia may mask a depressed CSF glucose level; when present, the CSF glucose-to-blood glucose ratio should be measured routinely. With extreme hyperglycemia, a ratio of 0.3 is abnormal. In patients with systemic hyperglycemia, the ratio changes to 0.4:1.

The X-ray in the figure indicates: [image] A. Need to withdraw the endotracheal tube from the mainstem B. A chest radiograph that was taken with the patient rotated C. Right upper lobe pneumonia D. Mediastinal shift due to fluid in the right hemithorax

The answer is D. The patient is not intubated. The pathology in the right hemithorax appears as hyperdensity, rather than air density (not a pneumothorax), and involves more than the right upper lobe.

A 5 year old male is bitten by a snake while playing along a ditch. The child is brought to the ED by his parents with complaint of fang marks to the right index finger. On physical exam, you note absence of swelling to the right hand or fingers. He does appear to have 2 small superficial fang marks, but no bleeding or oozing is present. Vital signs are normal. What is the next most appropriate step in the management of this patient? A. Admit for observation of potential compartment syndrome B. Administer prophylactic antibiotics with gram positive sensitivity C. Administer weight based antivenom in pediatric patients D. Discharge home in 8 hours if patient's exam remains unchanged

The answer is D. The patient is unlikely to suffer envenomation if he does not have any local or systemic symptoms in 8 hours. The dosage of antivenom is dependent on the degree of symptoms and children receive a proportionately higher dose compared to adults. Prophylactic antibiotics are not recommended

The components of the Figure (which is a photograph taken of the female perineal region) depict __________ (in the top of the Figure) which can be treated by placement of a __________ (in the lower part of the Figure): [image] A. an inguinal lymph node -- gel-applicator for antibiotics administration B. a benign tumor -- brachytherapy applicator C. a cystocele -- pessary D. a Bartholin's cyst -- Word catheter E. a urinoma -- pediatric Foley catheter

The answer is D. The patient's Bartholin's cyst will be drained, and placement of a Word catheter (inserted through an incision on the mucosal surface of the labia) will allow for continued drainage and healing.

Which of the following pairs of hypertension-associated disease and specific therapy represent reasonable therapeutic approaches? A. pheochromocytoma - hydrochlorothiazide B. pregnancy induced hypertension - furosemide C. angina - phentolamine D. aortic dissection - nitroprusside/propranolol E. bilateral renal artery stenosis - captopril

The answer is D. The specific utilization of various medications for the above-mentioned disease processes is subject to debate. For example, aortic dissection therapy generally includes nitroprusside and a beta-blocker, and labetalol is considered a reasonable drug of first choice for many hypertensive conditions. However, captopril is not safe in patients with renal artery stenosis. The problem with using captopril in these patients is that its mechanism of action incurs risk of renal failure in patients with some types of chronic renal disease including renal artery stenosis. Patients with pregnancy induced hypertension have a decreased intravascular volume, despite the edema, and pheochromocytoma is treated with phentolamine.

A 26 year old woman presents to the emergency department with episodes of spinning associated with nausea, vomiting, and unsteady gait. These occurred three times in the past 12 hours and come on suddenly when she is lying down and turns onto her right side. The spinning is violent and she has vomited several times. Her symptoms resolve spontaneously in 5 to 10 minutes and she feels fine in the interim. She has recently had an upper respiratory infection and has started no new medications. Her neurologic exam is normal. Laying her down quickly over the side of the bed with her head turned to the left reproduces symptoms. Which of the following medications may be effective in preventing further episodes of vertigo? A. Promethazine B. Diphenhydramine C. Meclizine D. Any of the above E. Diazepam

The answer is D. This patient has symptoms consistent with benign positional vertigo. It is caused by vestibular stimulation, usually from loose debris in the semicircular canals. Benzodiazipines are useful because of their sedative effect on the limbic system, thalamus, and hypothalamus. Vestibular neurons are mediated by acetylcholine; therefore, anticholinergic agents (e.g., meclizine, diphenhydramine, promethazine) are effective to minimize vertigo.

A 70 year old woman presents with chest pain that began 2 hours ago. She describes it as substernal radiating to her jaw and left shoulder; there is no other area of pain or radiation. She took an aspirin at home but the pain is not better. She also took 3 sublingual nitroglycerin tablets en route to the hospital. Her initial EKG shows ST elevation in the anterior leads >2mm and ST depression in the inferior leads. The nurse has already administered oxygen, placed her on an EKG monitor, and attained IV access. You order beta-blockade and nitroglycerin for pain relief, and the supervising resident asks you which of the following should be done next: A. Call her primary care physician. B. Send her to radiology for a good-quality chest X-ray. C. Give her a GI cocktail to check for pain relief from this. D. Call cardiology for a decision between thrombolytic and percutaneous coronary intervention. E. Call cardiology to request a stat echocardiogram to check for wall motion abnormalities and aortic dissection.

The answer is D. This patient is having an acute myocardial infarction. AMI is defined when two of the following three findings are present: clinical history of chest pain of at least 20 minutes duration, EKG changes and/or positive myocardial enzyme testing. This patient has ST elevation with concomitant ST depression in contiguous leads with chest pain. She needs immediate thrombolytic therapy or cardiac catheterization; if percutaneous coronary intervention (PCI) can be achieved within 90-120 minutes of emergency department arrival, the literature supports its selection over thrombolytic therapy as primary intervention. In preparation for either thrombolytic therapy or PCI, you need to control her pain, maximize O2 delivery, decrease work of the heart and inhibit platelet function. O2, nitroglycerin and morphine will increase O2 delivery to the heart. A beta blocker, which should also be administered to AMI patients who lack contraindications, will decrease the work of the heart, and aspirin will inhibit platelets. A glycoprotein IIb/IIIa-inhibitor should also be administered - selections will depend on the exact treatment course chosen for the patient. Anticoagulation with low molecular weight heparin or unfractionated heparin (dose being dependent on exact treatment course for patient) should be started if there are no patient historical or chest X-ray findings suggestive of aortic dissection.

A 24 year old woman, brought by her sister, enters the emergency department. The 24 year old is writhing in pain, clutching her abdomen and shivering. Her sister states that the patient had a therapeutic abortion performed 3 days ago and has been having worsening abdominal pain ever since. The patient's vital signs are: T 103.4 F, HR 128, BP 104/72, RR 28, O2 saturation 100% in room air and she has marked lower abdominal pain and voluntary guarding. The most appropriate steps in treatment for this woman's condition include all of the following EXCEPT: A. broad-spectrum antibiotics B. urgent ob/gyn consult to facilitate rapid transport of patient to the operating room C. laboratory studies including basic chemistry, complete blood count with differential, coagulations studies, DIC panel, serum pregnancy test, and blood cultures D. high-dose steroids E. intravenous fluids

The answer is D. This patient is most likely suffering from a septic abortion in which retained products of conception developed a local infection. This infection has now spread systemically causing the systemic inflammatory response and potentially causing sepsis. Immediate intravenous fluids and broad-spectrum antibiotics are necessary; however, steroids are contraindicated in such a situation and could worsen the infection. The patient needs to be taken to the OR urgently to remove the retained products of conception.

A 4-year old child sustained a large leg laceration while riding his bike. A medical student who was on his first clinical rotation was told to "numb up" the wound. The patient became symptomatic soon after the student gave the anesthetic. Which of the following is usually the earliest sign of lidocaine toxicity? A. Nystagmus B. Tonic-clinic seizures C. Nausea/vomiting D. Lightheadedness and dizziness

The answer is D. Toxic reactions to local anesthetics are usually due to intramuscular or intrathecal injection, or to an excessive dose. The maximal acceptable dose of lidocaine with and without epinephrine is 7mg/kg and 5mg/kg respectively. The initial symptoms of local anesthetic toxicity are lightheadedness and dizziness. Other symptoms noted are peri-oral numbness, tinnitus, progressive CNS excitatory effects including visual and auditory disturbances, shivering, twitching, and, alternatively, generalized clonic-tonic seizures. CNS depression can follow leading to respiratory depression or arrest.

Regarding the diagnosis of pulmonary embolism (PE), which is correct. A. Autospsy studies indicate that approximately 30% of patients who die in the hospital have undiagnosed PE B. By 80 years of age, one out of four people will develop deep venous thrombosis C. One-year mortality of PE is 25% even with early diagnosis and treatment D. Pulseless electrical activity in Emergency Department cardiac arrest victims is associated with PE in 33%

The answer is D. Up to two-thirds of patients with PE have no symptoms of DVT. PE is also found in about half of patients who have DVT but who do not have symptoms of PE. Thus, the presence or absence of DVT or symptoms of DVT does not correlate well with the finding of PE.

Once aortic dissection is suspected the physician should plan for early cardiothoracic surgery consultation; additionally, which of the following is the best next step? A. Start IV nitroglycerin to lower blood pressure and IV beta blocker to decrease shearing forces on the aorta. B. Start IV sodium nitroprusside to decrease shearing forces on the aorta. C. Order an MRI to characterize the dissection's anatomy. D. Start IV beta blocker to decrease shearing forces on the aorta and IV sodium nitroprusside to lower blood pressure. E. Start IV nitroglycerin to lower blood pressure and give aspirin to inhibit platelets.

The answer is D. When a patient has an aortic dissection, it is important to decrease further dissection (i.e. extension of the vascular tear) by reducing shearing forces on the aorta using negative inotropes (beta blockers) and to control hypertension. Sodium nitroprusside is often used for blood pressure control in dissections as it is an easily titratable antihypertensive. Because sodium nitroprusside increases heart rate and may increase shearing forces, a beta blocker should be started before (or concurrently with) it. The effects of nitroglycerin are not easily titratable, making it a less desirable drug for blood pressure control. Aspirin should be avoided, as it may increase bleeding complications. Morphine may be used for pain control and to decrease sympathetic tone. Imaging decisions surrounding aortic dissection are complex, incorporating such factors as patient safety (e.g. transport to imaging areas, administration of dye loads) and need for assessment of nonaortic structures (e.g. pericardial space) and functional anatomy (e.g. valvular regurgitation). As a general rule, MRI is not emergently available and lacks sufficient monitoring capabilities for a patient with suspected acute aortic dissection (MRI is useful for long-term, outpatient monitoring of dissection in most centers).

A 32 year old male, intravenous heroin abuser, presents with a one-day history of mid-back pain, progressive weakness of his legs, and an inability to urinate. He has a temperature of 38.3° C (100.8° F). On exam, absent patellar deep tendon reflexes are noted, he cannot stand or walk, a distended bladder is palpable, and he has tenderness to palpation over his T10 and T11 vertebrae. Which of the following is not an acceptable next step? A. Analgesia B. Foley catheter to drain the bladder C. Antibiotics to cover a broad spectrum of organism D. MRI of the spine E. Hospital admission for neurosurgical consultation in the morning

The answer is E. A spinal epidural abscess is a neurosurgical emergency, with the outcome being dependent on the speed of diagnosis and surgical decompression. Consequently, urgent neurosurgical evaluation is required. Although an uncommon disease, intravenous drug abuse, diabetes mellitus, chronic renal failure, and immunosuppression are risk factors for its development. Antibiotics to cover Staph. aureus, the most common cause, gram negative bacteria, and anaerobes are needed. Bladder decompression for symptomatic relief is important, as is analgesia. For further reading, please see Marx JA. Rosen's Emergency Medicine. 6th ed. 2006, page 1684.

A 14 year-old child presents to the emergency department. His blood pressure is 210/140. He complains of a headache, nausea, and recent blurred vision. Of the following choices, the best goal for lowering his mean arterial blood pressure is to have it drop by: A. 50% in the first hour B. To normal for his age in the first hour C. Until symptoms resolve D. 5% in the first 5-6 hours E. 25% in the first hour

The answer is E. A systolic BP of 210 or more, or a diastolic BP of 140 or greater, defines hypertensive urgency. With end-organs symptoms, as above, the presumptive diagnosis is hypertensive emergency. In hypertensive emergencies, the goal is to decrease mean arterial blood pressure by 10-25% within the first hour, thereby alleviating symptoms while not compromising cerebral perfusion.

Which of the following symptoms is not associated with epidural hematomas? A. Severe headache B. Neurologic deficits C. Nausea D. Sleepiness E. Hemotympanum

The answer is E. Although hemotympanum may be found in a patient with an epidural hematoma, it is specifically associated with basilar skull fracture.

Which of the following might suggest central rather than peripheral vertigo? A. Transient and episodically related to head movement B. Prominent vomiting and diaphoresis C. Sudden onset D. Horizontal nystagmus on extreme lateral gaze E. Diplopia

The answer is E. Any cranial nerve deficit should raise the suspicion for a central process as an etiology for vertigo. Some horizontal nystagmus (on extreme lateral gaze) can be a normal finding.

Regarding the treatment of suspected but not confirmed adrenal insufficiency, which of the following is most appropriate? A. withholding of steroids until confirmation of the diagnosis of adrenal insufficiency B. cosyntropin 0.25mg IV x 1 C. cortisone 100mg IM every 6 hours D. hydrocortisone 100mg IV every 6 hours E. dexamthasone 4mg IV every 6 hours

The answer is E. Dexamthasone is the treatment of choice in suspected but not confirmed adrenal insufficiency. It will not affect the serum cortisol level; therefore, it will not interfere with the diagnosis of adrenal insufficiency using the ACTH stimulation test. Administering cosyntropin, a synthetic form of ACTH, and measuring the serum cortisol levels typically perform the ACTH stimulation test. In confirmed adrenal insufficiency, hydrocortisone IV or cortisone IM are the treatments of choice.

Metabolic abnormalities often seen with hypothyroidism include all of thefollowing EXCEPT: A. anemia B. hyponatremia C. respiratory acidosis from hypoventilation D. hypercholesterolemia E. hyperglycemia

The answer is E. Hyperglycemia is not typically associated with hypothyroidism. Hypoglycemia may be present, but is unusual, and may suggest hypothalamic-pituitary involvement. Hyponatremia is common and corrects with thyroid replacement. Hypercholesterolemia to over 250 mg/dL is typical. A mild normochromic, normocytic anemia may be present, in addition to respiratory acidosis from hypoventilation.

A 65 year old male with a past medical history of poorly controlled hypertension presents with new onset unilateral arm and leg weakness. There is no disturbance of consciousness and there is no evidence of cortical findings (such as aphasia, agnosia, or hemianopsia). What is the most likely location of the vascular obstruction? A. posterior cerebral artery B. middle cerebral artery C. basilar artery D. anterior cerebral artery E. lacunar

The answer is E. Lacunar infarcts occur at the small, terminal branches of the vasculature and more commonly occur in African-Americans and patients with diabetes and hypertension. This patient's presentation, evidenced by pure loss of motor function without disturbances in other neurological modalities, is consistent with an infarct in the internal capsule. Because terminal branches of the vasculature supply the internal capsule, it is frequently affected in patients with diabetes and hypertension. A vascular obstruction in the MCA would affect not only motor functions, but also produce cortical findings such as aphasia or agnosia.

A 25 year old female presents with fever, productive cough, and shortness of breath. Physical exam reveals T101, RR 24 and SpO2 of 94%. Her left tympanic membrane is inflamed and there are TM bullae. The chest X-ray reveals a large right middle lobe pneumonia. The organism most commonly associated with this type of presentation is: A. H. influenzae B. Moraxella catarrhalis C. Legionella pneumoniae D. Strep pneumoniae E. Mycoplasma pneumoniae

The answer is E. Mycoplasma pneumoniae is a common cause of pneumonia in young adults. It is classically associated with bullous myringitis.

A 36 year old woman on chronic cyclosporine treatment for bilateral lung transplantation visits the emergency department complaining of extreme headache, nausea and vomiting. Her exam is notable for BP 239/165, normal cardiac exam, bibasilar pulmonary rales, and 1+ lower extremity edema. EKG showed asymmetric inverted T-waves in I, aVL, and V4-6. In an effort to acutely control her blood pressure, which of the following is TRUE? A. Nitroprusside would be contraindicated in this patient due to its relatively slow onset of action B. Prolonged nitroprusside therapy may potentially cause methemoglobinemia C. Esmolol works through both alpha-1 and selective beta-2 blockade D. Hydralazine decreases myocardial oxygen demand by decreasing afterload and would not be useful in this setting E. Nitroglycerin decreases BP by decreasing venous return and cardiac output

The answer is E. Relative to other anti-hypertensive agents, nitroprusside has an extremely rapid onset of action. Although rare, long-term nitroprusside treatment may lead to cyanide toxicity in renal failure patients secondary to the presence of cyanide as an intermediate metabolite. A history of long-term cyclosporine treatment suggests this patient likely has some degree of renal insufficiency.

A 2 year old male is brought to the ED in status epilepticus. He has not responded to adequate doses of benzodiazepines. Which of the following possible causes of a seizure must be evaluated for in the emergency department? A. Hypoxia B. Hypoglycemia C. Toxic ingestion D. Head trauma E. All of the above possible causes must be evaluated for

The answer is E. Seizures have a number of secondary causes, which must be identified and corrected before the seizure will end. Hypoxemia and hypoglycemia are easily detected by pulse oximetry and bedside measurement of glucose, respectively. Toddlers may ingest many toxins accidentally, such as INH, tricyclic antidepressants, and camphor. Trauma must be considered, too, including child abuse. Sickle cell disease, SLE, and leukemia are some of the medical causes of seizures and status epilepticus.

With respect to airway assessment, the Figure depicts what classification scale? [image] A. Macintosh B. Cormack-Lehane C. Miller D. LMA E. Mallampati

The answer is E. The Mallampati scale allows communication of ability to visualize structures of the posterior oropharynx, as a means of predicting ease of laryngoscopy and intubation. The Miller and Macintosh are types of laryngoscope blades, and the LMA (laryngeal mask airway) is a type of airway.

A patient presents with a question of foreign body in the foot. With reference to the figure, which of the following is true regarding anesthesia of the foot? [image] A. Local anesthesia (at the wound site) is preferred for wounds of the plantar foot. B. Adequate injection at the site depicted, will provide anesthesia for the entire sole of the foot. C. The patient is undergoing infusion of anesthesia into the saphenous vein. D. The patient shown is undergoing a sural nerve block. E. The patient shown is undergoing a posterior tibial nerve block.

The answer is E. The nerve block depicted is a posterior tibial block. Regional blocks are preferred for procedures involving the plantar foot, since there is rich innervation and significant discomfort associated with injections into the soles. The posterior tibial nerve, located between the medial malleolus and Achilles tendon, supplies the medial portion of the sole and the medial side of the foot. The nerve runs next to the posterior tibial artery and is posterior to the pulse. In addition to the posterior tibial nerve block, a sural nerve block is frequently provided when anesthesia is desired for the heel and lateral foot.

The arterial distribution in the Figure which is indicated by the letter "A", and shaded black, is the: [image] Figure used with permission from Hamilton et al, Emergency Medicine: An approach to clinical problem-solving A. internal carotid artery B. posterior cerebellar artery C. basilar artery D. anterior cerebral artery E. middle cerebral artery

The answer is E. The shaded area is the MCA. For further reading, see Hamilton et al, Emergency Medicine: An approach to clinical problem-solving (Chapter 34: Stroke).

The Figure depicts a patient with toe pain. She does not recollect any trauma. Regarding the Figure and her presentation, which of the following is true? [image] A. The cellulitis in the toe is the primary problem B. The indicated procedure will likely be able to be performed without anesthesia in this patient C. Occult trauma and fracture are the most likely diagnoses, and the patient will probably require splinting after X-ray D. This patient can be managed with a simple surgical procedure, with no need for outpatient medications other than analgesics E. Digital block, a surgical intervention, and antibiotic therapy are all indicated in this patient

The answer is E. This patient has a paronychia, complicated by relatively significant extension of purulence as well as cellulitis in the involved great toe. In cases where the paronychia is small, simple lifting of the eponychium (cuticle) may suffice; no digital block is necessary. In a case such as this one, more aggressive intervention, including removal of part of the nail, will necessitate digital block. The patient does not require hospitalization, but antibiotics and close follow-up (especially if there are complicating issues such as diabetes) are indicated.

The joint fluid from a patient's knee arthrocentesis shows 75,000 WBC with 75% PMNs, no organisms, no crystals and a glucose of 35. What does the patient need next? A. antibiotics for septic joint and admission for operation B. antibiotics and discharge home C. narcotic pain medicine and discharge home D. non-steroidal anti-inflammatory medicine for gout and discharge home E. treatment for gonorrhea and discharge home

The correct answer is A. The key to successful treatment of a septic joint is rapid diagnosis and treatment. Hospital admission for wash out of the joint and IV antibiotics are indicated for a septic joint. Antibiotics should be started based on gram stain or consideration of likely organisms and then adjusted based on final culture results.

A patient presents after slamming her index finger in a window one day ago (see Figure). Her X-ray is negative. Of the steps listed below, which is the best option for her management? [image] A. antibiotics and discharge B. drainage and discharge C. removal of the nail and suture of the underlying laceration D. splint and discharge E. none of the above

The correct answer is B. This patient has a subungual hematoma. Large subungual hematomas require drainage using an 18-gauge needle or a hot microcautery unit. There is debate concerning management of a subungual hematoma greater than 50% of the nail bed. One study showed there was a 60% incidence of nail bed laceration in these patients; therefore, the authors recommended that the nail be removed and the laceration sutured. However, another study found no difference in outcome between nail trephination alone versus formal nail bed repair.

which of the following eye drops will dilate the pupil? Answers: 1.pilocarpine 2.Pred Forte 3.proparacaine 4.sulfacetamide 5.tropicamide

Tropicamide (and homatropine) are muscarinic antagonists (mydriatic) and C/I in AACG, pilocarpine is mascarinic agonist (miotic), Pred Forte is prednisone, proparacaine is topical analgesic, sulfacetamide (antibiotic)

A patient with which of the following conditions classically presents with conjunctival injection (a "red" eye)? Answers: 1.central retinal artery occlusion 2.central retinal vein occlusion 3.retinal detachment 4.ultraviolet keratitis 5.vitreous hemorrhage

UV Keratitis

The pharmacologic interventions most likely to improve outcome when given in the field, as compared to those given upon arrival in the emergency department, include all of the following EXCEPT: Answers: 1.dextrose 2.epinephrine 3.albuterol 4.adenosine 5.diazepam

adenosine

Which of the following is FALSE concerning emergency intervention for traumatic emergencies encountered by EMS paramedics? Answers: 1.When short transport time is expected, use of pneumatic antishock garment appears to be associated with increased mortality in penetrating torso injuries. 2.Aggressive fluid resuscitation prior to surgical hemostasis is an absolute standard of care to minimize post-traumatic morbidity. 3.Severely injured patients require endotracheal intubation. 4.Intubating head injured patients may result in dental or soft tissue damage. 5.Increasing MAP to near normal levels may cause hemodilution and decreased oxygen saturation.

aggressive fluids prior to surgery

First-line interventions started by the emergency physician for suspected peptic ulcer disease (PUD) may include: Answers: 1.antacids 2.H2 blockers 3.proton pump inhibitors (PPIs) 4.stopping NSAIDs 5.all of the above

all

A 32 year old woman walks into the emergency department two weeks after a fall from a bus. She tripped stepping off the bus and landed with her right hip on the last stair. The fall was only a few feet in distance. She noted very little immediate pain, and there was no pain at all by the next day. However, the next week her right hip began to "burn." After a day's burning sensation she developed a rash involving the hip region (see Figure). She denies any chest pain, shortness of breath, nausea or vomiting. Physical examination shows a unilateral, erythematous, maculopapular rash extending from below her umbilicus to her back in a band-like pattern. There are no open lesions or cuts. What is the appropriate management of this patient? [image] Figure courtesy of eMedicine.com A. Obtain an X-ray for fat emboli. B. Discharge home with a first-generation cephalosporin for a bacterial process. C. Discharge home for rest and ice for the next few days for a late-appearing post-traumatic process. D. Discharge home with analgesics and an antiviral. E. Refer to orthopedics for evaluation of possible right hip fracture.

analgesics and antiviral; herpes zoster can be brought on by minor trauma

In dealing with the potential violent patient, the emergency physician should: Answers: 1.Approach the patient in a calm, controlled and professional manner 2.Assume that the strength of the doctor-patient relationship will ensure safety 3.Deal with the patient in a isolated room to protect the patient's privacy 4.Use a loud voice and threaten to call security if the patient becomes agitated

approach calmly

A 5 year old child was eating an almond when he experienced sudden, intermittent bouts of choking and wheezing. Assuming this child aspirated an almond, which of the following is most likely to be seen on chest X-ray? Answers: 1.A. Atelectasis of the affected lung 2.B. Diaphragmatic flattening of the non-affected lung 3.C. Foreign body in the shape of an almond 4.D. Hypoinflation of the non-affected lung

atelectasis of the affected lung

A 60 year old female presents with palpitations. Her EKG, shown below, reveals: Answers: 1.atrial flutter 2.ventricular tachycardia 3.sinus arrhythmia 4.atrial fibrillation

atrial flutter

A 60 year old male presents with new onset confusion. Which of the following suggests a functional, as opposed to an organic etiology? Answers: 1.abnormal vital signs 2.acute onset 3.auditory hallucinations 4.disorientation

auditory hal

A 20 year old college female recently returned from spring break after hiking in the Virginia woods approximately two weeks prior to her presentation to the E.D. One day prior to presentation, she developed fever, chills, and anorexia. In the emergency department, she complains of headache, photophobia, and myalgias. On exam, one notices a disseminated, non-blanching, papular rash. Which of the following statements regarding this condition is FALSE? A. A skin biopsy shows a necrotizing vasculitis. B. Characteristically the rash begins on the trunk and spreads to the extremities. C. If untreated, fatality rates range between 25-50%; however, treatment reduces the rate to 10%. D. It is caused by the tick borne parasite Rickettsia rickettsii. E. 95% of patients develop symptoms between the period of April 1 and September 30.

b

Which of the following is true regarding topical corticosteroid use? A. Hydrocortisone is the preferred agent for areas of the body characterized by thickened skin (e.g. palms and soles). B. Potency is measured by the ability to induce vasoconstriction. C. Fluorinated steroids should be used in pregnant woman. D. To achieve large differences in potency, it is more effective to vary the dose of a particular steroid than to change the type of steroid. E. Once initiating topical steroids, it is best to maintain the same application schedule without interruption until the symptoms no longer remain.

b; Corticosteroids are classified into seven groups. (I is the strongest; VII, the weakest). Each steroid's ability to cause vasoconstriction determines classification. To achieve large differences in potency, it is better to switch agents, as varying the dose of a particular agent does not affect potency as much. In general, vascoconstriction in response to corticosteroid use has been shown to decrease over time, a process known as tachyphylaxis. Therefore, interrupted application schedules are preferred (application for 2 weeks, then 1 week without application). Hydrocortisone is a relatively low potency steroid and does not adequately penetrate thickened parts of the skin such as the sole or palm. Fluorinated steroids are to be avoided in pregnant women.

A 12 year old boy complains of a pruritic rash on his inner thighs and under his axilla. The rash is unresponsive to topical and oral antihistamines. On examination, one notes circular, raised nodules on an erythematous base. The rash appears as shown in Figure A below. (Figure B depicts a hand rash of identical etiology in an older patient.) Which of the following statements is FALSE regarding this rash? [image] Photo courtesy of eMedicine.com A. The organism involved does not penetrate the dermis for it relies on oxygen for survival. B. Antibiotic therapy is contraindicated because it may exacerbate the underlying condition. C. The parasite is a mite known as Sarcopetes scabii. D. Treatment is permethrin 5% cream applied to skin for 8-12 hours. E. Often this condition occurs in young adults by sexual contact or in the elderly hospitalized population.

b; This condition is commonly associated with secondary bacterial infection and antibiotics may be indicated if signs of superinfection (i.e. surrounding erythema) are present

For several conditions of internal malignancies there may be associated cutaneous manifestations. The conditions below all describe common dermatological conditions associated with malignancy EXCEPT: A. dermatomyositis B. pruritis C. pemphigus D. acanthosis nigricans E. erythema nodosum

c

Of the following, which is most likely distributed in a Christmas-tree pattern on the posterior thorax? A. eczema B. acanthosis nigrans C. pityriasis rosea D. atopic dermatitis E. candidiasis

c

The patient in the figure presents with findings of petechiae and mucosal lesions after beginning to take a sulfonylurea for diabetes. He is nontoxic, afebrile, and has no allergic symptoms. Based upon the most likely diagnosis, which of the following statements is true? [image] A. Prothrombin time is usually twice normal. B. Intra-articular bleeding is commonly seen with this patient's condition. C. Steroids are usually indicated in adults. D. Intracranial hemorrhage is not a major concern unless the platelet count falls below 2,500/mm3. E. Platelet transfusions are indicated if platelet count is below 100,000/mm3.

c; The patient's presentation is most consistent with ITP (immune thrombocytopenic purpura). Platelet transfusions can induce inflammatory (autoantibody) response and worsen the patient's condition by increasing platelet destruction. Intracranial hemorrhage is the most feared complication of ITP, and intracranial bleeding is a risk even if levels do not fall as low as 2,500/mm3 (20,000/mm3 is a commonly cited threshold for spontaneous intracranial bleeding). The PT is expected to be normal, and intra-articular bleeding is not a major feature of ITP.

A 42 year old male presents to the emergency department with a 5-day history of a pruritic vesiculobullous rash on his right forearm. The rash has spread to involve his left palm and elbow (see Figure). He reports that he developed the rash 48 hours after working in the woods last week with his brother, who developed a similar rash. Antihistamines have decreased the pruritis, but the rash has continued to spread. Appropriate management of this patient's exposure can include all of the following EXCEPT: [image] A. continued antihistamine therapy B. drainage of large bullae for cosmetic purposes C. avoidance of contact with the rash to reduce spreading of the oleoresin antigen D. careful washing of all clothing that was worn in the woods E. oral steroid treatment

c; poison ivy, The allergen is not present in the bullae of vesicles and so, after the initial washing of the involved site, contact with the rash does not cause it to spread.,

Contact lens use predisposes patients to which condition? Answers: 1.acute angle closure glaucoma 2.central retinal vein occlusion 3.corneal ulcer 4.hyphema 5.retinal detachment

corneal ulcer

A 19 year old with eczema (atopic dermatitis) presents to the urgent care clinic demonstrating pustules within an area affected by his chronic eczema. A Tzanck preparation yields positive results, confirming the diagnosis of eczema herpeticum. Which of the following is FALSE regarding this condition? A. It can be caused by either HSV or VZV. B. Oral or, if necessary, IV acyclovir is used to treat this condition. C. It is often mistaken as an exacerbation or a superimposed impetigo infection. D. Death from this condition is very rare. E. Constitutional symptoms and adenopathy are often present.

d

A 53 year old man with non-insulin dependent diabetes mellitus presents with pain, redness, and swelling of the right foot and lower leg, accompanied by a temperature of 102. X-rays reveal no subcutaneous gas and show no evidence of osteomyelitis. The patient has been admitted on three previous occasions for cellulitis of the right foot and notes that this episode is identical. Which of the following antibiotics is the most appropriate initial therapy for a presumed diagnosis of cellulitis? A. intravenous nafcillin B. intravenous cefazolin C. oral dicloxacillin D. intravenous ampicillin-sulbactam E. intravenous gentamicin

d; Gram-positive bacteria (Streptococcus species and S. aureus) most commonly cause cellulitis in non-diabetic hosts. Penicillinase-resistant penicillins (e.g. dicloxacillin, nafcillin, oxacillin) or 1st-generation cephalosporins (cephalexin, cefazolin) can effectively treat cellulitis. However, in diabetics with recurrent cellulitis, the infection is more likely to be polymicrobial and involve gram-negative organisms. A broader-spectrum antibiotic (e.g. ampicillin-sulbactam) with coverage of gram-positive and gram-negative bacteria is recommended

A 19 year old male presents to the emergency department with allergic-mediated pruritis over large portions of his body. Which of the following is true regarding this condition? A. There is no role for therapies such as Domeboro solution (1:10 diluted aluminum sulfate soaks), potassium permanganate baths, and oat-meal baths. B. Topical antihistamine agents should be encouraged to manage the pruritis. C. Second generation antihistamines such astemizole, fexofenadine, and loratadine are more effective but cause increased levels of sedation and should be avoided if possible. D. H2 antagonists such as ranitidine or famotidine have never been shown to provide benefit. E. Oral doses of antihistamines should be encouraged initially.

e

A 45 year old male presents to the emergency department with sharp pains on the right side of the head. Upon exam, there are vesicular eruptions with crusting lesions on the patient's right forehead terminating in the patient's right eyebrow. The lesions are depicted in the figure below. The patient is extremely sensitive to light in his right eye. Which of the following is false regarding this patient? [image] Photo courtesy of eMedicine.com A. These lesions can occur anywhere on the body. B. Acyclovir is an accepted treatment of this condition. C. Treatment is most effective if given within 72 hours of when the eruption begins. D. The patient probably had chicken pox as a child E. A Tzanck preparation can distinguish Herpes simplex virus from herpes zoster virus infection.

e

A 50 year-old male presents with a chief complaint of chronic nasal itching and "a sore that won't go away" over the past few months. Of the following choices, which is the most appropriate management? [image] A. CT scan of the face for signs of nasal or facial trauma B. screening of the eyes for zoster opthalmicus C. prescription of topical steroid cream and follow-up in 5-7 days D. provision of an antibiotic-soaked dressing and reassessment in the E.D. within 48 hours E. urgent or next-day outpatient follow-up in Dermatology Clinic

e

A Wood's light is an ultraviolet source that emits light of 365nm wavelength resulting in different fluorescent patterns when directed at different organisms. Which of the following correctly matches the organism to its fluorescent pattern? A. Pseudomonas - yellow or green B. Porphyria cutanea - urine color change to orange or yellow C. Erythrasma - red or pink D. Tinea versicolor - green or yellow E. All of the above

e

Drug-induced urticaria is a common side effect of many drugs. Which statement about urticaria is FALSE? A. Nonimmunological urticaria may be caused by degranulation of mast cells. B. Drug-induced urticaria may be immunological or nonimmunological. C. Association with malignancy is not strong enough to investigate for possible cancer when urticaria of unknown origin exists. D. Penicillins and opiates are the most common offenders. E. Steroids should always be used in treatment to avoid possible anaphylaxis.

e

The rate at which a topical medication absorbs into the skin is determined in large part by the "vehicle," or medication base. Which of the following is true regarding medication bases? A. Alcohol-free bases are best for exudative lesions such as poison ivy dermatitis. B. Gels are composed of petroleum jelly and do not contain water. C. Bases containing alcohol are best for dry scaly conditions or denuded areas. D. Ointments are greaseless mixtures of propylene glycol and are contraindicated for dry lesions. E. Creams are a mixture of oils, water and preservatives and are best used for acute rather than chronic conditions.

e

A patient develops a rash that starts as multiple bilateral target-like macules and papules on the palms and soles. It progresses to widespread sloughing of the skin requiring admission to the burn unit. Which of the following etiologic agents has NOT been implicated in this syndrome? A. penicillins B. sulfa antibiotics C. barbiturates D. recent immunization E. corticosteroids

e;

A patient with no medical history presents to the emergency department for a second opinion. Nearly a week ago, there was a rapid onset of malaise, headache, backache, and fever. A rash began to appear on the leg a few days after the initial symptoms and the patient presented to a hospital-affiliated walk-in clinic for assessment. The patient had no respiratory symptoms but was noted to have some lesions on the tongue. At the time of his assessment at the walk-in clinic, the fever had resolved and the patient was diagnosed as having a viral syndrome. At the walk-in clinic, a digital photograph of the rash was taken and inserted into his computer records (see top of Figure). It is now 3 days after the initial walk-in clinic visit. You are working in the emergency department where the patient presents complaining of a recurrence of high fevers and a spread of the rash which now involves the face, extremities, and torso (see bottom of Figure). What diagnosis is most consistent with the history and accompanying images? [image] Image courtesy of eMedicine.com A. varicella-zoster virus B. measles C. insect bites with secondary infection D. erythema multiforme E. smallpox

e;

A 67 year old known alcoholic female is brought to the emergency department by EMS after being found somnolent with the odor of alcohol on her breath. After the primary survey is completed and she is stabilized, she is found to have scaly, sharply marginated, bright red eczematous plaques, vesicles and pustules in her perioral and anogenital areas (see Figure for a similar appearing rash). Consistent with this exam, she is also noted to have a red, glossy tongue and loss of her nails. With what nutritional deficiency are these findings associated? [image] A. phosphorus B. calcium C. potassium D. magnesium E. zinc

e; Low blood levels of potassium, magnesium, calcium, phosphorus, and zinc can occur as a consequence of dietary deficiency and/or acid-base imbalances in alcoholics. Hypokalemia can cause periodic muscle paralysis and areflexia. Hypocalcemia can cause tetany and weakness. Low phosphorus levels can contribute to myocardial dysfunction, CNS symptoms, muscle weakness and bleeding disorders. Hypomagnesemia can cause a clouded sensorium and other neurological deficiencies. Chronic zinc deficiency leads to the dermatologic consequences described in this patient, including patches and plaques of dry, scaly, sharply marginated and brightly red eczematous dermatitis evolving into vesiculobullous, pustular, erosive, and crusted lesions that initially involve the perioral and anogenital areas. Progression can involve the scalp, hands, feet, trunk and flexural regions. There may be diffuse alopecia and graying of remaining hair. Nail manifestations may include loss of nails or paronychia. A red, glossy tongue is common, and the oropharynx may reveal aphthous-like ulcers. Patients with zinc deficiency tend to be photophobic, irritable, and depressed. Treatment of zinc deficiency consists of dietary or intravenous zinc salt supplementation for 2-3 weeks.

A 58 year old construction worker who has no primary care doctor comes to the emergency department because of a non-pruritic rash above his ankles (see Figure). Physical examination reveals bilateral erythematous and swollen ankles that are mildly warm but non-tender to touch. His overlying skin of both legs show mild scaling and thickened skin. There is no puncture site or visible opening in the skin, although he has prominent varicose veins bilaterally (see Figure). His temperature is 37C (98.6F). He appears to be in no acute distress. What is the most likely diagnosis? [image] Figure courtesy of eMedicine.com A. eczema B. cellulitis C. psoriasis D. necrotizing fasciitis E. stasis dermatitis

e; The syndrome described is the spectrum of erythema multiforme/Stevens-Johnson syndrome/toxic epidermal necrolysis. Although the etiology is not always clear, many drugs (including sulfa antibiotics, penicillins, and barbiturates), viral or Mycoplasma infections, and recent immunization have been implicated. Corticosteroids are a controversial adjunct in treatment, but have not been implicated as a cause of the disease.

A 45 year old man is brought to the E.D., with markedly altered mental status as reported by someone who stays with him at a homeless shelter. The patient is very confused and obtunded, and unable to provide a cogent history; the person who brought him to the E.D. notes the patient has a "drinking problem." The patient's vital signs are normal, except for a respiratory rate of 22. As he lays in the stretcher, his appearance is as depicted in the Figure. Of the following choices, which physical finding is most likely to be present on physical examination? Answers: 1.Homan's sign 2.anterior chest pain upon leaning forward, which is relieved by lying flat 3.dendritic rash on the posterior thorax, with a sentinel lesion noted on the left shoulder 4.upon elevation of the arms to 90-degrees (in 0-degrees abduction), and pronation of the hands with fingers spread, wrists and interphalangeal joints are characterized by jerky alternations of extension and flexion 5.increase in systolic blood pressure by more than 10 mmHg with inspiration

elevation of hands jerky alternations

A 63 year old female presents to the ED at noon, stating that she noticed marked facial swelling (see Figure - top half) upon awakening that morning. She has breast cancer, without brain metastases on a recent MRI. She has no urticaria or respiratory symptoms. A CT scan of the chest was performed from the ED (see Figure - bottom half). Regarding this patient's condition, which of the following is true? Answers: 1.Seizures are a common presenting sign. 2.The vascular component of this problem commonly poses a threat to the patient's airway integrity. 3.Schizophreniform disorder is present when a patient meets the diagnostic criteria for schizophrenia but the process has been present for less than one year 4.Elevation of the head of the patient's bed is recommended.

elevation of patients head

Which of the following groups has an increased risk of ingested foreign body? Answers: 1.Asthmatics 2.Diabetics 3.Edentulous 4.Smokers

endentulous

A 58 year old woman presents to the emergency department one day after her cat bit her index finger. Physical examination shows signs of flexor tenosynovitis. She is admitted to the hospital for IV antibiotics, hand elevation, and emergent hand surgery consultation to consider debridement of the finger. Which of the following would be least likely expected in a patient with flexor tenosynovitis? Answers: 1.tenderness over the flexor tendon sheath 2.swelling of the finger 3.pain with passive extension 4.erythema of the flexor surface of the involved digit

erythema of flexor surface

A 19-year-old G1P0 female, at 38 weeks EGA, presents to the emergency department complaining of headache, blurry vision and leg swelling. The physical examination reveals BP of 150/100, facial and hand edema, and hyperreflexia. Fetal heart monitoring demonstrates a reassuring pattern with no uterine contractions. Urine dipstick reveals 2+ proteinuria. Of the following, which factor is the most critical in formulating an ultimate management plan for this patient? Answers: 1.Extent of maternal edema 2.Fetal age 3.Presence of variable decelerations on nonstress testing 4.Symptom duration

fetal age

A 7 year old boy presents with erythema and edema of the left eyelid. Which of the following physical examination findings is consistent with a pre-septal (or periorbital) cellulitis? Answers: 1.extraocular movement impairment 2.fever 3.pain with extraocular movement 4.proptosis 5.visual acuity impairment

fever (2) preseptal: Staph A, Strep pneum, fever, eyelid swelling, eyelid erythema, tx TMP-SMX OR Clinda PLUS amox or cefdinir....Orbital (postseptal) Cellulitis- pain, movement impairment, opthalmoplegia, proptosis, strep pneumonia, spread from sinus infection, tx with vanc and CTX

19 year old man is brought in to the emergency department by EMS after being found obtunded in his apartment by a friend. No additional history is available. On arrival, the patient is minimally responsive with sonorous respirations and a palpable rapid pulse. The most appropriate initial diagnostic test would be Answers: 1.Arterial blood gas 2.Electrocardiogram 3.Fingerstick glucose 4.Urine drug screen

fingerstick glucose

A 20 year old college student is brought to the emergency department by campus police after he was found by his roommate saying people in the TV were trying to kill him. Which of the following criteria is not an indication for admission? Answers: 1.first-time psychiatric episode 2.demonstrates risk for suicide 3.inadequate psychosocial support 4.lacks capacity to cooperate with treatment

first time

The Figure below depicts laryngoscopy and endotracheal intubation (ETI) occurring in an in-flight EMS helicopter. Regarding the patient depicted, and prehospital airway management in general, which of the following is true? Answers: 1.ETI in the helicopter cabin is technically no more difficult than it would be in the hospital emergency department. 2.For the patient in the Figure, post-intubation breath sounds will be a critical component of tube placement confirmation. 3.Postponing ETI until the aircraft is en route to the receiving center should save time when a flight crew decides a community hospital patient will require the procedure. 4.If the patient in the Figure has an easy ETI with minimal requirement for manual (bag-valve-mask) ventilation, gastric decompression (e.g. with an orogastric tube) is unnecessary. 5.Flight crew ETI success rates tend to be high in part because of their enhanced drug formulary (e.g. neuromuscular blockade) as compared to most ground EMS units.

flight crew ETI high due to advanced pharmacotherapy options

A 54 year old male comes to the emergency department complaining of intermittent pain, swelling and constant burning sensation involving his right leg. He tells you that six months ago he injured his leg in a car accident and his x-rays were negative. The symptoms have gradually worsened over the past few months. He is presenting to the emergency department because he doesn't have a primary care doctor. On physical examination you notice that his leg is edematous, erythematous, dry and warm. The leg is also characterized by hair loss, allodynia and hyperesthesia. Of the following, which is the most appropriate emergency department course? Answers: 1.Arrange follow-up for presumed complex regional pain syndrome 2.Obtain a CT scan of his leg to rule out osteomyelitis 3.Order a venogram to rule out a deep venous thrombosis 4.Perform a femoral nerve block to control his pain

follow up CRPS

Delirium is defined as: Answers: 1.a stressed psychological state resulting from extreme emotional stimulus 2.abnormal behavior associated with decreased alertness and decreased psychomotor activity 3.abnormal behavior accompanied by hallucinations, occurring in an oriented patient 4.a global inability to relate to the environment and process sensory input

global inability

In treating hepatitis secondary to alcoholic liver disease, all of the following are true EXCEPT: Answers: 1.Treatment is primarily supportive including fluids and electrolyte correction. 2.Glucose should be administered prior to thiamine to avoid precipitating acute Wernicke's encephalopathy. 3.Magnesium replacement should be initiated empirically except in the setting of contraindications such as renal failure or hypermagnesemia. 4.Coexisting gastritis should be sought out and treated appropriately. 5.Nutritional status should be assessed with attention to possible protein restriction.

glucose prior to thiamine

The primary etiology of peptic ulcer disease is: Answers: 1.Helicobactor pylori 2.cigarette smoking 3.NSAID use 4.stress 5.Zollinger-Ellison Syndrome

h pylori

A 42 year old former custodial worker presents stating, "I think that people can hear what I am thinking." In the emergency department, he becomes extremely agitated and threatening, and his psychoses become more severe. In choosing an antipsychotic medication, which of the following would be the most appropriate choice? Answers: 1.haloperidol 2.chlorpromazine 3.thioridazine 4.ketamine

haloperidol

Which medication is ideal for the agitated or combative patient? Answers: 1.Nitrous oxide 2.Hydromorphone 3.Haloperidol 4.Propofol

haloperidol

All of the following are true regarding acute hepatitis EXCEPT: Answers: 1.An IgM antibody to HAV indicates acute infection with HAV. 2.HBe antigen indicates active acute or chronic infection with HBV of high infectivity. 3.HBV core antibody indicates immunity to HBV. 4.Fecal excretion of hepatitis A virus usually occurs prior to symptoms of acute HAV infection. 5.The incubation period of HCV is 30-90 days.

hbv core

The differential diagnosis of papilledema includes all of the following EXCEPT: Answers: 1.hydrocephalus 2.hypertensive encephalopathy 3.hyphema 4.intracranial mass 5.pseudotumor cerebri

hyphema, no papilledema, anterior chamber blood

A 55 year-old male presents with new onset agitation and confusion. Which of the following medical histories would suggest a psychiatric (non-organic) cause? Answers: 1.History of diabetes mellitus only 2.History of alcohol abuse only 3.History of hypothyroidism only 4.History of chronic obstructive pulmonary disease only

hypothyroidism

Recurrent cellulitis in the distal phalanx of the right thumb in a 32 year old carpenter who is otherwise healthy should prompt the emergency room physicians to perform: Answers: 1.A. A deep, longitudinal 2-cm incision to explore the thumb for an abscess 2.B. Radiographic imaging for suspected retained foreign body 3.C. Screening for Human Immunodeficiency Virus 4.D. Testing for diabetes mellitus

imaging for suspected retained body

A 53 year-old known alcoholic presents with agitation, vomiting and altered mental status. His fingerstick glucose is 148. His serum ethanol level is undetectable and his head CT is normal. An ABG shows a pH of 7.21, pCO2 of 34, pO2 of 98 on room air. His basic chemistry panel includes a sodium of 136, potassium 4.1, chloride 108, bicarbonate 14, BUN 12, creatinine 1.1. What substance are you concerned that he may have ingested Answers: 1.Ethylene glycol 2.Salicylates 3.Isopropyl alcohol 4.Methanol

isopropyl alcohol

A 32-year-old G1P0 woman at 32 weeks gestation presents to the emergency department complaining of a worsening headache. Her vital signs are T 98.4, BP 160/115, P 95, R 16. Her urinalysis reveals 3+ protein. Which of the following is the first choice agent to decrease her blood pressure? Answers: 1.fenoldopam 2.labetolol 3.nitroglycerin 4.nitroprusside 5.phentolamine

labetolol, nitroprusside

The five diagnostic criteria for Ludwig's angina include all the following EXCEPT: Answers: 1.large amounts of visible pus in the submandibular space 2.bilateral cellulitis 3.presence of gangrene 4.sparing of the glandular tissues 5.spread of cellulitis by continuity, not by lymphatics

large amounts of visible pus in submandibular space

With regard to U.S. Emergency Medical Services (EMS), all the following are true EXCEPT: Answers: 1."First responders" are not always Emergency Medical Technicians (EMTs). 2.The levels of EMT training and EMT-level nomenclature are the same throughout the United States. 3.The Department of Transportation is the federal government agency tasked with promulgation of EMT training requirements. 4.A community is said to have "E-911" when the telephone number of a 911 caller is displayed at the operator's console. 5.Most EMTs in the field operate under off-line medical control.

levels of emt training

A 25 year old man returns to the ED, 24 hours after being released from the hospital with a new diagnosis of schizophrenia. He has recently started to take haloperidal for his psychotic symptoms. In the ED he is noted to have involuntary contractions of the muscles of the face, a protruding tongue, deviation of the head to one side, and sustained upward deviation of the eyes. Vital signs are stable, and initial labs show no electrolyte or hematological abnormalities. Of the following choices, the preferred medication for this condition is: Answers: 1.diphenhydramine 2.lorazepam 3.phenobarbital 4.metoprolol

lorazepam

A 35 year-old male is placed on his back on the gurney in physical restraints for violent behavior. Which life-threatening complication can arise? Answers: 1.circulatory obstruction 2.Metabolic acidosis 3.Asphyxia 4.Rhabdomyolysis

metabolic acidosis

A 27 year old is found unresponsive in his car in the hospital parking lot and brought in by security. During your initial evaluation you find him to be cyanotic with pulse oximetry reading 82% on room air with a respiratory rate of 4 breaths per minute. Radial pulses are present at 120 bpm. Pupils are 1mm bilaterally. Your team is having difficulty finding a vein for an intravenous line due to extensive scarring of his arms. You are suspicious of an overdose, which medication would you want to rapidly administer as a potential antidote in this situation? Answers: 1.Glucose 2.Naloxone 3.Thiamine 4.Flumazenil

naloxone

A 28 year old female is 4 months pregnant and presents with dysuria. Her UA reveals leukocyte esterase and nitrates. Of the following, the best treatment is: Answers: 1.Ciprofloxacin 2.Macrodantin (nitrofurantoin) 3.Bactrim 4.Amoxicillin 5.Doxycycline

nitrofurantoin

A patient with a history of difficult-to-control hypertension is now 6 to 8 weeks pregnant and presents with a hypertensive emergency. Which of the following IV antihypertensives should be avoided? Answers: 1.nitroprusside 2.hydralazine 3.esmolol 4.labetolol 5.nicardipine

nitroprusside

With regard to U.S. Emergency Medical Services (EMS) systems, all of the following are true EXCEPT: Answers: 1.EMS systems operated by a government agency (e.g. a city department of public health) have no malpractice liability. 2.EMS is an integral component of disaster management. 3.There is evidence that helicopter EMS transport for injured patients results in improved mortality. 4.EMS medical directors do not have to be trained, or board-certified, in emergency medicine. 5.EMS fellowships are available for both ground and air transport, however no EMS subspecialty certification exists.

no malpractice

Which of the following statements regarding use of Helicopter Emergency Medical Services (HEMS) for trauma scene transports is true? Answers: 1.If the patient is in cardiopulmonary arrest, HEMS transport should be expedited to maximize chances of patient survival. 2.The flight physician should take command of the trauma scene upon HEMS arrival. 3.The potential benefit of HEMS in improving trauma outcome remains unstudied. 4.Nonphysician crews staff most HEMS vehicles in the United States 5.Transport of a patient from a motor vehicle collision to a trauma center is termed "secondary" transport if a helicopter is used.

non-physicians crews

A 2 year old is brought in to the emergency department by his mother for difficulty breathing. The mother thinks he might have swallowed or aspirated something. In regards to this patient, which of the following is TRUE regarding foreign body aspiration or ingestion? Answers: 1.A. Large objects in the upper airway typically present with mild symptoms 2.B. Objects lodged in the proximal airway have the worst prognosis 3.C. Small objects in the lower airways typically present with the most severe symptoms 4.D. The most difficult objects to remove are stiff, non-conformable objects

objects lodged in proximal airway have the worst prognosis

Horner's syndrome is characterized by all of the following EXCEPT: Answers: 1.anhidrosis 2.enophthalmos 3.miosis 4.ophthalmoplegia 5.ptosis

opthalmoplegia; Horners miosis, pstosis, anhidrosis, and enopthalmos (posterior displacement of eyeball, apparent due to eyelid droop)

A patient presents to the emergency department with alcohol intoxication (precluding a useful history) and eye pain. The extrnal eye exam reveals an eliptically misshapen pupil, and a hyphema is also noted. A slice from a head CT (obtained due to patient's unclear history and intoxication). Of the following choices, which is the best next step for the emergency physician evaluating this patient? Answers: 1.A. palpation of the globe with mild digital pressure, to determine if increased intraocular pressure is present 2.B. performance of emergency lateral canthal tendon release 3.C. ophthalmology consultation for intraocular foreign body 4.D. topical corneal anesthesia followed by gentle sweep of a cotton swab over the eye to remove corneal foreign body

optho consult for intraocular foreign body

A 70 year old male with acute delirium requires administration of haloperidol for agitation. Which of the following is a recognized side effect of haloperidol? Answers: 1.first degree heart block 2.nephrogenic diabetes insipidus 3.prolonged QT interval 4.transient hepatitis

prolonged QT

If the parents are present and refuse treatment for their child in a life-threatening emergency, prehospital care providers should: Answers: 1.Confirm the identities of the parents and follow parental wishes 2.Provide treatment for the child 3.Contact on-line medical control for physician permission to treat 4.Contact their ambulance service's legal counsel to discuss whether to treat 5.Call the police to have the parents arrested

provide treatment

You suspect that your patient has swallowed a nail. Which of the following is an indication for endoscopic or surgical removal of this object? Answers: 1.A. Abdominal CT scan shows a 1cm nail in the distal sigmoid colon 2.B. Plain films do not reveal a radiopaque foreign body in the chest or abdomen 3.C. Radiography visualizes the nail in the gastric fundus 4.D. The object has progressed from the jejunum through the ileum after 24 hours

radiography visualization in gastric fundus

A pediatric patient is brought in by his mother, who notes he's had persistent nasal drainage. Plain films for sinusitis are obtained. Of the choices listed, which is the best next step for the patient? Answers: 1.A. Admission for IV antibiotics and oral decongestants 2.B. Discharge on antibiotics 3.C. MRI to further assess the sinuses 4.D. Removal of foreign body.

removal of foreign body

A patient presents with a self-inflicted wound, with resultant loss of vision in the right eye. With regard to the figure, which of the following statements is most likely true? Answers: 1.The patient will probably recover visual function. 2.Medial canthotomy should be performed immediately. 3.Search for other self-inflicted injuries (or ingestions) is paramount. 4.If ambulance providers contact medical control about a patient with this injury, they should be directed to replace the globe back into the orbit. 5.Life-threatening hemorrhage is a major risk with this injury.

search self inflicted

A 54 year old male presents with complaints of a foreign body sensation in his left eye. Which of the following is an indication of urgen ophthalmologic referral? Answers: 1.A. You discover a moderate-sized linear corneal abrasion 2.B. You note a small puncture wound in the globe 3.C. Your examination reveals a metallic foreign body that is loosely adherent to the white of the eye 4.D. You note an intraocular pressure of 20 in the affected eye

small puncture in globe

Management of a 4 year old child with a two-week history of malodorous purulent nasal drainage should include, as an initial step: Answers: 1.A. Oral amoxicillin for 10-14 days 2.B. Referral to an otolaryngologist 3.C. Social services consultation to investigate potential child abuse 4.D. Speculum examination of the nares

speculum exam of nares

All of the following are common pathogens in otitis media EXCEPT: Answers: 1.Strep pneumoniae 2.H. influenzae 3.Staph. aureus 4.viral agents 5.Moraxella catarrhalis

staph aureus

A "BLS" ambulance differs from an "ALS" ambulance in that the BLS ambulance: Answers: 1.is staffed by one EMT crew member (and one driver) rather than two EMTs 2.is a smaller "van"-type ambulance 3.arrives at the patient first 4.operates under off-line, as opposed to on-line, medical control 5.is stocked with different supplies and equipment

stocked different

A patient presents with itching and swelling in the leg, which is depicted in the Figure, which shows both the posterior thigh and an insect removed from the thigh. Which of the following is true regarding this patient's management? Answers: 1.Use of forceps in an attempt to remove this insect is contraindicated 2.This insect rarely transmits multiple diseases simultaneously 3.To be exposed to this insect, the patient probably has traveled out of the U.S. within the last month 4.There is a vaccine available for U.S. use, which is active against a disease transmitted by this insect

there is a vaccine available for lyme disease though rarely used

Regarding the EMS role in prehospital care, all the following are true EXCEPT: Answers: 1.Survival after cardiac arrest is less than 10% when resuscitation efforts are initiated after 10 minutes from the arrest. 2.Thrombolytic therapy in the field is the standard of care for patients suspected of having an acute coronary syndrome. 3.Pharmacotherapy that can be initiated in the field by paramedics in most jurisdictions includes naloxone for opiate overdose, diazepam for seizure control, and beta agonists for acute asthma exacerbations. 4.Automatic external defibrillators (AEDs) can be used by firefighters, policemen and other trained first responders. 5.EMS personnel evaluate and release many patients they deem well enough not to need hospital treatment.

tpa standard of care

All of the following are common non-infectious causes of fever EXCEPT: Answers: 1.Neuroleptic malignant syndrome 2.Trauma 3.CVA 4.Thyroid storm 5.Pulmonary embolism

trauma

Regarding the diagnosis of pyelonephritis, which of the following is FALSE? Answers: 1.Patients with pyelonephritis typically have symptoms for greater than 5 days. 2.White blood cell casts on urinalysis support a diagnosis of pyelonephritis. 3.Abnormal vaginal discharge is typically seen on pelvic exam in patients with pyelonephritis. 4.Diabetic patients with bacteriuria are at increased risk for developing pyelonephritis. 5.Pyelonephritis is more common in indigent populations.

vaginal discharge

What corneal pathology is predicted in a patient with a foreign body lodged in the conjunctiva of the upper eyelid? Answers: 1.branching abrasions with terminal bulbs 2.corneal ulcer directly opposite the location of the foreign body 3.dendrites 4.rust ring 5.vertical corneal abrasions

vertical corneal abrasions

A 19 year old woman is brought to the emergency department by her friends because she has been saying that she is a superhero and trying to run into traffic to prove that she is indestructible. The friends report that she has been using drugs but they do not know which ones. Which of the following pairs of ocular finding and recreational drug is commonly observed? Answers: 1.dilated pupils - heroin 2.internuclear ophthalmoplegia - marijuana 3.pinpoint pupils - amphetamines 4.vertical nystagmus - phencyclidine

vertical nystagmus PCP


Ensembles d'études connexes

Genetics CRISPER and EPIGENETICS

View Set

Chapter 23: Substance-Related and Addictive Disorders

View Set

Fundamentals Exam 1 (EBP Chapter 2)

View Set

森林之王 The King of Forest # 2

View Set

16 PROPERTY VALUATION AND APPRAISAL

View Set

The Legislative Branch: Introduction

View Set

EGEE 101 Exam 2 Review Questions

View Set

ACG 2071 Test 2- Concepts/ Definitions Only

View Set

CH. 3 practice homework questions

View Set